Расчет площади поверхности цилиндра: Площадь поверхности цилиндра формула и калькулятор онлайн

Площадь поверхности цилиндра – формула полной и боковой площади поверхности

4.6

Средняя оценка: 4.6

Всего получено оценок: 9394.

4.6

Средняя оценка: 4.6

Всего получено оценок: 9394.

Цилиндр представляет собой геометрическое тело, ограниченное двумя параллельными плоскостями и цилиндрической поверхностью. В статье поговорим о том, как найти площадь поверхности цилиндра и, применив формулу, решим для примера несколько задач.

Материал подготовлен совместно с учителем высшей категории Харитоненко Натальей Владимировной.

Опыт работы учителем математики — более 33 лет.

У цилиндра есть три поверхности: вершина, основание, и боковая поверхность.

Основаниями цилиндра (их два: верхние и нижнее) являются окружности, их легко определить.

Известно, что площадь окружности равна πr2. Поэтому, формула площади двух окружностей (двух оснований цилиндра) будет иметь вид πr2 + πr2 = 2πr2.

Боковая поверхность цилиндра

Третья, боковая поверхность цилиндра, является изогнутой стенкой цилиндра. Для того чтобы лучше представить эту поверхность попробуем преобразовать её, чтобы получить узнаваемую форму. Представьте себе, что цилиндр, это обычная консервная банка, у которой нет верхней крышки и дна. Сделаем вертикальный надрез на боковой стенке от вершины до основания банки (Шаг 1 на рисунке) и попробуем максимально раскрыть (выпрямить) полученную фигуру (Шаг 2).

После полного раскрытия полученной банки мы увидим уже знакомую фигуру (Шаг 3), это прямоугольник. Площадь прямоугольника вычислить легко. Но перед этим вернемся на мгновение к первоначальному цилиндру. Верхнее основание исходного цилиндра является окружностью, а мы знаем, что длина окружности вычисляется по формуле: L = 2πr. На рисунке она отмечена красным цветом.

Когда боковая стенка цилиндра полностью раскрыта, мы видим, что длина окружности становится длиной полученного прямоугольника. Сторонами этого прямоугольника будут длина окружности(L = 2πr) и высота цилиндра(h). Площадь прямоугольника равна произведению его сторон – S = длина х ширина = L x h = 2πr x h = 2πrh. В результате мы получили формулу для расчета площади боковой поверхности цилиндра.

Формула площади боковой поверхности цилиндра
Sбок. = 2πrh
r – радиус цилиндра, h – высота цилиндра

Площадь полной поверхности цилиндра

Наконец, если мы сложим площадь всех трёх поверхностей, мы получим формулу площади полной поверхности цилиндра. Площадь полной поверхности цилиндра равна площадь верхнего основания цилиндра + площадь нижнего основания цилиндра + площадь боковой поверхности цилиндра или S = πr2 + πr2 + 2πrh = 2πr2 + 2πrh. Иногда это выражение записывается идентичной формулой 2πr (r + h).

Формула площади полной поверхности цилиндра
S = 2πr2 + 2πrh = 2πr(r + h)
r – радиус цилиндра, h – высота цилиндра

Примеры расчета площади поверхности цилиндра

Для понимания приведенных формул, попробуем посчитать площадь поверхности цилиндра на примерах.

1. Радиус ос­но­ва­ния цилиндра равен 2, высота равна 3. Определите площадь боковой поверхности цилиндра.

Площадь боковой поверхности рассчитывается по формуле: Sбок. = 2πrh

Sбок. = 2 * 3,14 * 2 * 3

Sбок. = 6,28 * 6

Sбок. = 37,68

Площадь боковой поверхности цилиндра равна 37,68.

2. Как найти площадь поверхности цилиндра, если высота равна 4, а радиус 6?

Площадь полной поверхности рассчитывается по формуле: S = 2πr2 + 2πrh

S = 2 * 3,14 * 62 + 2 * 3,14 * 6 * 4

S = 2 * 3,14 * 36 + 2 * 3,14 * 24

S = 226,08 + 150,72

S = 376,8

Площадь поверхности цилиндра равна 376,8.

3. Площадь боковой поверхности прямого кругового цилиндра равна 24π, а диаметр основания — 3. Найдите высоту цилиндра.

Из формулы расчета площади боковой поверхности цилиндра Sбок. = 2πrh следует, что высота равна:

h = Sбок./2πr

Значение радиуса получаем из формулы: d = 2r

h = 24π / (2π * 0,5d)

h = 24π / (2π * 0,5 * 3)

h = 12 / 1,5

h = 8

Высота цилиндра равна 8.

Тест по теме

Доска почёта

Чтобы попасть сюда — пройдите тест.

  • Dark Spyro

    5/5

  • Татьяна Матвиенко

    5/5

  • Юрий Евдокимов

    5/5

  • Михаил Чепурной

    5/5

  • Дмитрий Новиков

    5/5

  • Дилноза Саипова

    4/5

  • Darya Simonova

    5/5

  • Тамара Иванова

    5/5

  • Татьяна Немчинова

    5/5

  • Рамиль Сагиров

    5/5

Оценка статьи

4.6

Средняя оценка: 4.6

Всего получено оценок: 9394.


А какая ваша оценка?

расчет боковой, полной поверхности цилиндра, формула нахождения

Математика

12. 11.21

6 мин.

Как вычислить площадь поверхности цилиндра — тема данной статьи. В любой математической задаче начать нужно с ввода данных, определить, что известно и чем оперировать в дальнейшем, и лишь затем приступить непосредственно к расчету.

Оглавление:

  • Площадь поверхности цилиндра — онлайн калькулятор
  • Площадь боковой поверхности цилиндра
  • Площадь полной поверхности цилиндра
  • Площадь цилиндра — формула через диаметр
  • Примеры расчета площади цилиндра
  • Заключение

Данное объёмное тело представляет собой геометрическую фигуру цилиндрической формы, ограниченную сверху и снизу двумя параллельными плоскостями. Если приложить немного воображения, то можно заметить, что геометрическое тело образуется вращением прямоугольника вокруг оси, причем осью является одна из его сторон.

Отсюда вытекает, что описываемая кривая сверху и снизу цилиндра будет окружностью, основным показателем которой является радиус или диаметр.

Площадь поверхности цилиндра — онлайн калькулятор

Данная функция окончательно облегчает процесс расчета, и все сводится лишь автоматическому подставлению заданных значений высоты и радиуса (диаметра) основания фигуры. Единственное, что требуется — точно определить данные и не ошибиться при вводе цифр.

Площадь боковой поверхности цилиндра

Сначала нужно представить, как выглядит развертка в двухмерном пространстве.

Это не что иное, как прямоугольник, одна сторона которого равна длине окружности. Формула ее известна с незапамятных времен —2π * r, где r — радиус окружности. Другая сторона прямоугольника равна высоте h. Найти искомое не составит труда.

Sбок = 2π * r * h,

где число π = 3.14.

Площадь полной поверхности цилиндра

Для нахождения полной площади цилиндра нужно к полученной Sбокдобавить площади двух окружностей, верха и низа цилиндра, которые считаются по формуле Sо = 2π * r2.

Конечная формула выглядит следующим образом:

Sпол = 2π * r2 + 2π * r * h.

Площадь цилиндра — формула через диаметр

Для облегчения расчетов иногда требуется произвести вычисления через диаметр. Например, имеется кусок полой трубы известного диаметра.

Не утруждая себя лишними расчетами, имеем готовую формулу. На помощь приходит алгебра за 5 класс.

Sпол = 2π * r2 + 2π * r * h = 2π * d2/4 + 2π * h * d/2 = π * d2/2 + π * d * h,

Вместо r в полную формулу нужно вставить значение r = d/2.

Примеры расчета площади цилиндра

Вооружившись знаниями, приступаем к практике.

Пример 1. Нужно вычислить площадь усеченного куска трубы, то есть цилиндра.

Имеем r = 24 mm, h = 100 mm. Использовать необходимо формулу через радиус:

Sпол = 2 * 3.14 * 242 + 2 * 3.14 * 24 * 100 = 3617,28 + 15072 = 18689,28 (мм2).

Переводим в привычные м2 и получаем 0,01868928, приблизительно 0.02 м2.

Пример 2. Требуется узнать площадь внутренней поверхности печной асбестовой трубы, стенки которой облицованы огнеупорным кирпичом.

Данные следующие: диаметр 0,2 м; высота 2 м. Используем формулу через диаметр:

Sпол = 3.14 * 0.22/2 + 3,14 * 0.2 * 2 = 0,0628 + 1.256 = 1.3188 м2.

Пример 3. Как узнать, сколько материла нужно для пошива мешка, r = 1 м и высотой 1 м.

Один момент, есть формула:

Sбок = 2 * 3. 14 * 1 * 1 = 6.28 м2.

Заключение

В конце статьи назрел вопрос: а так ли необходимы все эти вычисления и переводы одних значений в другие. Зачем все это нужно и самое главное, для кого? Но не стоит пренебрегать и забывать простые формулы из средней школы.

Мир стоял и будет стоять на элементарных познаниях, из математики, в том числе. И, приступая к какой-нибудь важной работе, никогда не лишне освежить в памяти данные выкладки, применив их на практике с большим эффектом. Точность – вежливость королей.

Не успеваете написать работу?

Заполните форму и узнайте стоимость

Вид работыПоиск информацииДипломнаяВКРМагистерскаяРефератОтчет по практикеВопросыКурсовая теорияКурсовая практикаДругоеКонтрольная работаРезюмеБизнес-планДиплом MBAЭссеЗащитная речьДиссертацияТестыЗадачиДиплом техническийПлан к дипломуКонцепция к дипломуПакет для защитыСтатьиЧасть дипломаМагистерская диссертацияКандидатская диссертация

Контактные данные — строго конфиденциальны!

Указывайте телефон без ошибок! — потребуется для входа в личный кабинет.

* Нажимая на кнопку, вы даёте согласие на обработку персональных данных и соглашаетесь с политикой конфиденциальности

Подтверждение

Ваша заявка принята.

Ей присвоен номер 0000.
Просьба при ответах не изменять тему письма и присвоенный заявке номер.
В ближайшее время мы свяжемся с Вами.

Ошибка оформления заказа

Кажется вы неправильно указали свой EMAIL, без которого мы не сможем ответить вам.
Пожалуйста проверте заполнение формы и при необходимости скорректируйте данные.

Площадь поверхности цилиндра

Горячая математика

Поскольку цилиндр тесно связана с призма , формулы их площадь поверхности относятся к.

Вспомните формулу площади боковой поверхности призмы. п час а общая площадь поверхности п час + 2 Б . Поскольку основанием цилиндра является окружность, подставляем 2 π р для п и π р 2 для Б где р это радиус основания цилиндра.

Итак, формула для площадь боковой поверхности цилиндра л . С . А . «=» 2 π р час .

Пример 1:

Найдите площадь боковой поверхности цилиндра с радиусом основания 3 дюймов и высотой 9дюймы.

л . С . А . «=» 2 π ( 3 ) ( 9 ) «=» 54 π дюймы 2

≈ 169,64 дюймы 2

Общая формула для общая площадь поверхности цилиндра Т . С . А . «=» 2 π р час + 2 π р 2 .

Пример 2:

Найдите площадь полной поверхности цилиндра с радиусом основания 5 дюймов и высотой 7 дюймы.

Т . С . А . «=» 2 π ( 5 ) ( 7 ) + 2 π ( 5 ) 2 «=» 120 π дюймы 2 ≈ 376,99 дюймы 2

Калькулятор площади поверхности цилиндра.

?
  • Какова площадь поверхности цилиндра?
  • Пример расчетов
  • Часто задаваемые вопросы
  • Этот калькулятор площади поверхности цилиндра является удобным инструментом, который быстро находит все три типа площадей поверхности:

    • Базовая площадь поверхности цилиндра;
    • Площадь боковой поверхности цилиндра; и
    • Общая площадь поверхности цилиндра.

    Цилиндр представляет собой трехмерное тело, состоящее из двух конгруэнтных поверхностей ( оснований ) и одной боковой поверхности . Хотя цилиндры могут принимать разные формы, термин 9Цилиндр 0079 обычно означает правый круговой цилиндр . Наш калькулятор площади поверхности цилиндра предназначен для этого типа цилиндра. Цилиндр прямой , когда одно из его оснований лежит точно над другим основанием, и косой , если нет. Стоит отметить, что основанием цилиндра может быть любая плоская замкнутая поверхность, например, круглый цилиндр имеет круглое основание, а прямоугольный цилиндр имеет прямоугольное основание

    Продолжайте читать, если хотите узнать, что такое площадь поверхности цилиндра и как найти площадь поверхности цилиндра. Вы также можете рассчитать другие параметры цилиндра — просто воспользуйтесь нашим правильным калькулятором цилиндров!

    Как найти площадь поверхности цилиндра?

    Чтобы оценить площадь поверхности цилиндра, вам нужно представить его как сеть . Это как если бы вы открыли цилиндр, как картонную коробку, и расплющили его. Использовать ваше воображение! И что вы получите? Ответ заключается в том, что правильный круговой цилиндр состоит из двух кругов и одного прямоугольника, как вы можете видеть на рисунке ниже.

    Следовательно, площадь основания цилиндра равна удвоенной площади круга с радиусом r , а площадь боковой поверхности цилиндра равна площади прямоугольника. Первая сторона этого прямоугольника – высота цилиндра h , а вторая – длина окружности основания, равная 2 × π × r , согласно калькулятору длины окружности.

    Какова площадь поверхности цилиндра?

    Теперь, когда мы знаем, как найти площадь поверхности цилиндра, давайте выведем соответствующие формулы для площади поверхности прямоугольного цилиндра. Чтобы вычислить площадь базовой поверхности, вам нужно вычислить площадь круга с радиусом р . Но помните, что у каждого цилиндра два основания! Таким образом, вам нужно умножить его на два:

    base_area = 2 × π × r²

    Оценка площади боковой поверхности еще проще. Поскольку площадь прямоугольника является произведением его сторон, мы можем написать, что:

    lateral_area = (2 × π × r) × h ,

    , где

    • 2 × π × r — длина окружности. опорного круга,
    • h высота цилиндра.

    Наконец, формула общей площади поверхности цилиндра представляет собой просто сумму площади основания и площади боковой поверхности :

    общая_площадь = базовая_площадь + боковая_площадь ,

    или общая_площадь = 2 × π × r² + (2 × π × r) × h ,

    или общая_площадь = 2 × π × r × (r + h) .

    С помощью калькулятора площади поверхности цилиндра вы можете выполнять все расчеты в различных единицах измерения. Если вы хотите узнать больше о преобразовании единиц площади, ознакомьтесь с нашим конвертером площади прямо сейчас!

    В расширенном режиме этого калькулятора вы также можете рассчитать объем цилиндра, но у нас также есть специальный инструмент, называемый калькулятором объема цилиндра.

    💡 Интересен тот факт, что каждый цилиндр с одинаковой высотой и площадью основания имеет одинаковый объем . Неважно, прямой это цилиндр или косой.

    Примеры расчетов

    Давайте решим несколько примеров задач с вычислением площади поверхности цилиндра.

    1. Вопрос : Какова площадь поверхности цилиндра с радиусом основания r = 2 см и высотой h = 3 см?

      • Ответ : Площадь основания равна 25,133 см², площадь боковой поверхности равна 37,7 см², а общая площадь поверхности равна 62,83 см².
    1. Вопрос : Какова площадь поверхности цилиндра с диаметром основания d = 10 см и высотой h = 5 см?

      • Ответ: Во-первых, вам нужно разделить диаметр на два, чтобы оценить радиус окружности по формуле r = d/2 = 5 см или использовать другой инструмент: вычисление окружности: найти r. Затем введите его вместе с высотой в пустые поля нашего калькулятора. В этой задаче площадь поверхности основания равна 157,08 см², площадь боковой поверхности равна 157,08 см², а площадь полной поверхности равна 314,16 см².
    1. Вопрос : Какова высота цилиндра с общей площадью поверхности 200 см² и радиусом r = 2 см?

      • Ответ : Вы можете использовать наш калькулятор площади поверхности цилиндра и в этом случае! Просто введите приведенные выше значения, и вы обнаружите, что высота равна 13,915 см.

    Часто задаваемые вопросы

    Как найти площадь боковой поверхности цилиндра?

    Чтобы найти площадь боковой поверхности цилиндра с радиусом r и высотой h , выполните следующие действия:

    1. Вычислите периметр круглого основания, используя C = 2πr .
    2. Умножьте это значение на высоту цилиндра, чтобы получить площадь его боковой поверхности, Aₗ = 2πrh .
    3. Проверьте свои результаты, используя калькулятор площади поверхности цилиндра!

    Как найти радиус по площади поверхности цилиндра?

    Чтобы найти радиус , r , цилиндра по его площади поверхности A , необходимо также знать высоту цилиндра , h :

      3 высота ч в площадь поверхности уравнения цилиндра, A = 2πr² + 2πrh .
    • Сложите всех членов этого уравнения в одну сторону, чтобы получить 2πr² + 2πrh - A = 0 . Обратите внимание, что это квадратное уравнение относительно р .
    • Решите это уравнение, используя квадратную формулу, чтобы получить r = (-2πh ± √(4π²h² + 8πA))/4π .
    • Возьмите только положительный корень этого квадратного уравнения, так как радиус должен быть положительным.

      Сколько поверхностей у цилиндра?

      Цилиндр имеет три поверхности . К ним относятся две основные круглые поверхности и одна боковая (или изогнутая) поверхность .

      Как рассчитать площадь поверхности полого цилиндра?

      Для расчета площади поверхности полого цилиндра с внутренним радиусом , rᵢ , внешним радиусом , rₒ , и высотой 06, выполните следующие действия:

      1. Рассчитайте площадь поверхности колец сверху и снизу по формуле Aᵣ = 2π(rₒ² — rᵢ²) .

    Х 8 0 реши уравнение: Реши уравнения подбирая значения х х*8=0

    2 шгарып бериндерши пожалуйста умаляю
  • Алгебра

    13 минут назад

    Допоможіть розв’язати 5х²-10=0
  • Алгебра

    18 минут назад

    185+60-2=? всем спасибо и удачки
  • Алгебра

    28 минут назад

    Позначте лінійні функціїу=-0,7х²у=х+2,7у=7ху=5х-0,7у=х³+5,2​
  • Алгебра

    33 минут назад

    Побудуйте графік функції у = -3 + х на множині натуральних чисел, менших за 8.​
  • Алгебра

    33 минут назад

    3°. x
  • Алгебра

    48 минут назад

    4. Перетворіть вираз на многочлен: 1) (2a + 3)² = 2) (5 + 4b)(4b — 5) = Відповідь: 1) 2)​
  • Алгебра

    59 минут назад

    Функция задана формулой у = –2х + 7. Определите значение функции, если значение аргумента равно 6. В ответ запишите только число

  • Алгебра

    59 минут назад

    Задайте формулой прямую пропорциональность, если её график проходит через точку М (3; -12). Укажите полученный результат.

  • Алгебра

    59 минут назад

    Задайте формулой прямую пропорциональность, если её график проходит через точку М (-1; 7). Укажите полученный результат.

  • Алгебра

    1 час назад

    В чем является особенность графика прямой пропорциональности?

    1)Всегда возрастает

    2)Графиком является точка

    3)Параллелен оси ординат

    4)Проходит через начало координат

  • Все предметы

    Выберите язык и регион

    English

    United States

    Polski

    Polska

    Português

    Brasil

    English

    India

    Türkçe

    Türkiye

    English

    Philippines

    Español

    España

    Bahasa Indonesia

    Indonesia

    Русский

    Россия

    How much to ban the user?

    1 hour 1 day 100 years

    Решение квадратных уравнений с помощью квадратной формулы

    FactoringRootsComplete the SquareGraphingExamples

    Purplemath

    Кто-то (возможно, в Индии седьмого века) решал множество квадратных уравнений, дополняя квадрат. В какой-то момент он (и да, тогда это был бы парень) заметил, что всегда выполняет одни и те же шаги в одном и том же порядке для каждого уравнения.

    Великая сила алгебры заключается в том, что она дает нам возможность работать с абстракциями, такими как формулы, которые всегда работают. Это может избавить нас от бремени и беспорядка, связанного с необходимостью возиться с цифрами каждый раз, когда мы делаем одно и то же. Используя эту способность в отношении решения квадратичных уравнений путем завершения квадрата, он составил формулу из того, что он делал; а именно, Квадратичная формула, которая гласит:

    Content Continues Below

    MathHelp.com

    The Quadratic Formula

    The Quadratic Formula: Given a quadratic equation in the following form:

    ax 2 + bx + c = 0

    …где a , b и c — числовые коэффициенты членов квадратного уравнения, значение переменной x определяется следующим уравнением:

    Преимущество квадратичной формулы в том, что она работает всегда. Есть некоторые квадратичные уравнения (на самом деле большинство из них), которые мы не можем решить с помощью факторизации. Но квадратичная формула всегда выдает ответ, независимо от того, можно ли было факторизовать квадратное выражение.

    Давайте снова попробуем решить первую задачу с предыдущей страницы, но на этот раз мы будем использовать квадратичную формулу вместо трудоемкого процесса заполнения квадрата:

    Квадратная формула требует, чтобы у меня было квадратное выражение на одной стороне Знак «равно» с «нулем» на другой стороне. Они дали мне уравнение уже в этой форме. Кроме того, формула формулируется через числовые коэффициенты членов квадратного выражения. Глядя на коэффициенты в этом уравнении, я вижу, что a  = 1, b  = -4 и c  = -8. Я подставлю эти числа в формулу и упрощу. (Я должен получить тот же ответ, что и раньше.)

    Это тот же ответ, который я получил раньше, что подтверждает, что квадратичная формула работает так, как предполагалось. Еще раз, мой окончательный ответ:


    Преимущество квадратной формулы (по сравнению с завершением квадрата) заключается в том, что мы просто подставляем формулу. Нет «шагов», которые нужно помнить, и, следовательно, меньше возможностей для ошибок. При этом:

    Не пропускайте знак «±» перед радикалом.

    Не рисуйте линию дроби только под квадратным корнем, потому что она также находится под начальной частью «− b ».

    Не забывайте, что знаменатель формулы «2 a «, а не просто «2». То есть, когда ведущий член имеет вид «5 x 2 », вам нужно будет не забыть поставить в знаменателе значение « a = 5».

    Используйте круглые скобки вокруг коэффициентов, когда вы впервые подставляете их в формулу, особенно если какой-либо из этих коэффициентов отрицательный, чтобы не потерять знаки «минус».


    Сначала я прочитаю значения коэффициентов, которые я подставлю в формулу:

    a = 4

    b = 3

    c 2 = 0 2 Теперь все, что мне нужно сделать, это подставить эти значения в формулу и упростить, чтобы получить ответ:

    x = [−(3) ± sqrt{(3) 2 − 4(4)(−2} ]/[2(4)]

    = [−3 ± sqrt{9 + 32}]/[8]

    = [−3 ± sqrt{41}]/[8]

    Здесь абсолютно ничего не упростишь, так что я закончил. Мой ответ:

    x = [−3 ± sqrt{41}]/[8]


    Вы обязательно должны выучить квадратную формулу. Меня не волнует, скажет ли ваш учитель, что она отдаст его вам на следующем тесте; в любом случае запомните его, потому что он вам понадобится позже. Он не такой длинный, и есть даже песня, которая поможет вам его запомнить, на мотив «Pop Goes the Weasel»:

    X равно отрицательному значению B
    Плюс или минус квадратный корень
    Из B в квадрате минус четыре A C
    Всего над двумя A

    (Вышеупомянутая песня не является оригинальной для меня. Я выучил ее в другом месте.)

    При использовании формулы будьте осторожны, потому что, пока вы делаете свою работу аккуратно, квадратичная формула даст вам правильный ответ каждый раз.

    У меня есть урок по квадратичной формуле, который содержит рабочие примеры и показывает связь между дискриминантом (» b 2  − 4 ac » часть квадратного корня), количество и тип решений квадратного уравнения и график соответствующей параболы. Если вам нужна дополнительная помощь по формуле, то пожалуйста, изучите урок по гиперссылке выше.


    Вы можете использовать виджет Mathway ниже, чтобы попрактиковаться в решении квадратных уравнений с помощью квадратной формулы. Попробуйте введенное упражнение или введите свое собственное упражнение. Затем нажмите кнопку и выберите «Решить с помощью квадратичной формулы», чтобы сравнить свой ответ с ответом Mathway. (Или пропустить виджет и продолжить на следующей странице.)

    Пожалуйста, примите куки-файлы настроек, чтобы включить этот виджет.

    (Нажмите «Нажмите, чтобы просмотреть шаги», чтобы перейти непосредственно на сайт Mathway для платного обновления.)



    Страница 1Страница 2Страница 3Страница 5Страница 6

    урок-8-8-практика-б-заполнение-квадрата-ответы0159

    [PDF] Практика B 9-8

    cboy.noip.me › Работа › Холт › chap09 › section08 › практика_b

    Решите, заполнив квадрат. Округлите ответы до ближайшей десятой доли фута. 14. Небольшая картина имеет площадь 400 см 2 . Длина …

    [PDF] Завершение квадрата — г-жа Болюс — интегрированная математика 1 и 2

    bolusmath.weebly.com › uploads › complete_the_square

    Решите каждое уравнение, заполнив квадрат. … 8. х2 + 6х + 34 = 0 … б. Не существует действительного числа, квадрат которого отрицателен. УРОК 9–8. Практика A.

    [PDF] 5-4 Практика B

    asb-bangna-highschoolmath.weebly.com › 8 › 5.4_practice_b.pdf

    07.12.2005 · УРОК. Практика B. Дата. Сорт. Заполнение квадрата… Решите каждое уравнение, заполнив квадрат. 5. 2d2. = 8 + 10д.

    [PDF] Решите квадратные уравнения, заполнив квадрат.

    www.twinsburg.k12.oh.us › Downloads

    8-8 Завершение квадрата. На предыдущем уроке вы решали квадратные уравнения, выделяя х2, а затем используя квадратные корни.

    Ähnliche Fragen

    Как составить формулу квадрата?

    Как решить дополнить квадрат коэффициентами?

    Какой средний член завершает квадрат?

    [PDF] 8. 8 Практика B

    www.twinsburg.k12.oh.us › Загрузки › 8-8 HW Практика B

    Длина на 10 футов больше ширины. Найдите размеры бассейна. Решите, заполнив квадрат. Округлите ответы до ближайшей десятой доли фута.

    Практика B 9-8 — Yumpu

    www.yumpu.com › документ › просмотреть › практика-b-9-8

    13.11.2012 · TEKS A.10.A. УРОК. 9-8. Практика Б. Завершение квадрата. Дополните квадрат до… Округлите ответы до ближайшей десятой части. сантиметр.

    [PDF] 9-7 Завершение квадрата — Плезантон Moodle

    moodle.pleasantonusd.net › mod_page › content › Answers9-7

    УРОК. Упражняться. 9-7 Завершение квадрата. Заполните квадрат, чтобы сформировать … Решите каждое уравнение, заполнив квадрат. Решить по. 4. х² + 6х = -8. -8.

    [PDF] Практика B | smilardo

    smilardo.files.wordpress.com › 2015/06 › alg-1-9-7-practice-b

    Практика B. Решение квадратных уравнений с использованием квадратных корней.

    Вероятностей формула: Теория вероятностей — урок. Основной государственный экзамен 9 класс, Математика.

    Теория вероятностей и математическая статистика на примерах. Что это такое, основные формулы, теории

    Данная статья является переводом. Ссылка на оригинальную статью.

    ❓ Что такое теория вероятностей?

    Теория вероятностей использует случайные величины и распределения вероятностей для математической оценки неопределенных ситуаций. Понятие вероятности используется для присвоения числового описания вероятности наступления события. Вероятность можно определить как число благоприятных исходов, деленное на общее число возможных исходов события.

    Определение теории вероятностей

    Теория вероятностей – это область математики и статистики, которая занимается определением вероятностей, связанных со случайными событиями. Существует два основных подхода к изучению теории вероятностей: теоретический и экспериментальный. Теоретическая вероятность определяется на основе логических рассуждений без проведения экспериментов. В отличие от нее, экспериментальная вероятность определяется на основе исторических данных путем проведения повторных экспериментов.

    Пример теории вероятностей

    Предположим, нам необходимо определить вероятность выпадения числа 4 при бросании игральной кости. Число благоприятных исходов равно 1. Возможные исходы игральной кости – {1, 2, 3, 4, 5, 6}. Из этого следует, что всего существует 6 исходов. Таким образом, вероятность выпадения 4 при бросании игральной кости, используя теорию вероятности, можно вычислить как 1 / 6 ≈ 0,167.

    🎲 Основы теории вероятностей

    Мы можем понять эту область математики с помощью нескольких основных терминов, напрямую связанных с теорией вероятностей.

    Случайный эксперимент

    Случайный эксперимент в теории вероятностей – это испытание, которое повторяется несколько раз для получения четко определенного набора возможных результатов. Подбрасывание монеты является примером случайного эксперимента.

    Пространство выборки

    Пространство выборки можно определить как множество всех возможных исходов, полученных в результате проведения случайного эксперимента. Например, пространство выборки при подбрасывании симметричной монеты (fair coin), стороны которой – это орел и решка.

    Событие

    Теория вероятностей определяет событие как набор исходов эксперимента, который образует подмножество пространства выборки.

    Примеры событий:

    1. Независимые – те, на которые не влияют другие события, являются независимыми.
    2. Зависимые – те, на которые влияют другие события.
    3. Взаимоисключающие – события, которые не могут произойти в одно и то же время.
    4. Равновероятные – два или более события, которые имеют одинаковые шансы произойти.
    5. Исчерпывающие – это события, которые равны выборочному пространству эксперимента.

    Случайная величина

    В теории вероятностей случайную переменную можно определить как величину, которая принимает значение при всех возможных исходах эксперимента.

    Существует два типа случайных величин:

    1. Дискретная случайная величина – принимает точные значения, такие как 0, 1, 2…. Описывается кумулятивной функцией распределения и функцией массы вероятности.
    2. Непрерывная случайная величина – переменная, которая может принимать бесконечное число значений. Для определения характеристик этой переменной используются кумулятивная функция распределения и функция плотности вероятности.

    Вероятность

    Вероятность мы можем определить как численную вероятность наступления события. Вероятность того, что событие произойдет, всегда лежит между 0 и 1. Это связано с тем, что число желаемых исходов никогда не может превысить общее число исходов события. Теоретическая вероятность и эмпирическая вероятность используются в теории вероятностей для измерения шанса наступления события.

    Формула вероятности P(A): количество благоприятных исходов для A делимое на общее количество возможных исходов.

    Условная вероятность

    Ситуация, когда необходимо определить вероятность наступления события, притом что другое событие уже произошло.

    Обозначается как P(A | B).

    Если хочешь подтянуть свои знания по математике, загляни на наш курс «Математика для Data Science», на котором ты:

    • Усвоишь специальную терминологию и сможешь читать статьи по Data Science без постоянных обращений к поисковику.
    • Подготовишься к успешной сдачи вступительных экзаменов в Школу анализа данных Яндекс.
    • Овладеешь математическим аппаратом, который необходим, чтобы стать специалистом в Data Science.

    Интересно, хочу попробовать

    Ожидание

    Ожидание случайной величины X можно определить как среднее значение результатов эксперимента, проводимого многократно. Ожидание обозначается как E[X]. Также известно как среднее значение случайной величины.

    Дисперсия

    Дисперсия – это мера, которая показывает, как распределение случайной величины изменяется относительно среднего значения. Дисперсия определяется как среднее квадратичное отклонение от среднего значения случайной величины. Обозначается как Var[X].

    Функция распределения теории вероятностей

    Распределение вероятностей или кумулятивная функция распределения – это функция, которая моделирует все возможные значения эксперимента, используя случайную переменную. Распределение Бернулли и биномиальное распределение – это примеры дискретных распределений вероятностей. Например, нормальное распределение представляет собой пример непрерывного распределения.

    Массовая функция вероятности

    Массовая функция вероятности определяется как вероятность того, что дискретная случайная величина будет в точности равна определенному значению.

    Функция плотности вероятности

    Функция плотности вероятности – это вероятность того, что непрерывная случайная величина принимает множество возможных значений.

    Формулы теории вероятностей

    В теории вероятностей существует множество формул, которые помогают рассчитать различные вероятности, связанные с событиями.

    Наиболее важные формулы:

    1. Теоретическая вероятность: Число благоприятных исходов / Число возможных исходов.
    2. Эмпирическая вероятность: Число случаев, когда событие происходит / Общее число испытаний.
    3. Правило сложения: P(A ∪ B) = P(A) + P(B) – P(A∩B), где A и B – события.
    4. Правило комплементарности: P(A’) = 1 – P(A). P(A’) означает вероятность того, что событие не произойдет.
    5. Независимые события: P(A∩B) = P(A) ⋅ P(B).
    6. Условная вероятность: P(A | B) = P(A∩B) / P(B).
    7. Теорема Байеса: P(A | B) = P(B | A) ⋅ P(A) / P(B).
    8. Массовая функция вероятности: f(x) = P(X = x).
    9. Функция плотности вероятности: p(x) = p(x) = dF(x) / dx, где F(x) – кумулятивная функция распределения.
    10. Ожидание непрерывной случайной величины: ∫xf(x)dx, где f(x) является МФВ (Массовой функцией вероятности).
    11. Ожидание дискретной случайной величины: ∑xp(x), где p(x) – это ФПВ (Функцией плотности вероятности).
    12. Дисперсия: Var(X) = E[X2] – (E[X])2.

    Применение теории вероятностей

    Теория вероятностей используется во многих областях и помогает оценить риски, которые связаны с теми или иными решениями. Некоторые из направлений, где применяют теорию вероятностей:

    • В финансовой отрасли теория вероятностей используется для создания математических моделей фондового рынка с целью прогнозирования будущих тенденций. Это помогает инвесторам вкладывать средства в наименее рискованные активы, которые дают наилучший доход.
    • В потребительской индустрии теория вероятностей используется для снижения вероятности неудачи при разработке продукта.
    • Казино использует теорию вероятностей для разработки азартных игр с максимизацией своей прибыли.

    🏋️ Практические задания

    Задача 1: При бросании двух игральных костей, какова вероятность того, что выпадет комбинация, сумма которой будет равна 8?

    Решение

    При бросании двух игральных костей существует 36 возможных исходов. Для получения суммы, равной 8, существует 5 благоприятных исходов: [(2, 6), (6, 2), (3, 5), (5, 3), (4, 4)]. Используя формулы теории вероятностей: Вероятность = Число благоприятных исходов / общее число возможных исходов = 5 / 36. Ответ: Вероятность получения суммы 8 при бросании двух игральных костей равна 5 / 36.

    Задача 2: Какова вероятность вытащить карту королеву из колоды?

    Решение

    Колода карт имеет 4 масти. Каждая масть состоит из 13 карт. Таким образом, общее число возможных исходов = (4) * (13) = 52. Может быть, 4 королевы, по одной из каждой масти. Следовательно, количество благоприятных исходов = 4. Карточная вероятность = 4 / 52 = 1 / 13. Ответ: Вероятность получить королеву из колоды карт равна 1 / 13

    Задача 3: Из 10 человек 3 купили карандаши, 5 купили тетради, а 2 купили и карандаши, и тетради. Если покупатель купил тетрадь, какова вероятность того, что он также купил карандаш?

    Решение

    Используя понятие условной вероятности, P(A | B) = P(A∩B) / P(B). Пусть A – событие, когда люди покупают карандаши, а B – событие, когда люди покупают тетради. P(A) = 3 / 10 = 0,3P(B) = 5 / 10 = 0,5P(A∩B) = 2 / 10 = 0,2. Подставим полученные значения в приведенную формулу, P(A | B) = 0,2 / 0,5 = 0,4. Ответ: Вероятность того, что покупатель купил карандаш, при условии, что он купил блокнот, равна 0,4.

    В заключение

    Подведем итоги:

    • Теория вероятностей – это раздел математики, в котором рассматриваются вероятности случайных событий.
    • Понятие вероятности объясняет возможность наступления того или иного события.
    • Значение вероятности всегда лежит между 0 и 1.
    • В теории вероятностей все возможные исходы случайного эксперимента составляют пространство выборки.
    • Теория вероятностей использует такие важные понятия, как случайные величины и кумулятивные функции распределения для моделирования случайного события. Сюда же относится определение различных вероятностей, связанных с этим.

    Если хочешь подтянуть свои знания по математике, загляни на наш курс «Математика для Data Science», который включает в себя:

    • 47 видеолекций и 150 практических заданий.
    • Консультации с преподавателями курса.

    Интересно, хочу попробовать

    Теория вероятностей: формулы, виды событий, алгебра событий и решение задач

    Теория вероятностей (разг. сокр. “тервер”) — это раздел математики, который занимается анализом случайных событий. С её помощью можно вычислить вероятность события — оно показывает насколько вероятно, что какое-то событие произойдёт. Это число всегда находится в интервале между 0 и 1, где 0 — означает невозможность, а 1 — оно точно произойдёт (достоверное событие).

    Например: в мешке есть 6 шаров: 3 красных, 2 жёлтых и 1 синий. Какова вероятность вытащить красный?

    Вероятность считается так: количество красных шаров поделить на общее количество шаров в мешке, т. е. 3/6 = 1/2.

    Основные формулы теории вероятностей

    Теоремы сложения и умножения вероятностей

    ПрименениеФормула
    Сложение противоположных событийP(A) + P(A̅) = 1
    Сложение несовместных событийP(A + B) = P(A) + P(B)
    Сложение совместных событийP(A + B) = P(A) + P(B) — P(AB)
    Умножение независимых событийP(AB) = P(A) × P(B)

    Основные формулы вычисления

    НазваниеФормулаПрименение/Пояснение
    Классическое определение вероятности

    Где m — количество элементарных событий, благоприятствующих событию А, и n — число всех элементарных событий данного испытания.
    Комбинаторика — Размещение

    Соединения, в которых каждое содержит m элементов (без повторений между ними), взятых из числа данных n элементов.
    Комбинаторика — Размещения с повторениями

    Число размещений с повторениями из n элементов по m элементов; соединения могут отличаться только порядком расположения элементов, но из m каких угодно и как угодно повторяющихся элементов.
    Комбинаторика — Сочетания

    , где 0 ≤ m ≤ n

    Соединения, в которых каждое содержит m элементов, взятых из числа данных n элементов; применяется когда порядок безразличен.
    Перестановки

    Соединения содержат все n элементов, отличие лишь в порядке их расположения.

    Виды событий

    В теории вероятностей события бывают невозможными, случайными и достоверными.

    Невозможное событие

    Это то, которое уже известно, что в ходе испытания НЕ произойдёт, т. е. вероятность данного события равна нулю. Например: при бросании одной игральной кости (один раз), какова вероятность того, что выпадет 7 очков?

    Случайное событие

    Это событие может произойти или нет, обычно оно именно случайное. Например: при бросании игральной кости, какова вероятность того, что выпадет чётное число очков?

    Достоверное событие

    Это то, которое в ходе испытания обязательно произойдёт, т. е. вероятность данного события равна 1. Например: при бросании игральной кости, какова вероятность того, что она не останется в воздухе, а упадёт?

    Совместные и несовместные события

    Несовместные события — это когда появление одного исключает появление другого (в одном и том же испытании). Например: при бросании одной игральной кости выпадет одновременно и «2» и «3»?

    Совместные события могут произойти одновременно. Например: два спортсмена плывут одновременно, два студента сдают экзамен.

    Противоположные события

    Это два несовместимых события, которые образуют полную группу событий (третьего не существует). Например:

    • А — при подбрасывании монеты выпадет орёл, A̅ — при подбрасывании монеты выпадет решка;
    • D — из колоды карт будет извлечена дама, D̅ — из колоды карт будет извлечена не дама.

    Алгебра событий

    Логическое ИЛИ означает, что нужно произвести операцию сложения (сумма событий). Т. е. считаем возможность или событие А, или событие В, или оба (одновременно).

    Логическое И — операция умножения (произведение событий). Т. е. считаем возможность и событие А, и событие В.

    Задачи

    Пример 1

    В классе 27 учеников. Из них:

    17 изучали немецкий язык,

    6 — английский,

    2 — оба языка.

    Найти вероятность того, что случайно выбранный ученик изучал хотя бы один язык.

    Что мы знаем:

    𝑃(N) = 17/27,

    𝑃(A) = 6/27,

    𝑃(N ∙ A) = 2/27.

    Значит вместе это будет:

    𝑃(N + A) = 𝑃(N) + 𝑃(A) − 𝑃(N ∙ A) = 17/27 + 6/27 − 2/27 = 21/27 = 7/9.

    Пример 2

    Лотерейные билеты пронумерованы от 1 до 100. Какова вероятность того, что в выбранном билете будет стоять число больше 40 или чётное число?

    Что мы знаем:

    P(>40) = 60/100 = 6/10 = 3/5

    P(Ch) = ½ = 5/10

    Логическое ИЛИ означает, что нам нужно произвести операцию сложения (т. е. сумма событий).

    Нам понадобится формула сложения совместных событий P(A + B) = P(A) + P(B) — P(AB).

    Для этого нам нужно узнать сколько будет P(>40 . Ch), для этого используем формулу P(AB) = P(A) . P(B).

    P(>40 . Ch) = P(>40) . P(Ch) = ⅗ . ½ = 3/10

    Теперь можем подставить всё в формулу P(A + B) = P(A) + P(B) — P(AB):

    P(>40 + Ch) = P(>40) + P(Ch) — P(>40.Ch) = 6/10 + 5/10 — 3/10 = 8/10 = ⅘.

    Пример 3

    В финале международного турнира по стрельбе из лука участвовали 8 спортсменов: 3 американца, 1 англичанин, 1 немец, 1 француз и 2 русских. Какова вероятность того, что хотя бы один русский попадёт в тройку лучших, учитывая, что все спортсмены имеют равные условия для получения медали (золотой, серебряной и бронзовой).

    Что мы знаем:

    Когда в вопросе появляется «хотя бы один», можно «пойти от противного» — мы должны найти вероятность того, что этого не произойдёт (на пьедестале русских не будет), а затем вычесть это из 1.

    P (никакой русский не выиграет золото) = 6/8 = 3/4

    P (никакой русский не выиграет серебро) = 5/7 (убираем золотую медаль)

    P (никакой русский не выиграет бронзу) = 4/6 = 2/3 (убираем золотую и серебряную медали)

    P (на пьедестале не будет русских) = 3/4 x 5/7 x 2/3 = 30/84 = 5/14

    P (хотя бы один русский на пьедестале) = 1 – 5/14 = 14/14 – 5/14 = 9/14.

    Кто придумал теорию вероятностей

    Основателями теории вероятностей являются два французских математика Блез Паскаль и Пьер Ферма. В 1654 г. французский писатель Антуан Гомбо (известный как Шевалье де Мере), интересовавшийся игрой и азартными играми, вызвал заинтересованность Паскаля насчёт популярной в то время игры в кости.

    Кости бросались 24 раза, а вопрос стоял в том, стоит ли ставить деньги на выпадение хотя бы одной «двойной шестёрки». В то время считалось, что это было выгодно, но последующие расчёты показали прямо противоположное.

    Узнайте про Метод Крамера, Интегралы, Корреляции, Математическое ожидание, Стандартное отклонение и Космологию.

    Формула, определение, теоремы, типы, примеры

    Вероятность определяет вероятность возникновения события. В реальной жизни существует множество ситуаций, в которых нам, возможно, придется предсказывать исход события. Мы можем быть уверены или не уверены в результатах события. В таких случаях мы говорим, что существует вероятность того, что это событие произойдет или не произойдет. Вероятность, как правило, имеет большое применение в играх, в бизнесе для прогнозирования на основе вероятности, а также вероятность имеет широкое применение в этой новой области искусственного интеллекта.

    Вероятность события можно рассчитать по формуле вероятности, просто разделив число благоприятных исходов на общее число возможных исходов. Значение вероятности того, что событие произойдет, может находиться в диапазоне от 0 до 1, потому что число благоприятных исходов никогда не пересекается с общим числом исходов. Кроме того, благоприятное число исходов не может быть отрицательным. Давайте подробно обсудим основы вероятности в следующих разделах.

    1. Что такое вероятность?
    2. Терминология теории вероятностей
    3. Формула вероятности
    4. Диаграмма дерева вероятностей
    5. Типы вероятностей
    6. Определение вероятности события
    7. Вероятность подбрасывания монеты
    8. Вероятность броска кубиков
    9. Вероятность вытягивания карт
    10. Теоремы о вероятности
    11. Часто задаваемые вопросы о вероятности

    Что такое вероятность?

    Вероятность можно определить как отношение числа благоприятных исходов к общему числу исходов события. Для эксперимента с числом исходов n количество благоприятных исходов можно обозначить как x. Формула для расчета вероятности события выглядит следующим образом.

    Вероятность (событие) = Благоприятные исходы/Всего исходы = x/n

    Давайте проверим простое применение вероятности, чтобы лучше понять ее. Предположим, нам нужно предсказать, будет дождь или нет. Ответ на этот вопрос либо «Да», либо «Нет». Есть вероятность дождя или его отсутствия. Здесь мы можем применить вероятность. Вероятность используется для предсказания результатов подбрасывания монет, бросания костей или извлечения карты из колоды игральных карт.

    Вероятность подразделяется на теоретическую и экспериментальную.

    Терминология теории вероятностей

    Следующие термины вероятности помогают лучше понять концепции вероятности.

    Эксперимент: Испытание или операция, проводимая для получения результата, называется экспериментом.

    Пространство выборки: Все возможные результаты эксперимента вместе составляют пространство выборки. Например, выборочное пространство подбрасывания монеты — это орел и решка.

    Благоприятный исход: Событие, приведшее к желаемому результату или ожидаемому событию, называется благоприятным исходом. Например, когда мы бросаем два кубика, возможные/благоприятные результаты получения суммы чисел на двух кубиках как 4 равны (1,3), (2,2) и (3,1).

    Испытание: Испытание означает проведение случайного эксперимента.

    Случайный эксперимент: Эксперимент с четко определенным набором результатов называется случайным экспериментом. Например, когда мы подбрасываем монету, мы знаем, что выиграем или опередим, но не уверены, какая из них выпадет.

    Событие: Общее количество исходов случайного эксперимента называется событием.

    Равновероятные события: События, которые имеют одинаковые шансы или вероятность наступления, называются равновероятными событиями. Исход одного события не зависит от другого. Например, когда мы подбрасываем монету, есть равные шансы выпадения орла или решки.

    Исчерпывающие события: Когда множество всех результатов эксперимента равно выборочному пространству, мы называем это исчерпывающим событием.

    Взаимоисключающие события: События, которые не могут произойти одновременно, называются взаимоисключающими событиями. Например, климат может быть как жарким, так и холодным. Мы не можем испытывать одну и ту же погоду одновременно.

    Формула вероятности

    Формула вероятности определяет вероятность наступления события. Это отношение благоприятных исходов к общему количеству благоприятных исходов. Формула вероятности может быть выражена как,

    где

    • P(B) — вероятность события «B».
    • n(B) — количество благоприятных исходов события «В».
    • n(S) — общее количество событий, происходящих в пространстве выборки.

    Различные формулы вероятности

    Формула вероятности с правилом сложения: Всякий раз, когда событие является объединением двух других событий, скажем, A и B, тогда
    P(A или B) = P(A) + P(B) — P(A∩B)
    Р(А ∪ В) = Р(А) + Р(В) — Р(А∩В)

    Формула вероятности с дополнительным правилом: Всякий раз, когда событие является дополнением другого события, в частности, если А является событием, тогда P(не A) = 1 — P(A) или P(A’) = 1 — П(А).
    P(A) + P(A′) = 1,

    Формула вероятности с условным правилом : Когда событие A уже известно, что оно произошло, и требуется вероятность события B, тогда P(B, учитывая A) = P(A и B), P(A при заданном B). В случае события B может быть и наоборот.
    P(B∣A) = P(A∩B)/P(A)

    Формула вероятности с правилом умножения : Всякий раз, когда событие является пересечением двух других событий, то есть события A и B должны произойти одновременно. Тогда P(A и B) = P(A)⋅P(B).
    P(A∩B) = P(A)⋅P(B∣A)

    Пример 1 : Найдите вероятность выпадения числа меньше 5 при броске игральной кости, используя формулу вероятности.

    Решение

    Найти:
    Вероятность выпадения числа меньше 5
    Дано: Пример пространства = {1,2,3,4,5,6}
    Получение числа меньше 5 = {1,2,3,4}
    Следовательно, n(S) = 6
    п(А) = 4
    Использование формулы вероятности,
    P(A) = (n(A))/(n(s))
    р(А) = 4/6
    m = 2/3

    Ответ: Вероятность выпадения числа меньше 5 равна 2/3.

    Пример 2: Какова вероятность выпадения 9 при бросании двух игральных костей?

    Решение:

    Всего есть 36 возможностей, когда мы бросаем два кубика.
    Чтобы получить желаемый результат, то есть 9, мы можем иметь следующие благоприятные исходы.
    (4,5),(5,4),(6,3)(3,6). Возможны 4 благоприятных исхода.
    Вероятность события P(E) = (Количество благоприятных исходов) ÷ (Всего исходов в выборке)
    Вероятность выпадения числа 9 = 4 ÷ 36 = 1/9

    Ответ: Следовательно, вероятность выпадения числа 9 равна 1/9.

    Диаграмма дерева вероятностей

    Древовидная диаграмма вероятности — это визуальное представление, которое помогает найти возможные результаты или вероятность того, что какое-либо событие произойдет или не произойдет. Древовидная диаграмма подбрасывания монеты, приведенная ниже, помогает понять возможные результаты при подбрасывании монеты и, таким образом, определить вероятность выпадения орла или решки при подбрасывании монеты.

    Типы вероятности

    Могут существовать различные точки зрения или типы вероятностей, основанные на характере результата или подходе, используемом при определении вероятности события. Четыре типа вероятностей:

    • Классическая вероятность
    • Эмпирическая вероятность
    • Субъективная вероятность
    • Аксиоматическая вероятность

    Классическая вероятность

    Классическая вероятность, часто называемая «априорной» или «теоретической вероятностью», утверждает, что в эксперименте, где есть B равновероятных исходов, а событие X имеет ровно A из этих исходов, тогда вероятность X есть A/B, или P(X) = A/B. Например, когда бросается правильная игральная кость, есть шесть возможных исходов, которые равновероятны. Это означает, что вероятность выпадения каждого числа на кубике составляет 1/6.

    Эмпирическая вероятность

    Эмпирическая вероятность или экспериментальная перспектива оценивает вероятность посредством мысленных экспериментов. Например, если бросается взвешенный кубик, так что мы не знаем, какая сторона имеет вес, то мы можем получить представление о вероятности каждого исхода, бросая кубик определенное количество раз и вычисляя долю раз, когда кубик дает этот результат и, таким образом, найти вероятность этого результата.

    Субъективная вероятность

    Субъективная вероятность рассматривает собственную веру человека в происходящее событие. Например, вероятность того, что конкретная команда выиграет футбольный матч, по мнению болельщика, больше зависит от их собственной веры и чувства, а не от формального математического расчета.

    Аксиоматическая вероятность

    В аксиоматической вероятности ко всем типам применяется набор правил или аксиом Колмогорова. Вероятность наступления или ненаступления любого события может быть количественно определена применением этих аксиом, заданных как 9.0003

    • Наименьшая возможная вероятность равна нулю, а наибольшая — единице.
    • Вероятность достоверного события равна единице.
    • Любые два взаимоисключающих события не могут произойти одновременно, а объединение событий говорит, что может произойти только одно из них.

    Определение вероятности события

    В эксперименте вероятность события — это вероятность того, что это событие произойдет. Вероятность любого события — это значение между (включительно) «0» и «1».

    Вероятностные события

    В теории вероятностей событие — это набор результатов эксперимента или подмножество выборочного пространства.

    Если P(E) представляет вероятность события E, то мы имеем

    • P(E) = 0 тогда и только тогда, когда E — невозможное событие.
    • P(E) = 1 тогда и только тогда, когда E — некоторое событие.
    • 0 ≤ P(E) ≤ 1.

    Предположим, что нам даны два события, «А» и «В», тогда вероятность события А, Р(А) > Р(В), тогда и только тогда, когда событие «А» более вероятно, чем событие «Б». Выборочное пространство (S) представляет собой набор всех возможных результатов эксперимента, а n (S) представляет количество результатов в выборочном пространстве.

    P(E) = n(E)/n(S)

    P(E’) = (n(S) — n(E))/n(S) = 1 — (n(E)/n (S))

    E’ означает, что событие не произойдет.

    Следовательно, теперь мы также можем заключить, что P(E) + P(E’) = 1

    Вероятность подбрасывания монеты

    Теперь рассмотрим вероятность подбрасывания монеты. Довольно часто в таких играх, как крикет, для принятия решения о том, кто будет играть первым, мы иногда используем подбрасывание монеты и принимаем решение на основе результата подбрасывания. Давайте проверим, как мы можем использовать понятие вероятности при подбрасывании одной монеты. Далее мы также рассмотрим подбрасывание двух и трех приходов соответственно.

    Подбрасывание монеты

    Подбрасывание одной монеты имеет два исхода: решка и решка. Понятие вероятности, которое представляет собой отношение благоприятных исходов к общему количеству исходов, можно использовать для определения вероятности выпадения орла и вероятности выпадения решки.

    Общее количество возможных исходов = 2; Образец пространства = {H, T}; H: голова, T: хвост

    • P(H) = количество голов/общее количество результатов = 1/2
    • P(T)= количество решек/общее число исходов = 1/2

    Подбрасывание двух монет

    В процессе подбрасывания двух монет у нас есть четыре исхода. Формулу вероятности можно использовать для определения вероятности выпадения двух орлов, одного орла или отсутствия орла, и аналогичную вероятность можно рассчитать для количества решек. Расчеты вероятности для двух орлов следующие.

    Общее количество исходов = 4; Пространство выборки = {(H, H), (H, T), (T, H), (T, T)}

    • P(2H) = P(0 T) = количество результатов с двумя орлами/всего Исходы = 1/4
    • P(1H) = P(1T) = количество результатов только с одной головкой/общее количество результатов = 2/4 = 1/2
    • P(0H) = (2T) = Количество исходов с двумя орлами/Всего исходов = 1/4

    Подбрасывание трех монет

    Общее количество исходов при одновременном подбрасывании трех монет равно 2 3 = 8. Для этих исходов можно найти вероятность выпадения одного орла, двух орлов, трех орлов и ни одного орла. . Аналогичную вероятность можно рассчитать и для количества решек.

    Общее количество результатов = 2 3 = 8 Пространство выборки = {(H, H, H), (H, H, T), (H, T, H), (T, H, H), ( T, T, H), (T, H, T), (H, T, T), (T, T, T)}

    • P(0H) = P(3T) = Количество исходов без голов /Всего результатов = 1/8
    • P(1H) = P(2T) = количество результатов с одной головкой/общее количество результатов = 3/8
    • P(2H) = P(1T) = Количество исходов с двумя орлами / Всего исходов = 3/8
    • P(3H) = P(0T) = количество исходов с тремя орлами/общее количество исходов = 1/8

    Вероятность броска кубиков

    Во многих играх для определения ходов игроков используются кости. Игра в кости имеет шесть возможных результатов, а результаты игры в кости — это игра на удачу, и их можно получить, используя концепции вероятности. В некоторых играх также используются два кубика, и существует множество вероятностей, которые можно рассчитать для исходов с использованием двух кубиков. Давайте теперь проверим исходы, их вероятности для одного и двух кубиков соответственно.

    Бросание одного игрального кубика

    Общее количество результатов при бросании игральной кости равно 6, а выборочное пространство равно {1, 2, 3, 4, 5, 6}. Здесь мы вычислим следующие несколько вероятностей, чтобы помочь лучше понять концепцию вероятности при броске одного игрального кубика.

    • P(четное число) = количество четных исходов/общее число исходов = 3/6 = 1/2
    • P(Нечетное число) = Количество результатов с нечетным числом/Общее количество результатов = 3/6 = 1/2
    • P(простое число) = количество исходов простых чисел/общее число исходов = 3/6 = 1/2

    Бросание двух игральных костей

    Общее количество результатов при бросании двух игральных костей равно 6 2 = 36. На следующем рисунке показано примерное пространство из 36 исходов при бросании двух игральных костей.

    Проверим несколько вероятностей исходов двух игральных костей. Вероятности следующие.

    • Вероятность получить дублет (одинаковое число) = 6/36 = 1/6
    • Вероятность выпадения числа 3 хотя бы на одной кости = 11/36
    • Вероятность получения суммы 7 = 6/36 = 1/6

    Как мы видим, когда мы бросаем один кубик, есть 6 возможностей. Когда мы бросаем два кубика, у нас есть 36 возможностей. Когда мы бросаем 3 кубика, мы получаем 216 возможностей. Таким образом, общая формула для представления количества результатов при бросании игральных костей: 6 n .

    Вероятность вытягивания карт

    Колода, содержащая 52 карты, сгруппирована в четыре масти: трефы, бубны, червы и пики. Каждая из треф, бубен, червей и пик имеет по 13 карт, что в сумме дает 52. Теперь давайте обсудим вероятность вытягивания карт из колоды. Ниже показаны символы на картах. Пики и трефы — черные карты. Червы и бубны — красные карточки.

    13 карт каждой масти: туз, 2, 3, 4, 5, 6, 7, 8, 9, 10, валет, дама, король. В них валет, дама и король называются фигурными картами. Мы можем понять вероятность карты из следующих примеров.

    • Вероятность вытянуть черную карту равна P(черная карта) = 26/52 = 1/2
    • Вероятность вытянуть червовую карту равна P(Червы) = 13/52 = 1/4
    • Вероятность вытягивания лицевой карты равна P(лицевая карта) = 12/52 = 3/13
    • Вероятность вытянуть карту с номером 4 равна P(4) = 4/52 = 1/13
    • Вероятность вытянуть красную карточку с номером 4 равна P(4 Red) = 2/52 = 1/26

    Теоремы вероятности

    Следующие теоремы вероятности полезны для понимания приложений вероятности, а также для выполнения многочисленных вычислений, связанных с вероятностью.

    Теорема 1: Сумма вероятностей наступления и ненаступления события равна 1. \(P(A) + P(\bar A) = 1\)

    Теорема 2: Вероятность невозможного события или вероятность того, что событие не произойдет, всегда равна 0. \(\begin{align}P(\phi) =0\end{align}\)

    Теорема 3: Вероятность достоверного события всегда равна 1. P(A) = 1

    Теорема 4: Вероятность наступления любого события всегда лежит между 0 и 1. 0 < P( A) < 1

    Теорема 5: Если есть два события A и B, мы можем применить формулу объединения двух множеств и вывести формулу вероятности наступления события A или события B следующее.

    \(P(A\cup B) = P(A) + P(B) — P(A\cap B)\)

    Также для двух взаимоисключающих событий A и B имеем P( A U B) = P(A) + P(B)

    Теорема Байеса об условной вероятности

    Теорема Байеса описывает вероятность события на основе условия возникновения других событий. Ее также называют условной вероятностью. Это помогает в расчете вероятности возникновения одного события на основе условия возникновения другого события.

    Например, предположим, что есть три мешка, в каждом из которых находится несколько синих, зеленых и желтых шаров. Какова вероятность того, что из третьего мешка вынут желтый шар? Поскольку есть также синие и зеленые шары, мы можем получить вероятность на основе этих условий. Такая вероятность называется условной вероятностью.

    Формула теоремы Байеса: \(\begin{align}P(A|B) = \dfrac{ P(B|A)·P(A)} {P(B)}\end{align}\ )

    где \(\begin{align}P(A|B) \end{align}\) обозначает, как часто происходит событие A при условии, что B происходит.

    где \(\begin{align}P(B|A) \end{align}\) обозначает, как часто происходит событие B при условии, что происходит A.

    \(\begin{align}P(A) \end{align}\) вероятность возникновения события A.

    \(\begin{align}P(B) \end{align}\) вероятность наступления события B.

    Закон полной вероятности

    Если в эксперименте имеется n событий, то сумма вероятностей этих n событий всегда равна 1.

    \(P(A_1) + P(A_2) + P (A_3) + ….P(A_n) = 1\)

    Также проверьте:

    • Вероятность и статистика
    • Вероятностные правила
    • Взаимоисключающие события
    • Независимые события
    • Биномиальное распределение
    • Формула Байе
    • Формула распределения Пуассона

    Важные примечания о вероятности:

    Давайте проверим следующие пункты, которые помогут нам обобщить ключевые знания по этой теме вероятности.

    • Вероятность — это мера вероятности того, что событие произойдет.
    • Вероятность представлена ​​в виде дроби и всегда находится в диапазоне от 0 до 1.
    • Событие может быть определено как подмножество выборочного пространства.
    • При бросании монеты выпадает орел или решка, а при бросании игральной кости выпадает 1, 2, 3, 4, 5 или 6.
    • Случайный эксперимент не может предсказать точные результаты, а только некоторые вероятные результаты.

    Cuemath — одна из ведущих в мире платформ для обучения математике, которая предлагает онлайн-уроки по математике в режиме реального времени один на один для классов K-12. Наша миссия — изменить то, как дети изучают математику, чтобы помочь им преуспеть в школе и на конкурсных экзаменах. Наши опытные преподаватели проводят 2 или более живых занятий в неделю в темпе, соответствующем потребностям ребенка в обучении.

     

    Решенные примеры на вероятности

    1. Пример 1: Какова вероятность того, что при бросании двух игральных костей в сумме выпадет 10?

      Решение:

      Есть 36 вариантов, когда мы бросаем две кости.

      Желаемый исход равен 10. Чтобы получить 10, у нас может быть три благоприятных исхода.

      {(4,6),(6,4),(5,5)}

      Вероятность события = количество благоприятных исходов/пространство выборки

      Вероятность выпадения числа 10 = 3/36 = 1/12

      Ответ: Следовательно, вероятность выпадения суммы 10 равна 1/12.

    2. Пример 2: В мешке 6 синих и 8 желтых шаров. Из мешка случайным образом выбирается один шар. Найдите вероятность выпадения синего шара.

      Решение:

      Предположим, что вероятность извлечения синего шара равна P(B)

      Количество благоприятных исходов для получения синего шара = 6

      Общее количество шаров в мешке = 14

      P(B) = Количество благоприятных исходов/Общее количество исходов = 6/14 = 3/7

      Ответ: Следовательно, вероятность вытащить синий шар равна 3 /7.

    3. Пример 3: Есть 5 карточек с номерами: 2, 3, 4, 5, 6. Найдите вероятность того, что выпадет простое число, и, положив его обратно, вы выберете составное число.

      Решение:

      Два события независимы. Таким образом, мы используем произведение вероятности событий.

      P(получение простого числа) = n(благоприятные события)/ n(пространство выборки) = {2, 3, 5}/{2, 3, 4, 5, 6} = 3/5

      p(получение составное) = n(благоприятные события)/ n(пространство выборки) = {4, 6}/{2, 3, 4, 5, 6}= 2/5

      Таким образом, общая вероятность двух независимых событий = P( простое) × P(составное)

      = 3/5 × (2/5)

      = 6/25

      Ответ: Следовательно, вероятность выбора простого числа и еще одного простого числа равна 6/25.

    4. Пример 4. Найдите вероятность получения лицевой карты из стандартной колоды карт по формуле вероятности.

      Решение: Найти:
      Вероятность получения лицевой карты
      Дано: Общее количество карт = 52
      Количество лицевых карт = Благоприятные исходы = 12
      Использование формулы вероятности,
      Вероятность = (Благоприятные исходы)÷(Всего благоприятных исходов)
      P (лицевая карта) = 12/52
      m = 3/13

      Ответ: Вероятность выпадения лицевой карты равна 3/13

    перейти к слайдуперейти к слайдуперейти к слайдуперейти к слайду

    Разбивайте сложные концепции с помощью простых визуальных средств.

    Математика больше не будет сложным предметом, особенно когда вы понимаете концепции с помощью визуализаций.

    Записаться на бесплатный пробный урок

    Практические вопросы по вероятности

     

    перейти к слайдуперейти к слайду

    Часто задаваемые вопросы о вероятности

    Что такое вероятность?

    Вероятность — это раздел математики, который занимается определением вероятности наступления события. Вероятность измеряет вероятность того, что событие произойдет, и равна количеству благоприятных событий, деленному на общее количество событий. Значение вероятности колеблется от 0 до 1, где 0 обозначает неопределенность, а 1 обозначает уверенность.

    Как рассчитать вероятность с помощью формулы вероятности?

    Вероятность любого события зависит от количества благоприятных исходов и общего числа исходов. В общем случае вероятность представляет собой отношение числа благоприятных исходов к общему числу исходов в этом пространстве выборки. Это выражается как Вероятность события P(E) = (Количество благоприятных исходов) ÷ (Выборочное пространство).

    Как определить вероятность?

    Вероятность можно определить, предварительно зная выборочное пространство результатов эксперимента. Вероятность обычно рассчитывается для события (x) в пространстве выборки. Вероятность события получается путем деления количества исходов события на общее количество возможных исходов или выборочное пространство.

    Какие существуют три типа вероятности?

    Существует три типа вероятностей: теоретическая вероятность, экспериментальная вероятность и аксиоматическая вероятность. Теоретическая вероятность вычисляет вероятность на основе формул и входных значений. Экспериментальная вероятность дает реалистичное значение и основана на экспериментальных значениях для расчета. Довольно часто теоретическая и экспериментальная вероятности расходятся в своих результатах. А аксиоматическая вероятность основана на аксиомах, управляющих понятиями вероятности.

    Что такое условная вероятность?

    Условная вероятность предсказывает наступление одного события на основе наступления другого события. Если есть два события А и В, условная вероятность — это вероятность наступления события В при условии, что событие А уже произошло. Формула условной вероятности наступления события B при условии, что событие A произошло: P(B/A) = P(A ∩ B)/P(A).

    Что такое экспериментальная вероятность?

    Экспериментальная вероятность основана на результатах и ​​значениях, полученных в ходе экспериментов с вероятностью. Экспериментальная вероятность определяется как отношение общего количества случаев, когда событие произошло, к общему количеству проведенных испытаний. Результаты экспериментальной вероятности основаны на реальных случаях и могут отличаться по значениям от теоретической вероятности.

    Что такое распределение вероятностей?

    Двумя важными вероятностными распределениями являются биномиальное распределение и распределение Пуассона. Биномиальное распределение определяется для событий с двумя вероятностными исходами и для событий с кратным количеством таких событий. Распределение Пуассона основано на множестве вероятностных исходов в ограниченном промежутке времени, расстоянии, пространстве выборки. Примером биномиального распределения является подбрасывание монеты с двумя исходами и проведение такого эксперимента с подбрасыванием n монет. Распределение Пуассона предназначено для таких событий, как обнаружение антигена в образце плазмы, вероятность которых многочисленна.

    Как связаны вероятность и статистика?

    Вероятность вычисляет возникновение эксперимента и вычисляет возникновение конкретного события по отношению ко всему набору событий. Для простых событий из нескольких чисел легко рассчитать вероятность. Но для расчета вероятностей, связанных с многочисленными событиями, и для управления огромными данными, относящимися к этим событиям, нам нужна помощь статистики. Статистика помогает правильно анализировать

    Как вероятность используется в реальной жизни?

    Вероятность широко применяется в играх и анализе. Также в реальной жизни и в областях промышленности, где речь идет о прогнозировании, мы используем вероятность. Прогнозирование цены акции или выступления команды в крикете требует использования концепций вероятности. Кроме того, новая технологическая область искусственного интеллекта в значительной степени основана на вероятности.

    Как была открыта вероятность?

    Использование слова «вероятность» впервые началось в семнадцатом веке, когда оно относилось к действиям или мнениям, которых придерживались разумные люди. Далее, слово вероятное в юридическом содержании относилось к суждению, имеющему материальные доказательства. Поле перестановок и комбинаций, статистический вывод, криптоанализ, частотный анализ в целом внесли свой вклад в это современное поле вероятностей.

    Где мы используем формулу вероятности в нашей реальной жизни?

    Следующие действия в нашей реальной жизни, как правило, следуют формуле вероятности:

    • Прогноз погоды
    • Игральные карты
    • Стратегия голосования в политике
    • Бросание игральной кости.
    • Вытягивание точно совпадающих носков одного цвета
    • Шансы на победу или поражение в любом виде спорта.

    Что такое формула условной вероятности?

    Условная вероятность зависит от наступления одного события на основе наступления другого события. Формула условной вероятности наступления события B при условии, что событие A уже произошло, выражается как P(B/A) = P(A ∩ B)/P(A).

    Как рассчитать вероятность (с примерами)

    • Что такое вероятность?
    • Как рассчитать вероятность
    • Шансы против. Вероятность
    • Как рассчитать вероятность с несколькими случайными событиями
    • Вакансии, связанные со статистикой и вероятностями
    • Часто задаваемые вопросы о вероятностях
    • Заключительные мысли
    • Ссылки
    • Зарегистрируйтесь для получения дополнительных советов и вакансий

    Показать больше

    Резюме. Чтобы вычислить вероятность, разделите количество вариантов, в которых событие может произойти, на общее количество исходов. Помните, что это отличается от расчета шансов, которые представляют собой вероятность того, что что-то произойдет, деленное на вероятность того, что это не произойдет.

    Вы когда-нибудь проверяли погоду утром, слышали, что в этот день с вероятностью 80% будет дождь, и меняли одежду или планы? Если да, то вы приняли решение, основанное на вероятности.

    Мы познакомим вас с основами вероятностных концепций и способами их расчета. Таким образом, когда вам нужно предсказать результат или принять решение, основанное на вероятности, вы готовы быть на высоте.

    Основные выводы:

    • Вероятность — вероятность того, что что-то произойдет.

    • Вероятность рассчитывается путем деления количества возможных вариантов возникновения события на общее количество исходов.

    • Вероятность и шансы — разные понятия. Шансы — это вероятность того, что что-то произойдет, деленная на вероятность того, что этого не произойдет.

    • Почти все профессии связаны с использованием вероятности в тот или иной момент. Однако есть определенные профессии, которые в значительной степени зависят от расчета вероятности, например, статистики, метеорологи и медицинские работники.

    Что такое вероятность?

    Вероятность — это вероятность того, что что-то произойдет. Когда вы вычисляете вероятность, вы аппроксимируете вероятность того, что что-то произойдет, и представляете это точным числом. Поэтому, когда погода сообщает о 80-процентной вероятности дождя, это означает, что в этот день будет 80-процентная вероятность дождя. Другими словами, если вы попадете в одни и те же погодные условия десять раз, то восемь из этих раз будет идти дождь.

    Невозможно предсказать вещи с абсолютной точностью, например погоду, но вероятность позволяет приблизиться к ней как можно ближе. Независимо от того, рассчитываете ли вы что-то простое, например, подбрасывание монеты, или что-то сложное, вы можете использовать вероятность, чтобы понять результат, например, прогноз продаж.

    Вероятно, вы сможете принимать более взвешенные решения и подкреплять свои прогнозы данными, чтобы приводить убедительные аргументы. Вы можете найти способы использовать вероятность в каждой сфере своей жизни, от простых повседневных догадок до сложных прогнозов продаж и маркетинговых планов.

    Как рассчитать вероятность

    Самое замечательное в вероятности то, что она использует простую формулу, независимо от того, вероятность чего вы хотите измерить. Если вы можете выполнить простое умножение и деление, вы сможете вычислить вероятность для любой ситуации в кратчайшие сроки. Вот основная формула вероятности:

    Вероятность того, что что-то произойдет = количество способов, которыми может произойти событие ÷ общее количество исходов

    Давайте разберем, как найти нужные числа и рассчитать вероятность события. Мы будем использовать простой пример извлечения шариков из мешка, чтобы упростить понимание процесса, но вероятность может стать намного сложнее, если вам нужно обрабатывать большие события.

    1. Найдите свое мероприятие. Во-первых, вам нужно выяснить, какая переменная помогает вам определить вероятность. Например, если у вас есть мешок с цветными шариками и вы хотите вычислить вероятность того, что выпадет синий, вам нужно вычислить эту переменную. Результат, который вы хотите рассчитать, — это вероятность того, что вы вытащите синий шарик из смешанного мешка.

    2. Найдите все результаты. Далее вам нужно найти общее количество исходов, которые вы можете получить в этой ситуации. Итак, если в вашем мешке с шариками всего 20, у вас есть 20 возможных исходов. Этот шаг определяет все ваши возможности в этой ситуации, а не конкретный результат, который вы хотите рассчитать.

    3. Найдите желаемый результат. Вам нужно выяснить, сколько шансов получить желаемый результат. Для этого примера подсчитайте, сколько из этих 20 шариков синих, чтобы вы могли вычислить свои шансы выбрать синий шарик. Для этого примера предположим, что вы насчитали 11 синих шариков в мешке с 20 шариками.

    4. Сделай свой расчет. Теперь, когда у вас есть все нужные числа, вы можете перейти к следующему шагу и использовать формулу, чтобы найти вероятность. Разделите 11 на 20, и вы должны получить 0,55 или 55%. Что означает это число? Это означает, что вероятность того, что вы вытащите синий шарик из мешка, составляет 55%.

    Коэффициенты против. Вероятность

    Когда вы говорите о вероятности того, что что-то произойдет, легко спутать шансы и вероятность. Люди определяют шансы как вероятность того, что что-то произойдет, деленное на вероятность того, что этого не произойдет.

    Знание шансов события — отличный способ проверить желаемый результат. Если вы обращаете внимание только на вероятность того, что событие произойдет, вы можете упустить вероятность того, что оно не произойдет.

    Если вероятность того, что что-то произойдет, составляет 70 %, вы можете подумать, что это число достаточно близко к 100 %, чтобы на него можно было положиться. Но вы должны понимать, что это также означает 30%-ную вероятность того, что событие не произойдет. Как вы можете видеть, расчет шансов влияет на вероятность того, что что-то произойдет, а что-то не произойдет.

    Пример расчета шансов

    Например, еще раз взгляните на мраморный мешок. В мешке по-прежнему 20 шариков, но на этот раз вы хотите найти шансы выбрать зеленый шарик. Есть два зеленых шарика, поэтому теперь вы хотите разделить два на 20 и получить 0,1.

    Что означает этот номер? Это означает, что существует вероятность 0,9, что вы не выберете зеленый шарик. Чтобы найти шансы, вам нужно разделить 0,1 на 0,9, чтобы получить шансы 0,1111 или 11,11%.

    Как рассчитать вероятность с несколькими случайными событиями

    К сожалению, не все может быть так просто, как один человек достает шарики из мешка. Иногда вам нужно рассчитать вероятность события, когда действуют несколько факторов. К счастью, вы можете рассчитать вероятность того, что что-то произойдет, когда задействовано несколько событий. Это легко сделать, если вы знаете, как получить вероятность одного события.

    Мы снова будем использовать что-то похожее на пример с мрамором. Допустим, ваша компания присуждает приз, если кто-то выберет из мешка шарик, соответствующий цветам компании. Цвета компании — красный и белый, и вы хотите вычислить вероятность того, что один человек выберет красный из мешка А, а другой игрок в то же время выберет белый из мешка Б.

    1. Расчет вероятности каждого события

      Вы знаете, что в мешке с шариками находится 500 шариков: 100 красных, 250 белых, 50 синих и 100 зеленых. Таким образом, вы можете рассчитать вероятность того, что кто-то возьмет красный шарик из мешка А, взяв 100 красных шариков и разделив их на 500 шариков, чтобы получить 0,2. Для мешка B вы берете 250 белых шариков и делите их на общее количество 500 шариков и получаете 0,5.

    2. Расчет вероятности двух событий одновременно

      Теперь, когда вы знаете вероятность того, что эти два события произойдут, вы можете рассчитать вероятность того, что они произойдут одновременно, перемножив отдельные вероятности. Таким образом, вы умножаете 0,2 на 0,5, чтобы получить 0,1. Это означает, что существует 10%-ная вероятность того, что кто-то вытащит красный шарик из мешка А, а кто-то вытащит белый шарик из мешка Б.

    Сделав еще один шаг вперед, вы также можете рассчитать вероятность того, что это произойдет. Чтобы найти шансы этой ситуации, вы можете разделить 0,1 на 0,9.чтобы получить 11,11% шансов, что это произойдет.

    Некоторые люди лучше понимают, вычисляют и интерпретируют вероятности, чем другие. Если вам нравится вычислять вероятность того, что что-то произойдет, или вам нравится использовать данные для принятия решений, возможно, вы захотите найти работу, связанную со статистикой и вероятностью. Вот несколько профессий, которые в значительной степени полагаются на вероятность, предсказания и прогнозирование.

    Просто имейте в виду, что почти каждая работа потребует от вас использования вероятности и анализа, поэтому, даже если вы не будете заниматься одной из этих профессий, вы все равно будете использовать статистику и вероятность в своей работе. Просто это может быть реже, чем в тех областях, где вероятность управляет работой, которую вы делаете каждый день.

    • Статистик

    • Метролог

    • Математик

    • Аналитик по исследованию рынка

    • Финансовый аналитик

    • Аналитик по исследованию операций

    • Ученый-актуарий

    • Биостатистика

    • Медицинские профессии

    • Оценка риска

    • Здравоохранение

    • Эпидемиолог

    • Профессор

    • Учитель математики

    Часто задаваемые вопросы о вероятности

    1. Как рассчитать вероятность нескольких событий?

      Чтобы вычислить вероятность нескольких событий, вы должны сначала вычислить вероятность каждого независимого события. Затем вы перемножаете вероятности каждого независимого события друг с другом.

    2. В чем разница между вероятностью и шансами?

      Разница между вероятностью и шансами заключается в том, что вероятность учитывает только вероятность того, что что-то произойдет, а шансы также учитывают вероятность того, что это НЕ произойдет.

      Чтобы рассчитать шансы, вы берете вероятность того, что что-то произойдет, и делите ее на вероятность того, что это не произойдет.

    3. Нужно ли мне знать о вероятности на моей работе?

      Да, вы должны знать основы вероятности для вашей работы. Хотя некоторые профессии требуют интенсивного использования вероятности, например, статистика, каждая профессия в тот или иной момент использует вероятность.

      Это связано с тем, что на рабочем месте так много динамичных ситуаций, что профессионалам необходимо учитывать множественные результаты в любом сценарии.

    4. Какова формула вероятности?

      Формула вероятности представляет собой количество способов, которыми может произойти желаемое событие, деленное на общее количество исходов.

      Например, если вы пытаетесь рассчитать вероятность вытащить синий шарик из мешка с 20 шариками, а 4 из этих 20 шариков синие, вы должны разделить 4 (количество синих шариков, то есть желаемое количество шариков). исход) на 20 (общее количество исходов).

      Это дает вам вероятность 0,2 или 20%.

    Заключительные мысли

    Независимо от того, в какой области или отрасли вы работаете, в какой-то момент вам придется использовать вероятность. Мы используем вероятность в нашей повседневной жизни, даже если мы этого не знаем. Знание того, как рассчитать вероятность чего-либо, и понимание того, что это означает, является ключевым навыком для любого человека.

    Вероятность может помочь вам во всех аспектах профессионального принятия решений, в зависимости от того, какой маркетинговый план использовать или какой подход позволит вам увеличить продажи. Возможности использования вероятности безграничны, если вы понимаете основы.

    Ссылки

    1. Школа общественного здравоохранения Бостонского университета – Основные понятия вероятности

    2. Академия Кана – Вероятность: основы

    Насколько полезным был этот пост?

    Нажмите на звездочку, чтобы оценить!

    Средний рейтинг / 5.

    Сравнить дроби онлайн калькулятор с решением: Сравнения дробей онлайн.

    2
    Функция — Квадрат x
    ctg(x)
    Функция — Котангенс от x
    arcctg(x)
    Функция — Арккотангенс от x
    arcctgh(x)
    Функция — Гиперболический арккотангенс от x
    tg(x)
    Функция — Тангенс от x
    tgh(x)
    Функция — Тангенс гиперболический от x
    cbrt(x)
    Функция — кубический корень из x
    gamma(x)
    Гамма-функция
    LambertW(x)
    Функция Ламберта
    x! или factorial(x)
    Факториал от x
    DiracDelta(x)
    Дельта-функция Дирака
    Heaviside(x)
    Функция Хевисайда

    Интегральные функции:

    Si(x)
    Интегральный синус от x
    Ci(x)
    Интегральный косинус от x
    Shi(x)
    Интегральный гиперболический синус от x
    Chi(x)
    Интегральный гиперболический косинус от x

    В выражениях можно применять следующие операции:

    Действительные числа
    вводить в виде 7. 3
    — возведение в степень
    x + 7
    — сложение
    x — 6
    — вычитание
    15/7
    — дробь

    Другие функции:

    asec(x)
    Функция — арксеканс от x
    acsc(x)
    Функция — арккосеканс от x
    sec(x)
    Функция — секанс от x
    csc(x)
    Функция — косеканс от x
    floor(x)
    Функция — округление x в меньшую сторону (пример floor(4.5)==4.0)
    ceiling(x)
    Функция — округление x в большую сторону (пример ceiling(4.5)==5.0)
    sign(x)
    Функция — Знак x
    erf(x)
    Функция ошибок (или интеграл вероятности)
    laplace(x)
    Функция Лапласа
    asech(x)
    Функция — гиперболический арксеканс от x
    csch(x)
    Функция — гиперболический косеканс от x
    sech(x)
    Функция — гиперболический секанс от x
    acsch(x)
    Функция — гиперболический арккосеканс от x

    Постоянные:

    pi
    Число «Пи», которое примерно равно ~3. 14159..
    e
    Число e — основание натурального логарифма, примерно равно ~2,7183..
    i
    Комплексная единица
    oo
    Символ бесконечности — знак для бесконечности

    Калькулятор сравнения дробей — beGalileo

    Сравнивать дроби вручную может быть сложно, особенно если речь идет о сложной паре дробей. Этот калькулятор может помочь вам с сравнением легко. Он может сказать вам, какая дробь больше, а какая меньше среди заданного набора дробей.

    Как сравнить дроби в калькуляторе?

    Шаг 1: Введите первую дробь
    Шаг 2: Введите вторую дробь
    Шаг 3: Нажмите кнопку Сравнить .
    При нажатии на кнопку сравнения будет показан расчет доли сравнения.

    Примеры для проверки калькулятора сравнения дробей

    Пример 1:
    Сравните 15/5 и 50/5
    Решение:
    15/5 (15/5 меньше 50/5)

    Пример 2:
    Сравните 25/15 и 75/55
    Решение:
    25/15 (25/15 меньше, чем 75/55)

    Дроби — это фундаментальное понятие в математике, и они используются во многих различных приложениях. Сравнение дробей — важный навык, особенно в повседневных ситуациях, таких как покупка, приготовление пищи и обмен. Понимание того, как сравнивать дроби, может помочь вам принимать обоснованные решения, а также может оказаться полезным навыком при работе с более сложными математическими понятиями. В этой статье мы обсудим, что такое дроби, как сравнивать дроби, и предоставим подробное руководство о том, как использовать онлайн-калькулятор сравнения дробей.

    Что такое дроби?

    Дробь — это математическое представление части целого. Он записывается в виде числителя и знаменателя с горизонтальной чертой между ними. Например, дробь 3/4 представляет собой три части из четырех равных частей. Числитель — это верхнее число, которое представляет количество частей, а знаменатель — это нижнее число, которое представляет общее количество частей.

    Сравнение дробей

    Сравнение дробей включает определение того, какая дробь больше или меньше другой дроби. При сравнении дробей возможны три исхода:

    1. Первая дробь больше второй.
    2. Первая дробь равна второй дроби.
    3. Первая дробь меньше второй дроби.

    Для сравнения двух дробей можно использовать несколько методов, в том числе:

    1. Нахождение общего знаменателя: Когда дроби имеют разные знаменатели, их может быть трудно сравнить. Один из способов упростить сравнение — найти общий знаменатель. Это включает в себя нахождение числа, кратного обоим знаменателям. Получив общий знаменатель, вы можете сравнить числители, чтобы определить, какая дробь больше.

    2. Перекрестное умножение. Другой метод сравнения дробей — перекрестное умножение. Это означает умножение числителя одной дроби на знаменатель другой дроби. Дробь с большим результатом является большей дробью.

    3. Преобразование в десятичные числа. Третий метод сравнения дробей заключается в преобразовании их в десятичные числа. Это предполагает деление числителя на знаменатель. Дробь с большим десятичным значением является большей дробью.

    Преимущества использования онлайн-калькулятора для сравнения дробей

    Использование онлайн-калькулятора для сравнения дробей имеет много преимуществ, в том числе:

    1. Скорость: Онлайн-калькулятор для сравнения дробей работает намного быстрее, чем сравнение дробей вручную.

    2. Точность: онлайн-калькулятор сравнения дробей менее подвержен ошибкам, чем сравнение дробей вручную.

    3. Простота использования: онлайн-калькулятор сравнения дробей прост в использовании даже для людей, не знакомых с математическими понятиями.

    Заключение

    Сравнение дробей — важный навык, особенно в повседневных ситуациях, таких как покупка, приготовление пищи и обмен. С помощью онлайн сравнения дробей.

    Используйте другие математические калькуляторы, такие как калькулятор процентного отношения к десятичному, калькулятор арктангенса, калькулятор четных или нечетных чисел, калькулятор вычитания дробей и калькулятор упрощенных дробей

    Онлайн-курсы по математике > Математические калькуляторы > Калькулятор сравнения дробей

    Калькулятор сравнения дробей | Какая дробь больше?

    К

    Энес Сульич

    Любит упрощать и объяснять сложные концепции. Искренне верит, что каждый может решить кажущиеся трудными проблемы, если приложит достаточно усилий. В свободное время любит читать или играть в шахматы.

    Последнее обновление: 1 марта 2023 г.

    5/5 — (1 голос)

    Наш калькулятор сравнения дробей даст вам ответ и покажет работу по сравнению дробей. Это может быть полезно, если вы хотите узнать, как сравнивать дроби, или вам просто нужна небольшая помощь с домашним заданием по математике.

    Что такое дробь?

    Когда вы смотрите на дробь, легко заметить, что верхнее число называется числителем, а нижнее — знаменателем. Особый тип дроби, в которой оба числа равны, называется равной или единичной дробью. Например, 1/2 и 2/4 равные дроби.

    Когда вы записываете дробь в словесной форме, вы можете сказать «четыре пятых» вместо «4/5» или «одна треть» вместо «1/3».

    Как сравнивать дроби

    Сравнивать дроби — значит наблюдать за двумя дробями и выяснять, какая из них больше другой. Для этого нужно знать некоторые правила.

    Сравнение дробей с разными знаменателями

    Поясню на примере.

    \frac {1}{3} \text { и } \frac {1}{2}

    Чтобы узнать, какой из них больше, нам нужно иметь одинаковые знаменатели для обоих из них. Для этого нам нужно найти их общий знаменатель и расширить их. Обычно лучший способ найти общий знаменатель — это умножить два знаменателя друг на друга. В этом случае результат будет 6,

    Затем нам нужно расширить первый. Мы делаем это, разделив общий знаменатель на его знаменатель. Это даст нам 2. Затем нам также нужно умножить числитель на 2. Как только мы это сделаем, мы расширим фракцию. Затем нам нужно сделать то же самое со второй дробью.

    Теперь, когда у них одинаковые знаменатели, больше тот, у которого больше числитель.

    \фракция {2}{6} < \фракция {3}{6}

    Сравнение дробей с одинаковыми числителями

    Для этой цели числители не важны. Процесс такой же, как и в предыдущем примере.

    Сравнение дробей с одинаковыми знаменателями

    Если у них одинаковые знаменатели, то больше та, у которой больше числитель.

    Сравнение неправильных и смешанных дробей

    Неправильными дробями называются дроби, у которых числитель больше знаменателя. Эти дроби можно превратить в смешанные дроби, представляющие собой смесь целых чисел и дробей, например:

    \ гидроразрыв {5} {4} = 1 \ гидроразрыв {1} {4}

    По сути, процесс тот же. Если у вас есть смешанная дробь, вы превращаете ее в неправильную дробь и повторяете процесс, начиная с предыдущего. Вы превращаете смешанную дробь в неправильную, умножая число, стоящее впереди, на знаменатель, а затем прибавляя результат к числителю.

    Как пользоваться калькулятором сравнения дробей

    Конечно, проще всего сравнивать дроби с помощью нашего калькулятора. Все, что вам нужно сделать, это ввести обе дроби в калькулятор, и калькулятор скажет вам, какая из них больше, или равны ли они.

    Dy y: Mathway | Популярные задачи

    2

    заказ решений на аукционе за минимальную цену с максимальным качеством

    Предлагаю идею сайта-аукциона по выполнению домашних заданий. Он будет включать:

    • решение задач по математике (сейчас доступен решебник Филиппова), физике, химии, экономике
    • написание лабораторных, рефератов и курсовых
    • выполнение заданий по литературе, русскому или иностранному языку.

    Основное отличие от большинства сайтов, предлагающих выполнение работ на заказ – сайт рассчитан на две категории пользователей: заказчиков и решающих задания. Причем, по желанию (чтобы заработать, увеличить свой рейтинг, получить решение сложной задачи) пользователи могут играть любую из этих ролей.

    Объединение сервисов в одну систему

    Основой для идеи послужили несколько работающих систем, объединение которых позволит сделать сервис для решения задач на заказ. Эти системы:

    • Форум, где посетители обмениваются идеями и помогают друг другу
    • Система bugtracking, где обнаруженные проблемы проходят путь от публикации до принятия в исполнение и решения
    • Аукцион, где цена за товар или услугу определяется в результате торгов
    • Система рейтингов, где участники могут оценивать ответы друг друга. Причем, чем больше рейтинг пользователя, тем более значимым становится его голос

    Принцип работы

    Для удобства и проведения аналогий с реальной жизнью назовем заказчиков студентами, а решающих задания – репетиторами.

    Итак, студенту необходимо решить несколько задач. Он заходит на сайт, выбирает раздел с соответствующей дисциплиной и создает новую тему (аналогия с форумом). Но при создании темы он также указывает стартовую (максимальную) цену, которую он готов заплатить за решение задач и крайний срок исполнения задания. Можно будет назначить и нулевую цену – если студенту нужно только бесплатное решение.

    Как только тема создана, все пожелавшие подписаться на раздел репетиторы получают уведомление. Причем, условие получения уведомлений можно настроить. Например, уведомлять только о заказах со стартовой ценой более 500 р. и сроком решения не менее недели.

    Заинтересовавшиеся репетиторы делают ставки. Причем студент (автор темы) видит ставки и может посмотреть информацию по каждому репетитору (его решения, рейтинг, дату начала участия в проекте). Когда студент посчитает нужным, он может остановить аукцион и назначить задание одному из репетиторов, сделавшему ставку (не обязательно самую низкую, т.к. можно учитывать и другие факторы – см. выше).

    Деньги блокируются на счете студента, и репетитор начинает решать задание. Он должен представить его к сроку, заданному изначально. Выполненное решение публикуется в свободном доступе и его может оценить как заказчик, так и другие репетиторы. На этих оценках и строится рейтинг. Если к решению нет претензий – деньги окончательно переводятся со счета студента на счет репетитора.

    За счет чего будет развиваться сервис

    Первое – положительная обратная связь. Чем больше условий задач и решений будет опубликовано на сайте, тем чаще его будут находить пользователи через поисковики, будет больше ссылок на готовые решения. Именно поэтому важно размещать решенные задачи в свободном доступе. Знаю это по опыту своего сайта exir.ru (ex irodov.nm.ru) – большая ссылочная база получена исключительно за счет благодарных пользователей.

    Второе – удобный сервис для заказчиков и для желающих заработать на решениях.

    Преимущества для заказчиков

    Студентам и школьникам не нужно перебирать десятки сайтов для сравнения цен, а потом надеяться, что после оплаты они получат качественное решение (и, вообще, все не закончится перечислением денег). Заказчики создают аукцион на понижение цены и могут смотреть на рейтинги желающих решить задачи и ранее выполненные ими решения. Кроме того, деньги окончательно перечисляются исполнителю только после полного решения.

    Преимущества для решающих задания

    Не нужно создавать и продвигать свой сайт, размещать множество объявлений во всех доступных источниках информации. Заказчики сами придут к вам. Не нужно решать все присланные задания с целью поддержания репутации – можно выбирать те, которые будут интересны по уровню сложности, цене и срокам решения.

    Преимущества для владельца сервиса

    Если вы не понимаете, какую выгоду получит делающий вам какое-нибудь предложение – будьте осторожны! 🙂 У меня уже есть большой опыт работы с сайтом, предоставляющим бесплатные решения по физике. И вариант с получением прибыли от размещения рекламы подходит и для нового сервиса. Кроме того, мне нравится помогать людям и довольно тяжело смотреть, как множество вопросов по задачам остаются на форуме без ответа. Предложенный аукцион решений сможет значительно сократить число вопросов без ответов.

    В будущем возможен вариант и с получением некоторого небольшого процента от оплаты заказов. Но процент этот должен быть минимален и на начальном этапе он взиматься точно не будет.

    Что необходимо для создания сервиса

    1. Самым важное сейчас – собрать команду, готовую принять участие в выполнении заданий. Если покупатели заходят в пустой магазин – они надолго забывают в него дорогу.

      Поэтому я собираю предварительные заявки от посетителей, готовых заниматься решениями. Не нужно подписания никаких договоров о намерениях. Просто сообщите, на какие темы вы готовы решать задания, какой у вас опыт подобной работы (e-mail: [email protected]). Когда сервис заработает – я пришлю приглашение на регистрацию.

    2. Выбрать платежную систему.
    3. Сделать подходящий движок для сайта. Нужно решить – создавать его с нуля или изменить какой-нибудь существующий движок (например, форумный) с открытой лицензией.
    4. Привлечь посетителей. Учитывая посещаемость exir.ru и число публикуемых на форуме вопросов, думаю, это не будет большой проблемой.

    Desertai be cukraus Vilniuje: tortai, pyragaičiai, saldainiai

    Купить акции DYY | Цена акций DYY сегодня и новости

    1. Акции
    2. DB Товарная двойная длинная позиция ETN

    DYY

    См. цену акций DYY и покупку/продажу DB Товарной двойной длинной позиции ETN. Обсуждайте новости и ценовые прогнозы аналитиков с сообществом инвесторов.

    Начать инвестировать

    Инвестировать в DYY

    Тип заказа

    Инвестировать в

    Акции Доллары

    Сумма

    Рыночная цена$3,50

    Оценочное количество0. 00

    Зарегистрируйтесь, чтобы купить

    DB Товарный двойной длинный ETN (DYY)

    Нам не удалось загрузить этот график.

    Некоторые биржевые диаграммы в настоящее время могут не поддерживаться. Чтобы повторить попытку, обновите эту страницу.

    1D1W1M3M6M1Y5Y

    О DB Commodity Double Long ETN (DYY)

    Полное название компании: Deutsche Bank AG London — DB Commodity Double Long ETN.

    2 соотношение цена/2 прибыль 051 —x 0,0052
    Рыночная капитализация
    Вчерашний объем $900
    Выручка (TTM)
    Ср. дневной объем $52,42
    EBITDA (TTM)
    Открытие $3,50
    Вчерашний диапазон $3,5 — $3,5
    Долг/капитал
    52-недельный диапазон 3,5–6,95 долл. США
    Бета-версия (LTM) 0,29x
    Дивиденды и доходность ()
    Следующий заработок

    Часто задаваемые вопросы (FAQ)

    52-недельный максимум для акций DYY составляет $6,95. Текущая цена акций DYY $3,5001 на 49,64% ниже ее 52-недельного максимума

    . 52-недельный минимум акций DYY составляет $3,50. Текущая цена акций DYY $3,5001 выросла на 0,00% по сравнению с 52-недельным минимумом 9.0011

    Нет, акции DYY не приносят дивиденды акционерам. внесение депозита с помощью дебетовой карты или банковского перевода.

    Выберите сумму, которую вы хотели бы инвестировать в акции DYY

    Перейдите на страницу «Обзор». Затем введите DYY в строку поиска. Когда в результатах появятся акции DYY, коснитесь их, чтобы открыть экран покупки.

    Управляйте своими инвестициями в одном месте

    Вы можете найти недавно купленные акции DYY в своем портфеле — наряду с остальными вашими акциями, ETF, криптовалютой и альтернативными активами.

    См. цену акций DYY и покупку/продажу товара DB Double Long ETN. Обсуждайте новости и ценовые прогнозы аналитиков с сообществом инвесторов.

    Начать инвестировать

    Инвестировать в DYY

    Тип заказа

    Инвестировать в

    Акции Доллары

    Сумма

    Рыночная цена$3,50

    Оценочное количество0,00

    Зарегистрируйтесь, чтобы купить основой для принятия инвестиционного решения, а не рекомендацией или советом.

    DB Товарный двойной длинный ETN

    • Эмитенты
    • Xshares Advisors LLC
    • ДИГ

    ‘Нет данных’

    Категория: Товары с кредитным плечом

    Последнее обновление:

    Реклама

    • ДГГ Профиль акций и цена
    • Дивиденды и оценка
    • Соотношение расходов и сборы
    • Холдинги
    • Графики анализа холдингов
    • Графики цен и объемов
    • Графики движения средств
    • Диаграммы влияния цены и потоков на AUM
    • ESG
    • Производительность
    • Технические характеристики
    • Рейтинг в реальном времени
    • НОВЫЙ! Отчет консультанта и информационный бюллетень
    • Читать дальше
    • Подробнее на Тренды ETF
    • Происхождение данных и раскрытие информации

    DYY Профиль акций и цена Дивиденды и оценка Соотношение расходов и сборы Холдинги Графики анализа холдингов Графики цен и объемов Графики движения средств Диаграммы влияния цены и потоков на AUM ESG Производительность Технические характеристики Рейтинг в реальном времени НОВЫЙ! Отчет консультанта и информационный бюллетень Читать дальше Подробнее на Тренды ETF Происхождение данных и раскрытие информации

    Этот ETF больше не активен. См. активные ETF в в Товары с кредитным плечом Категория базы данных ETF.

    Основные показатели

    Эмитент Xshares Advisors LLC

    Бренд Н/Д

    Структура ETN

    Коэффициент расходов 0,75%

    Домашняя страница ETF Домашняя страница

    Начало 01 мая 2008 г.

    Индекс отслеживается Индекс ликвидных товаров Deutsche Bank (200%)

    Аналитический отчет

    Отчет FA PDF

    Аналитический отчет для DYY недоступен.

    Темы базы данных ETF

    Категория Товары с кредитным плечом

    2x

    Класс активов Товар

    Тип товара Разнообразный

    Товар Широкая

    Товарная экспозиция На основе фьючерсов

    Классификации FactSet

    Сегмент Н/Д
    Категория Н/Д
    Фокус Н/Д
    Ниша Н/Д
    Стратегия Н/Д
    Схема взвешивания Н/Д

    Торговые данные

    • Открыть
    • Объем
    • День Ло
    • День привет
    • 52 неделя Ло 1,72 доллара США
    • 52 неделя привет 3,45 доллара США
    • АУМ 1,3 млн долларов
    • Акции 0,6 М

    Исторические торговые данные

    • 1 месяц в среднем Объем 14,900
    • 3 месяца в среднем Объем 7378

    Альтернативные ETF в категории товаров с кредитным плечом в базе данных ETF

    Тип Символ Коэффициент расходов Активы Ср. Ежедневный объем Возврат с начала года
    Самый дешевый ДГП 0,75% 89,3 млн долларов 7075 15,98%
    Самый большой (АУМ) КИПЯТИТЬ 1,33% 1,2 млрд долларов 69 м -80,54%
    Наиболее ликвидный (объем) КИПЯТИТЬ 1,33% 1,2 млрд долларов 69 м -80,54%
    Лучший исполнитель с начала года УГЛ 0,95% 211,2 млн долларов Daily Vol»> 126 384 16,06%

    15 крупнейших холдингов

    Новый

      Анализ концентрации

      В этом разделе сравнивается сбалансированность и глубина этого ETF по сравнению с аналогами.

      Сравнение активов

      ДИГ Средняя категория базы данных ETF Среднее значение сегмента FactSet
      Количество владений 6 1 Н/Д
      % активов в топ-10 99,99% 100,00% Н/Д
      % активов в топ-15 99,99% 100,00% Н/Д
      % активов в топ-50 99,99% 100,00% Н/Д

      Сравнение размеров

      ДИГ Средняя категория базы данных ETF Среднее значение сегмента FactSet
      Большой (> 12,9 млрд) Н/Д Н/Д Н/Д
      Середина (> 2,7 млрд) Н/Д Н/Д Н/Д
      Малый (> 600M) Н/Д Н/Д Н/Д
      Микро (<600M) Н/Д Н/Д Н/Д

      DYY Оценка

      В этом разделе сравнивается соотношение цена/прибыль этого ETF с аналогами.

      DYY

      P/E Ratio

      N/A

      Среднее значение по категории базы данных ETF

      P/E Ratio

      0,01

      FactSet Segment Average

      010 Н/Д

      ДГГ Дивиденд

      В этом разделе сравнивается дивидендная доходность этого ETF с аналогами.

      ДИГ Средняя категория базы данных ETF Среднее значение сегмента FactSet
      Дивиденд Н/Д Н/Д Н/Д
      Дата выплаты дивидендов Н/Д Н/Д Н/Д
      Годовая ставка дивидендов Н/Д Н/Д Н/Д
      Годовой дивидендный доход Н/Д Н/Д Н/Д

      Графики цен и объемов DYY

      Данные диаграммы для DYY недоступны.

      Диаграммы потоков фонда DYY

      Новый

      Просмотрите графики с данными о потоках средств ETF.

      Чистый поток за 5 дней: 0 Чистые потоки за 1 месяц: 0 Чистые потоки за 3 месяца: 0 Чистые потоки за 6 месяцев: 0 Чистые потоки за 1 год: 0 Чистый поток за 3 года: 0 Чистый поток за 5 лет: -1,41 М Чистые потоки за 10 лет: -4,8 М

      Графики влияния DYY Price vs Flows AUM

      Просмотрите диаграммы, которые показывают влияние денежных потоков и цен на активы в целом.

      Изменение чистого AUM за 5 дней: 0
      Изменение чистых активов под управлением за 1 месяц: 0
      Изменение чистого AUM за 3 месяца: 0
      Изменение чистого AUM за 6 месяцев: 0
      Изменение чистых активов под управлением за 1 год: 0
      Изменение чистых активов под управлением за 3 года: 0
      Изменение чистых активов под управлением за 5 лет: 0
      Изменение чистых активов под управлением за 10 лет: 0

      Рейтинг в реальном времени

      В соседней таблице инвесторы получают индивидуальный рейтинг DYY в реальном времени по нескольким различным показателям, включая ликвидность, расходы, производительность, волатильность, дивиденды, концентрацию активов в дополнение к общему рейтингу. Поле «ETF с метрическим рейтингом A+», доступное для членов ETF Database Pro, показывает ETF в товарах с кредитным плечом с самым высоким метрическим рейтингом в реальном времени для каждого отдельного поля. Чтобы просмотреть все эти данные, подпишитесь на бесплатную 14-дневную пробную версию базы данных ETF Database Pro. Чтобы просмотреть информацию о том, как работают рейтинги базы данных ETF в реальном времени, нажмите здесь.

      Общая оценка

      Сравнивать | Отчет по категории

      Метрическая система Метрика Рейтинг в реальном времени ETF с метрическим рейтингом A+
      Ликвидность С+ УКО
      Затраты А+ ОЙЛУ
      Производительность
      Волатильность
      Дивиденд
      Концентрация

      ETF с общим рейтингом A+:

      Сравнивать | Отчет по категории

      DYY Расходы и сборы

      В этом разделе сравнивается экономическая эффективность этого ETF с аналогами.

      Анализ соотношения расходов

      DYY

      Коэффициент расходов

      0,75%

      База данных ETF Категория Среднее

      Коэффициент расходов

      1,15%

      Среднее значение сегмента FactSet

      Коэффициент расходов

      Н/Д


      Налоговый анализ

      Максимальная ставка прироста капитала ST 35,00%
      Максимальная долгосрочная ставка прироста капитала 15,00%
      Налог на выплаты Н/Д
      Распределяет K1 Н/Д

      Темы и оценки ESG

      Новый

      DYY не имеет оценки ESG. Нет ESG-темы которые сопоставляются с этим ETF.

      Производительность ДГГ

      В этом разделе показано, как этот ETF работает по сравнению с его аналогами. Доходность за 1 год рассчитывается в годовом исчислении.

      ДГГ База данных ETF
      Средняя категория
      Сегмент набора фактов
      Среднее
      1 месяц возврата Н/Д 15,73% Н/Д
      Возврат через 3 месяца Н/Д -10,25% Н/Д
      Возврат с начала года 5,96% -9,20% Н/Д
      1 год возврата -29,54% -24,87% Н/Д
      3 года возврата 0,71% 1,37% Н/Д
      5-летний возврат -23,20% -10,21% Н/Д

      Географическое положение

      Следующие диаграммы отражают географическое распространение DYY-х базовые активы.

      Анализ холдингов

      Следующие диаграммы отражают распределение DYY-х базовые активы.

      Технические характеристики DYY

      Анализ волатильности

      В этом разделе показано, как волатильность этого ETF сравнивается с категорией базы данных ETF аналогичной группы.

      5-дневная волатильность 74,13%
      20-дневная волатильность 33,29%
      50-дневная волатильность 35,98%
      200-дневная волатильность 53,41%
      Бета 2,26
      Стандартное отклонение 0,34%

      &nbsp

      Посмотреть подробный анализ

      &nbsp

      Рейтинг отсутствует

      Рейтинг
      Н/Д

      Низкий
      ВТИУ (47,50%)

      Высокий
      УКО (652,72%)


      Рейтинг отсутствует

      Рейтинг
      Н/Д

      Низкий
      ДГП (29,92%)

      Высокий
      КИПЯЧЕНИЕ (119,19%)


      Рейтинг отсутствует

      Рейтинг
      Н/Д

      Низкий
      ДГП (29,54%)

      Высокий
      КИПЯЧЕНИЕ (145,82%)


      Рейтинг отсутствует

      Ранг
      Н/Д

      Низкий
      ДГП (29,63%)

      Высокий
      КИПЯЧЕНИЕ (152,48%)


      Рейтинг отсутствует

      Рейтинг
      Н/Д

      Низкий
      УГЛ (0,28)

      Высокий
      ОЙЛУ (2. 7)


      Рейтинг отсутствует

      Рейтинг
      Н/Д

      Низкий
      ДГП (3,70%)

      Высокий
      КИПЯЧЕНИЕ (37,90%)


      &nbsp

      View Summary Analysis

      &nbsp

      Технические характеристики

      • 20-дневная МА 2,18 доллара США
      • 60-дневная МА 2,20 доллара США
      • MACD 15 Период -0,04
      • MACD 100 Период -0,08
      • Williams % Range 10 Day 43,65
      • Williams % Range 20 Day 44.21
      • Индекс относительной силы 10 дней 43
      • RSI 20 дней 45
      • RSI 30 дней 46
      • Конечный осциллятор 47

      Полосы Боллинджера

      • Нижний Боллинджер (10 дней) 2,07 доллара США
      • Верхний Боллинджер (10 дней) 2,25 доллара США
      • Нижний Боллинджер (20 дней) 2,10 доллара США
      • Верхний Боллинджер (20 дней) 2,27 доллара США
      • Нижний Боллинджер (30 дней) 2,12 доллара США
      • Верхний Боллинджер (30 дней) 2,29 доллара США

      Поддержка и сопротивление

      • Уровень поддержки 1 2,05 доллара США
      • Уровень поддержки 2 1,97 доллара США
      • Уровень сопротивления 1 2,18 доллара США
      • Уровень сопротивления 2 2,22 доллара США

      Стохастический

      • Стохастический осциллятор %D (1 день) 62. 02
      • Стохастический осциллятор %D (5 дней) 63,79
      • Стохастический осциллятор %K (1 день) 52.00
      • Стохастический осциллятор %K (5 дней) 59,61

      Отчет финансового консультанта

      Отчеты финансовых консультантов базы данных ETF разработаны как удобный раздаточный материал для клиентов, чтобы объяснить ключевую информацию о фонде. Включает в себя новые идеи аналитиков и данные классификации.

      Загрузить отчет FA в формате PDF

      Информационный бюллетень

      Информационные бюллетени выпускаются поставщиком ETF и оформляются базой данных ETF. Своевременность и точность информации, содержащейся в информационном бюллетене, не гарантируется.

      Загрузить информационный бюллетень в формате PDF

      Читать дальше

      Последние новости DYY

      PRO

      Новости

      Sneha Shah21 июля 2018 г.

      2018-07-21

      Вот 25 лучших и 25 худших ETF за прошедшую неделю. Трейдеры могут использовать этот список, чтобы…

      PRO

      Новости

      Sneha Shah14 июля 2018 г.

      2018-07-14

      Вот 25 лучших и 25 худших ETF за прошедшую неделю. Трейдеры могут использовать этот список, чтобы…

      PRO

      Новости

      Sneha Shah17 марта 2018 г.

      2018-03-17

      Вот 25 лучших и 25 худших ETF за прошедшую неделю. Трейдеры могут использовать этот список, чтобы…

      Другие ETF в категории базы данных ETF

      • КИПЯТИТЬ ProShares Ultra Bloomberg Природный газ
      • УКО Сырая нефть ProShares Ultra Bloomberg
      • AGQ ProShares Ультра Серебро
      • УГЛ ProShares Ультра Золото
      • ОЙЛУ MicroSectors Oil & Gas Exp. & Прод. 3x ETN с кредитным плечом
      • ДГП DB Gold Двойные длинные биржевые ноты
      • ВТИУ MicroSectors Energy 3X ETN с кредитным плечом

      Посмотреть другие биржевые фонды сырьевых товаров с кредитным плечом

      Популярные темы

      Дивиденды
      • БРКА Неофициальная история Уоррена Баффета
      • ААПЛ Что можно купить за наличные Apple?
      • МСФТ 10 до смешного неверных бычьих и медвежьих сигналов

      Подробнее

      Data Lineage

      Тенденции ETF и база данных ETF, выдающиеся цифровые платформы для новостей ETF, исследований, инструментов, видео, веб-трансляций, каналов с собственным контентом и многого другого. Торговые марки ETF Trends и ETF Database пользуются доверием среди консультантов, институциональных инвесторов и индивидуальных инвесторов в течение 25 лет. Фирмы имеют уникальную возможность помочь консультантам в обучении, внедрении и использовании ETF, а также в переходе сообщества управляющих активами от традиционно аналогового к цифровому взаимодействию с сообществом консультантов.

      Наша команда в базе данных ETF стремится сделать наш веб-сайт главным источником информации об инвестировании в ETF с помощью инструментов, контента и ресурсов ETF самого высокого качества в мире.

      Есть вопросы? Связаться с нами.

      Приведено в действие:

      Авторы аналитических отчетов

      Аналитики базы данных ETF имеют общий 50-летний опыт работы на ETF и финансовых рынках, охватывая каждый класс активов и стиль инвестирования. Команда отслеживает новые заявки, новые запуски и новых эмитентов, чтобы убедиться, что мы помещаем каждый новый ETF в соответствующий контекст, чтобы финансовые консультанты могли создавать высококачественные портфели.

      Раскрытие информации

      Copyright © 2023 FactSet Research Systems Inc. Все права защищены.

      Copyright MSCI ESG Research LLC [2018]. Все права защищены. Продукты Fund Metrics компании MSCI ESG Research LLC («MSCI ESG») («Информация») предоставляют экологические, социальные и управленческие данные в отношении базовых ценных бумаг в более чем 23 000 взаимных фондов и ETF с несколькими классами активов по всему миру. MSCI ESG является зарегистрированным инвестиционным консультантом в соответствии с Законом об инвестиционных консультантах от 19 года.40. Материалы MSCI ESG не были представлены и не получили одобрения Комиссии по ценным бумагам и биржам США или какого-либо другого регулирующего органа. Никакая информация не является предложением о покупке или продаже, продвижением или рекомендацией какой-либо ценной бумаги, финансового инструмента, продукта или торговой стратегии, а также не должна рассматриваться как указание или гарантия какой-либо будущей деятельности, анализа, прогноза или прогноза. . Никакая информация не может быть использована для определения того, какие ценные бумаги покупать или продавать, а также когда их покупать или продавать. Информация предоставляется «как есть», и пользователь Информации принимает на себя весь риск любого использования или разрешения на использование Информации. Вся Информация предоставляется исключительно для вашего внутреннего использования и не может воспроизводиться или распространяться в любой форме без предварительного письменного разрешения MSCI. Ни MSCI ESG, ни какое-либо из ее аффилированных лиц, ни любое третье лицо, участвующее или связанное с созданием какой-либо Информации, не дает никаких явных или подразумеваемых гарантий, заявлений или гарантий, и ни при каких обстоятельствах MSCI ESG или любое такое аффилированное лицо или третье лицо не будут нести никакой ответственности за любые прямой, косвенный, специальный, штрафной, косвенный или любой другой ущерб (включая упущенную выгоду), относящийся к любой Информации. Дополнительную информацию о метриках фонда MSCI ESG, предоставленную MSCI ESG Research LLC, можно найти по адресу https://www.

      Что такое матрица в математике определение: для чего нужны, применение, кто придумал, история возникновения

      Матрицы: определение и основные понятия.

      Навигация по странице:

      • Определение матрицы
      • Обозначение матрицы
      • Элементы матрицы
      • Диагонали матрицы

      Онлайн калькуляторы с матрицами.

      Упражнения с матрицами.

      Определение матрицы

      Определение.

      Матрицей размера n×m называется прямоугольная таблица специального вида, состоящая из n строк и m столбцов, заполненная числами.

      Количество строк и столбцов задают размеры матрицы.


      Обозначение

      Матрица — это таблица данных, которая берется в круглые скобки:

      A =  4  1  -7 
       -1  0  2 

      Матрица обычно обозначаются заглавными буквами латинского алфавитв. Матрица содержащая n строк и m столбцов, называется матрицей размера n×m. При необходимости размер матрицы записывается следующим образом: An×m.


      Элементы матрицы

      Элементы матрицы A обозначаются aij, где i — номер строки, в которой находится элемент, j — номер столбца.

      Пример.

      Элементы матрицы A4×4:

      A =  4  1  -7  2 
       -1  0  2  44 
       4  6  7  9 
       11  3  1  5 

      a11 = 4

      Определение.

      Строка матрицы называется нулевой, если все ее элементы равны нулю.

      Определение.

      Если хотя бы один из элементов строки матрицы не равен нулю, то строка называется ненулевой.

      не не нулевой столбец


      Диагонали матрицы

      Определение.

      Главной диагональю матрицы называется диагональ, проведённая из левого верхнего угла матрицы в правый нижний угол.

      Определение.

      Побочной диагональю матрицы называется диагональ, проведённая из левого нижнего угла матрицы в правый верхний угол.

      Пример.

      Демонстрация главной и побочной диагонали матрицы:

       0  1  -7  — главнаяпобочная диагональ
       0  0  2 

       0  1  -7  — главнаяпобочная диагональ
       0  0  2 
       8  2  9 

      Определение.

      Следом матрицы называется сумма диагональных элементов матрицы.

      Обозначение.

      След матрицы обозначается trA = a11 + a22 + … + ann.

      Онлайн калькуляторы с матрицами.

      Упражнения с матрицами.

      Основы высшей математики — Матрицы — Высшая математика — Теория, тесты, формулы и задачи

      Оглавление:

      • Основные теоретические сведения
        • Матрицы
        • Обратная матрица
      • Матрицы. Вся теория и задачи с решениями или ответами

       

      Основные теоретические сведения

      Матрицы

      К оглавлению…

      Матрицей называют прямоугольную таблицу, заполненную числами. Важнейшие характеристики матрицы – число строк и число столбцов. Если у матрицы одинаковое число строк и столбцов, ее называют квадратной. Обозначают матрицы большими латинскими буквами.

      Сами числа называют элементами матрицы и характеризуют их положением в матрице, задавая номер строки и номер столбца и записывая их в виде двойного индекса, причем вначале записывают номер строки, а затем столбца. Например, a14 есть элемент матрицы, стоящий в первой строке и четвертом столбце, a32 стоит в третьей строке и втором столбце.

      Главной диагональю квадратной матрицы называют элементы, имеющие одинаковые индексы, то есть те элементы, у которых номер строки совпадает с номером столбца. Побочная диагональ идет «перпендикулярно» главной диагонали.

      Особую важность представляют собой так называемые единичные матрицы. Это квадратные матрицы, у которых на главной диагонали стоят 1, а все остальные числа равны 0. Обозначают единичные матрицы E. Матрицы называют равными, если у них равны число строк, число столбцов, и все элементы, имеющие одинаковые индексы, равны. Матрица называется нулевой, если все ее элементы равны 0. Обозначается нулевая матрица О.

      Простейшие действия с матрицами

      1. Умножение матрицы на число. Для этого необходимо умножить каждый элемент матрицы на данное число.

      2. Сложение матриц. Складывать можно только матрицы одинакового размера, то есть имеющие одинаковое число строк и одинаковое число столбцов. При сложении матриц соответствующие их элементы складываются.

      3. Транспонирование матрицы. При транспонировании у матрицы строки становятся столбцами и наоборот. Полученная матрица называется транспонированной и обозначается AT. Для транспонирования матриц справедливы следующие свойства:

      4. Умножение матриц. Для произведения матриц существуют следующие свойства:

      • Умножать можно матрицы, если число столбцов первой матрицы равно числу строк второй матрицы.
      • В результате получится матрица, число строк которой равно числу строк первой матрицы, а число столбцов равно числу столбцов второй матрицы.
      • Умножение матриц некоммутативно. Это значит, что от перестановки местами матриц в произведении результат меняется. Более того, если можно посчитать произведение A∙B, это совсем не означает, что можно посчитать произведение B∙A.
      • Пусть C = A∙B. Для определения элемента матрицы С, стоящего в i-той строке и k-том столбце необходимо взять i-тую строку первой умножаемой матрицы и k-тый столбец второй. Далее поочередно брать элементы этих строки и столбца и умножать их. Берем первый элемент из строки первой матрицы и умножаем на первый элемент столбца второй матрицы. Далее берем второй элемент строки первой матрицы и умножаем на второй элемент столбца второй матрицы и так далее. А потом все эти произведения надо сложить.

      Свойства произведения матриц:

      Определитель матрицы

      Определителем (детерминантом) квадратной матрицы А называется число, которое обозначается detA, реже |A| или просто Δ, и вычисляется определённым образом. Для матрицы размера 1х1 определителем является сам единственный элемент матрицы. Для матрицы размера 2х2 определитель находят по следующей формуле:

      Миноры и алгебраические дополнения

      Рассмотрим матрицу А. Выберем в ней s строк и s столбцов. Составим квадратную матрицу из элементов, стоящих на пересечении полученных строк и столбцов. Минором матрицы А порядка s называют определитель полученной матрицы.

      Рассмотрим квадратную матрицу А. Выберем в ней s строк и s столбцов. Дополнительным минором к минору порядка s называют определитель, составленный из элементов, оставшихся после вычеркивания данных строк и столбцов.

      Алгебраическим дополнением к элементу aik квадратной матрицы А называют дополнительный минор к этому элементу, умноженный на (–1)i+k, где i+k есть сумма номеров строки и столбца элемента aik. Обозначают алгебраическое дополнение Aik.

      Вычисление определителя матрицы через алгебраические дополнения

      Рассмотрим квадратную матрицу А. Для вычисления ее определителя необходимо выбрать любую ее строку или столбец и найти произведения каждого элемента этой строки или столбца на алгебраическое дополнение к нему. А дальше надо просуммировать все эти произведения.

      Когда будете считать алгебраические дополнения, не забывайте про множитель (–1)i+k. Чтобы счет был более простым, выбирайте ту строку или столбец матрицы, который содержит наибольшее число нулей.

      Расчет алгебраического дополнения может сводиться к расчету определителя размером более чем 2х2. В этом случае такой расчет также нужно проводить через алгебраические дополнения, и так далее до тех пор, пока алгебраические дополнения, которые нужно будет считать, не станут размером 2х2, после чего воспользоваться формулой выше.

       

      Обратная матрица

      К оглавлению. ..

      Рассмотрим квадратную матрицу А. Матрица A–1 называется обратной к матрице А, если их произведения равны единичной матрице. Обратная матрица существует только для квадратных матриц. Обратная матрица существует, только если матрица А невырождена, то есть ее определитель не равен нулю. В противном случае обратную матрицу посчитать невозможно. Для построения обратной матрицы необходимо:

      1. Найти определитель матрицы.
      2. Найти алгебраическое дополнение для каждого элемента матрицы.
      3. Построить матрицу из алгебраических дополнений и обязательно транспонировать ее. Часто про транспонирование забывают.
      4. Разделить полученную матрицу на определитель исходной матрицы.

      Таким образом, в случае, если матрица А имеет размер 3х3, обратная к ней матрица имеет вид:

       

      Матрицы. Вся теория и задачи с решениями или ответами

      К оглавлению…

      Матрица | Определение, типы и факты

      Ключевые люди:
      Артур Кэли Нильс Фабиан Хельге фон Кох
      Похожие темы:
      обратимая матрица определитель квадратная матрица нулевая матрица элемент

      Просмотреть весь связанный контент →

      матрица , набор чисел, расположенных в строках и столбцах так, чтобы сформировать прямоугольный массив. Числа называются элементами или элементами матрицы. Матрицы имеют широкое применение в технике, физике, экономике и статистике, а также в различных разделах математики. Матрицы также имеют важные приложения в компьютерной графике, где они использовались для представления поворотов и других преобразований изображений.

      Исторически сложилось так, что первой была распознана не матрица, а определенное число, связанное с квадратным массивом чисел, называемое определителем. Лишь постепенно возникло представление о матрице как об алгебраической сущности. Термин матрица был введен английским математиком XIX века Джеймсом Сильвестром, но именно его друг, математик Артур Кэли, разработал алгебраический аспект матриц в двух статьях в 1850-х годах. Кейли впервые применил их к изучению систем линейных уравнений, где они до сих пор очень полезны. Они важны еще и потому, что, как признал Кейли, определенные наборы матриц образуют алгебраические системы, в которых справедливы многие обычные законы арифметики (например, ассоциативный и распределительный законы), но в которых другие законы (например, коммутативный закон) справедливы. недействительно.

      Викторина «Британника»

      Числа и математика

      Если имеется м строк и n столбцов, матрица называется « м на n » и записывается как « м × n ». Например,

      — это матрица 2 × 3. Матрица с n строк и n столбцов называется квадратной матрицей порядка n . Обычное число можно рассматривать как матрицу 1 × 1; таким образом, 3 можно рассматривать как матрицу [3]. Матрица с одной строкой и n столбцов называется вектором-строкой, а матрица только с одним столбцом и n строками называется вектором-столбцом.

      В общепринятых обозначениях заглавная буква обозначает матрицу, а соответствующая строчная буква с двойным нижним индексом описывает элемент матрицы. Таким образом, a ij является элементом i -й строки и j -го столбца матрицы A . Если A — это матрица 2 × 3, показанная выше, то A 11 = 1, A 12 = 3, A 13 = 8, A 21 = 2, A 9 22 = 40060 = 2, A 9 22 = 40060 = A 2 22 = 40060 = 2, A 22 = 40060. 23 = 5. При определенных условиях матрицы можно складывать и умножать как отдельные сущности, что приводит к возникновению важных математических систем, известных как матричные алгебры.

      Матрицы естественным образом встречаются в системах одновременных уравнений. В следующей системе для неизвестных x и y массив чисел представляет собой матрицу, элементами которой являются коэффициенты при неизвестных. Решение уравнений полностью зависит от этих чисел и от их конкретного расположения. Если бы 3 и 4 поменять местами, решение было бы другим.

      Оформите подписку Britannica Premium и получите доступ к эксклюзивному контенту. Подпишитесь сейчас

      Две матрицы A и B равны друг другу, если они имеют одинаковое количество строк и одинаковое количество столбцов и если a ij = b ij для каждого i и каждого j . If A and B are two m × n matrices, their sum S = A + B is the m × n matrix whose elements s ij = а ij + b ij . То есть каждый элемент S равно сумме элементов в соответствующих позициях A и B .

      Матрица A может быть умножена на обычное число c , которое называется скаляром. Произведение обозначается cA или Ac и представляет собой матрицу, элементами которой являются ca ij .

      Умножение матрицы A на матрицу B для получения матрицы C определяется только тогда, когда количество столбцов первой матрицы A равно количеству строк второй матрицы B . Для определения элемента c ij , который находится в i -й строке и j -м столбце произведения, первый элемент в i -й строке A умножается на первый элемент в j -м столбце B , второй элемент в строке на второй элемент в столбце и так далее, пока последний элемент в строке не будет умножен на последний элемент столбца; сумма всех этих произведений дает элемент с ij . В символах, для случая, когда A имеет m столбцов, а B имеет m строк, матрица C имеет столько строк, сколько A , и столько же столбцов, сколько B .

      В отличие от умножения обычных чисел на и на , в котором ab всегда равно ba , умножение матриц A и B не является коммутативным. Однако он является ассоциативным и дистрибутивным по сравнению с сложением. То есть, когда операции возможны, всегда выполняются следующие уравнения: A ( до н.э. ) = ( AB ) C , A ( B + C ) = AB + AC и (122 + 2121212) AC и (122 + 212121212) AC и ( B + 212121212) AC и (122 + 212121212) . А = ВА + СА . Если матрицу 2 × 2 A , строками которой являются (2, 3) и (4, 5), умножить саму на себя, то произведение, обычно записываемое как A 2 , имеет строки (16, 21) и ( 28, 37).

      Матрица O , все элементы которой равны 0, называется нулевой или нулевой матрицей. Квадратная матрица Число с единицами на главной диагонали (от верхнего левого угла к нижнему правому) и нулями в остальных местах называется единичной или единичной матрицей. Его обозначают I или I n , чтобы показать, что его порядок равен n . Если B — любая квадратная матрица, а I и O — единичная и нулевая матрицы одного порядка, всегда верно, что B + O = O + B = B и БИ = БИ = Б . Следовательно, O и I ведут себя как 0 и 1 в обычной арифметике. (На самом деле обычная арифметика — это частный случай матричной арифметики, в которой все матрицы имеют размер 1 × 1.)

      Квадратная матрица A , в которой элементы a ij отличны от нуля только тогда, когда i = j называется диагональной матрицей. Диагональные матрицы обладают тем особым свойством, что их умножение коммутативно; то есть для двух диагональных матриц А и В , АВ = ВА . След квадратной матрицы представляет собой сумму элементов на главной диагонали.

      С каждой квадратной матрицей A связано число, известное как определитель A , обозначаемый det A . Например, для матрицы 2 × 2 det A = ad bc . Квадратная матрица B называется невырожденной, если det B ≠ 0. Если B невырожденна, существует матрица, обратная B , обозначаемый B -1 , такой, что BB -1 = B -1 B = I 90. Уравнение AX = B , в котором A и B — известные матрицы, а X — неизвестная матрица, решается однозначно, если A — невырожденная матрица, тогда A 2 −1 существует, и обе части уравнения можно умножить на него слева: А -1 ( АХ ) = А -1 В . Теперь A −1 ( AX ) = ( A −1 A ) X = IX = X 9; следовательно, решение X = A −1 B . Система m линейных уравнений с n неизвестными всегда может быть выражена в виде матричного уравнения AX = B , в котором A m × n матрица коэффициентов неизвестных, X n × 1 матрица неизвестных, B n × 1 матрица, содержащая числа на правая часть уравнения.

      Проблема большого значения во многих областях науки заключается в следующем: по квадратной матрице A порядка n, найти n × 1 матрицу X, , называемую n -мерный вектор, такой что AX = cX . Здесь c — число, называемое собственным значением, а X — собственный вектор. Существование собственного вектора X с собственным значением c означает, что некоторое преобразование пространства, связанное с матрицей A , растягивает пространство в направлении вектора X на коэффициент c .

      Эта статья была недавно отредактирована и обновлена ​​Эриком Грегерсеном.

      Фрактал | математика | Britannica

      Ключевые люди:
      Бенуа Мандельброт Вацлав Серпинский Гастон Морис Джулия
      Похожие темы:
      Прокладка Серпинского фрактальная размерность фрактальная кривая Юлия сет множество Мандельброта

      Просмотреть весь связанный контент →

      фрактал , в математике любая из класса сложных геометрических фигур, которые обычно имеют «дробную размерность» — понятие, впервые введенное математиком Феликсом Хаусдорфом в 1918. Фракталы отличаются от простых фигур классической или евклидовой геометрии — квадрата, круга, сферы и т. д. Они способны описывать многие объекты неправильной формы или пространственно неоднородные явления в природе, такие как береговые линии и горные хребты. Термин фрактал , происходящий от латинского слова fractus («фрагментированный» или «сломанный»), был введен математиком польского происхождения Бенуа Б. Мандельбротом. См. анимацию фрактального множества Мандельброта.

      Хотя ключевые понятия, связанные с фракталами, изучались математиками в течение многих лет, и многие примеры, такие как кривая Коха или «снежинка», были давно известны, Мандельброт был первым, кто указал, что фракталы могут быть идеальным инструментом в прикладных исследованиях. математика для моделирования самых разных явлений от физических объектов до поведения фондового рынка. С момента своего появления в 1975 году концепция фрактала породила новую систему геометрии, которая оказала значительное влияние на такие разнообразные области, как физическая химия, физиология и гидромеханика.

      Викторина «Британника»

      Числа и математика

      Многие фракталы обладают свойством самоподобия, хотя бы приблизительно, если не точно. Самоподобный объект — это объект, составные части которого похожи на целое. Это повторение деталей или паттернов происходит в постепенно уменьшающихся масштабах и может, в случае чисто абстрактных объектов, продолжаться бесконечно, так что каждая часть каждой части при увеличении будет выглядеть в основном как фиксированная часть целого объекта. Фактически самоподобный объект остается инвариантным при изменении масштаба, т. е. обладает масштабной симметрией. Это фрактальное явление часто можно обнаружить в таких объектах, как снежинки и кора деревьев. Все естественные фракталы такого рода, а также некоторые математические самоподобные фракталы являются стохастическими, или случайными; таким образом, они масштабируются в статистическом смысле.

      Еще одной ключевой характеристикой фрактала является математический параметр, называемый фрактальной размерностью. В отличие от евклидовой размерности, фрактальная размерность обычно выражается нецелым числом, то есть дробью, а не целым числом. Фрактальную размерность можно проиллюстрировать на конкретном примере: кривой снежинки, определенной Хельге фон Кохом в 1904 году. Это чисто математическая фигура с шестикратной симметрией, как и природная снежинка. Он самоподобн в том, что состоит из трех одинаковых частей, каждая из которых, в свою очередь, состоит из четырех частей, являющихся точными уменьшенными версиями целого. Из этого следует, что каждая из четырех частей сама состоит из четырех частей, которые являются уменьшенными версиями целого. Не было бы ничего удивительного, если бы коэффициент масштабирования также был равен четырем, поскольку это было бы верно для сегмента прямой или дуги окружности. Однако для кривой «снежинка» коэффициент масштабирования на каждом этапе равен трем. Фрактальная размерность, D , обозначает степень, в которую нужно возвести 3, чтобы произвести 4, т. е. 3 D = 4. Таким образом, размер кривой снежинки равен D  = log 4/log 3, или примерно 1,26. Фрактальная размерность является ключевым свойством и показателем сложности данной фигуры.

      Фрактальная геометрия с ее концепциями самоподобия и нецелочисленной размерности находит все более широкое применение в статистической механике, особенно при работе с физическими системами, состоящими из, казалось бы, случайных элементов. Например, фрактальное моделирование использовалось для построения графика распределения скоплений галактик по Вселенной и для изучения проблем, связанных с турбулентностью жидкости.

      Матрицы 3 на 3: Онлайн калькулятор. Определитель матрицы. Детерминант матрицы

      Определитель матрицы второго и третьего порядков и правила вычисления

      Матрица содержит в себе векторы-столбцы. Они по-разному ориентированы в пространстве. Характеристикой этого расположения и того матричного преобразования, которое может дать матрица, выступает определитель матрицы.

      Содержание

      Определитель матрицы 2×2

      Пусть дана квадратная матрица второго порядка:

      A=\begin{pmatrix} a_{11}& a_{12}\\ a_{21}& a_{22} \end{pmatrix}

      Определителем (или детерминантом) второго порядка, соответствующим данной матрице, называется число a_{11}a_{22}-a_{12}a_{21}.

      Определитель второго порядка записывается так:

      detA=\begin{vmatrix} a_{11}& a_{12}\\ a_{21}& a_{22} \end{vmatrix}=a_{11}a_{22}-a_{12}a_{21}

      Геометрический смысл определителя

      Если нам дана квадратная матрица

      A=\begin{pmatrix} a_{11}& a_{12}\\ a_{21}& a_{22} \end{pmatrix}

      То первый столбец дает нам координаты одного вектора, а второй столбец чисел — координаты второго вектора. Начало данных векторов — в точке начала координат.

      Тогда определитель матрица дает нам площадь параллелограмма, построенного на данных векторах.

      Рассмотрим на примере

      Пусть нам дана матрица с координатами:

      A=\begin{pmatrix} 3& 1\\ 1& 2 \end{pmatrix}

      Нарисуем координатную плоскость и отметим на ней данные векторы.

      Векторы и

      Где у вектора \overline{a} координаты (3; 1), а у вектора \overline{b} координаты (2; 1).

      Теперь построим на этих векторах параллелограмм, считая, что векторы a и b его стороны. Получим:

      Параллелограмм на векторах и

      Площадь данного параллелограмма и будет являться определителем матрицы. Площадь данного параллелограмма S_{ABCD}=5. И определитель матрицы:

      detA=\begin{vmatrix} 3& 1\\ 1& 2 \end{vmatrix}=3 \cdot 2-1 \cdot 1=6-1=5

      Однако, обычно в линейной алгебре говорят не о площади параллелограмма, а о матричном преобразовании. То есть о том, в какую фигуру матрица преобразует единичный квадрат, построенный на единичных векторах. Насколько она ее масштабирует в пространстве. То есть вот из такого квадрата (синий цвет) матричное преобразование делает параллелограмм с определенной площадью (отмечено красным цветом), равной по модулю определителю матрицы.

      Матричное преобразование площади определитель матрицы

      Однако иногда определитель матрицы может быть отрицательным числом. В этом случае площадь фигуры, построенной на векторах матрицы, будет равна модулю данного числа, а знак минус означает, что ориентация данной фигуры отрицательна.

      Геометрический смысл определителя матрицы

      Определитель показывает какой будет площадь единичного квадрата при матричном преобразовании. Она будет равна площади параллелограмма, который будет построен на векторах-столбцах матрицы. Первый столбец матрицы дает нам координаты первого вектора, а второй столбец  — координаты второго вектора.

      Можно расширить геометрический смысл матрицы и на матрицы другого размера.

      Таким образом, определитель матрицы 1×1 дает длину вектора, 2×2 — площадь параллелограмма, 3×3 — объем параллелепипеда, а nxn — объем n-мерного параллелепипеда. 2=0

      Ответ: 0

      Определитель третьего порядка

      Пусть дана квадратная матрица третьего порядка:

      A=\begin{pmatrix} a_{11}& a_{12} & a_{13}\\ a_{21}& a_{22} & a_{23} \\ a_{31}& a_{32} & a_{33} \end{pmatrix}

      Определителем (или детерминантом) третьего порядка, соответствующим данной матрице, называется число:

      a_{11}a_{22}a_{33}+a_{21}a_{32}a_{13}+a_{12}a_{23}a_{31}-a_{13}a_{22}a_{31}-a_{12}a_{33}a_{21}-a_{11}a_{23}a_{32}

      Определитель третьего порядка будет:

      detA=\begin{vmatrix} a_{11}& a_{12} & a_{13}\\ a_{21}& a_{22} & a_{23} \\ a_{31}& a_{32} & a_{33}\end{vmatrix}= \\ =a_{11}a_{22}a_{33}+a_{21}a_{32}a_{13}+a_{12}a_{23}a_{31}-a_{13}a_{22}a_{31}-a_{12}a_{33}a_{21}-a_{11}a_{23}a_{32}

      Правило треугольников

      При вычислении определителей третьего порядка удобно пользоваться правилом треугольников (правилом Саррюса). Это правило проиллюстрируем на схеме:

      Правило треугольников или правило Саррюса

      Как пользоваться правилом треугольника:

      На схеме есть две картинки — красная и синяя, красная картинка дает нам три положительных слагаемых в формуле определителя третьего порядка, а синяя — три отрицательных.

      Умножаем так — сначала умножаем элементы матрицы по главной диагонали потом в вершинах одного треугольника и в вершинах другого треугольника: . Все полученные множители складываем.

      Теперь обратимся к синей картинке. Тут мы начинаем сначала перемножать элементы по побочной диагонали: , а потом элементы в вершинах двух треугольников: и . Полученные множители записываем в формулу со знаком минус.

      Примеры на вычисление определителя третьего порядка

      a) Вычислить определитель матрицы:

      A=\begin{pmatrix} 2& 3 & 4\\ 6& 5 & 7 \\ 9& 0& 8 \end{pmatrix}

      Решение:

      det A=\begin{vmatrix} 2& 3 & 4\\ 6& 5 & 7 \\ 9& 0 & 8 \end{vmatrix}=2 \cdot 5 \cdot 8+ 3 \cdot 7 \cdot 9+6\cdot 0 \cdot 4 — 4 \cdot 5 \cdot 9 — 3 \cdot 6 \cdot 8 — 7 \cdot 0 \cdot 2=80+189+0-180-144-0=-55

      Ответ: det A=-55

      б) Вычислить определитель матрицы 3×3:

      A=\begin{pmatrix} 2& 3 & 1\\ 1& -5 & 6\\ 2& -3& 4 \end{pmatrix}

      Решение:

      Используем формулу определителя третьего порядка

      det B=\begin{vmatrix} 2& 3 & 1\\ 1& -5 & 6\\ 2& -3& 4 \end{vmatrix}=2 \cdot (-5) \cdot 4+ 3 \cdot 6 \cdot 2+1\cdot 1 \cdot (-3) — 1 \cdot (-5) \cdot 2 — 2 \cdot 6 \cdot (-3) — 3 \cdot 1 \cdot 4=-40+36-3+10+36-12=27

      Ответ: det B=27

      в) Вычислите определитель единичной матрицы 3×3.

      Решение:

      Единичная матрица 3×3 имеет вид:

      A=\begin{pmatrix} 1& 1 & 1\\ 1& 1 & 1\\ 1& 1& 1 \end{pmatrix}

      Используем формулу определителя третьего порядка

      det A=\begin{vmatrix} 1& 1 & 1\\ 1& 1 & 1\\ 1& 1& 1 \end{vmatrix}=1 \cdot 1 \cdot 1+ 1 \cdot 1 \cdot 1+1\cdot 1 \cdot 1 — 1 \cdot 1 \cdot 1 — 1 \cdot 1 \cdot 1 — 1 \cdot 1 \cdot 1=1+1+1-1-1-1=0

      Действительно, в столбцах единичной матрицы три совпадающих вектора, на которых невозможно построить объемную фигуру, объем которой и определяет определитель матрицы третьего порядка. Поэтому мы и получили число 0.

      Вообще говоря, любая матрица с одинаковыми строками и столбцами дает определитель, равный нулю. Можете проверить самостоятельно.

      Ответ: 0

      Разложение определителя по строке или столбцу, а также его свойства, миноры и дополнения элементов определителя рассмотрим далее.

      Функция МОПРЕД — Служба поддержки Майкрософт

      Excel для Microsoft 365 Excel для Microsoft 365 для Mac Excel для Интернета Excel 2021 Excel 2021 для Mac Excel 2019 Excel 2019 для Mac Excel 2016 Excel 2016 для Mac Excel 2013 Excel 2010 Excel 2007 Excel для Mac 2011 Excel Starter 2010 Еще. ..Меньше

      В этой статье описаны синтаксис формулы и использование MDETERM  в Microsoft Excel.

      Описание

      Возвращает определитель матрицы (матрица хранится в массиве).

      Синтаксис

      МОПРЕД(массив)

      Аргументы функции МОПРЕД описаны ниже.

      Замечания

      • Массив может быть задан как интервал ячеек, например A1:C3, как массив констант, например {1;2;3:4;5;6:7;8;9}, как имя для интервала или массива.

      • Функция МОПРЕД возвращает значение ошибки #ЗНАЧ! в случаях, указанных ниже.

      • org/ListItem»>

        Определитель матрицы — это число, вычисляемое на основе значений элементов массива. Для массива A1:C3, состоящего из трех строк и трех столбцов, определитель вычисляется следующим образом:

      МОПРЕД(A1:C3)
      равно A1*(B2*C3-B3*C2) + A2*(B3*C1-B1*C3) + A3*(B1*C2-B2*C1)

      • Определители матриц обычно используются при решении систем уравнений с несколькими неизвестными.

      • Функция МОПРЕД производит вычисления с точностью примерно 16 значащих цифр, что может в некоторых случаях приводить к незначительным ошибкам. Например, определитель сингулярной матрицы отличается от нуля на 1E-16.

      Пример

      Скопируйте образец данных из следующей таблицы и вставьте их в ячейку A1 нового листа Excel. Чтобы отобразить результаты формул, выделите их и нажмите клавишу F2, а затем — клавишу ВВОД. При необходимости измените ширину столбцов, чтобы видеть все данные.

      Данные

      Данные

      Данные

      Данные

      1

      3

      8

      5

      1

      3

      6

      1

      1

      1

      1

      0

      7

      3

      10

      2

      Формула

      Описание

      Результат

      =МОПРЕД(A2:D5)

      Определитель приведенной выше матрицы.

      88

      =МОПРЕД({3;6;1:1;1;0:3;10;2})

      Определитель матрицы, представленной в виде массива констант.

      1

      =МОПРЕД({3;6:1;1})

      Определитель матрицы в массиве констант.

      -3

      =МОПРЕД({1;3;8;5:1;3;6;1})

      Возвращает сообщение об ошибке, так как массив имеет разное количество строк и столбцов.

      #ЗНАЧ!

      Собственные значения и собственные векторы матриц 3 на 3

      Собственные значения и собственные векторы матриц 3 на 3

      Так же, как матрицы 2 на 2 могут представлять преобразования плоскости, матрицы 3 на 3 могут представлять преобразования трехмерного пространства. Картина более сложная, но, как и в случае 2 на 2, наши лучшие идеи приходят от нахождения собственных векторов матрицы: то есть тех векторов, направление которых преобразование оставляет неизменным.

      Если ненулевое {\bf e} является собственным вектором матрицы A 3 на 3, то

      А\,{\bf е}=\лямбда \,{\bf е}

      для некоторого скаляра \lambda . Этот скаляр называется собственным значением A.

      Это может быть переписано

      А\,{\bf е}=\лямбда \,I\,{\bf е},

      и в свою очередь как

      \влево(А\,-\лямбда \,I\вправо)\,{\bf e}={\bf 0}.

      Как и в случае 2 на 2, матрица A-\lambda \,I должна быть вырожденной. Тогда мы еще раз спрашиваем: при каких значениях \lambda A-\lambda \,I является сингулярным? То есть значения, удовлетворяющие характеристическому уравнению

      \text{det}\left(A-\lambda \,I\right)\,=\,0?

      Рассмотрим пример

      \left(\begin{array}{ccc}-2&-4&2\\-2&1&2\\4&2&5\end{массив}\right).

      Характеристическое уравнение

      \text{det}\left(\begin{array}{ccc}-2-\lambda &-4&2\\-2&1-\lambda &2\\4&2&5-\lambda \end{array}\right)=0 .

      Раскрывая определитель,

      \влево(-2-\лямбда\вправо)[\влево(1-\лямбда\вправо)\влево(5-\лямбда\вправо)-2\умножить на 2]+4[\влево(-2\вправо) )\times \left(5-\lambda\right)-4\times 2]+2[\left(-2\right)\times 2-4\left(1-\lambda\right)]=0. 9{2}-\лямбда-30\вправо)=\влево(\лямбда-3\вправо)\влево(\лямбда+5\вправо)\влево(\лямбда-6\вправо),

      это означает, что собственные значения равны 3, -5 и 6.

      Теперь приступаем к решению

      \left(\begin{array}{ccc}-2-\lambda &-4&2\\-2&1-\lambda &2\\4&2&5-\lambda \end{array}\right)\left(\begin{array }{c}X\\Y\\Z\конец{массив}\right)=\left(\begin{массив}{c}0\\0\\0\end{массив}\right)

      для каждого собственного значения \lambda . Теперь каждая такая система будет иметь бесконечно много решений, потому что если {\bf e} является собственным вектором, то и любой кратен {\bf e}. Таким образом, наша стратегия будет заключаться в том, чтобы попытаться найти собственный вектор с X = 1, а затем, при необходимости, увеличить его. (Если такого собственного вектора нет, мы знаем, что X на самом деле должно быть равно нулю, и вместо этого мы ищем собственный вектор с Y = 1 и так далее.)

      Собственный вектор, соответствующий собственному значению 3

      В случае \lambda =3 имеем

      \left(\begin{array}{ccc}-5&-4&2\\-2&-2&2\\4&2&2\end{массив}\right)\left(\begin{array}{c}X\\Y\ \Z\end{массив}\right)=\left(\begin{array}{c}0\\0\\0\end{массив}\right).

      Установка X=1 дает, как и наши первые два уравнения,

      -5-4Y+2Z=0,

      -2-2Y+2Z=0.

      Вычитание первого из второго:

      3+2Г=0,

      и, таким образом, Y=-\frac{3}{2}.

      Подставляя обратно во второе уравнение,

      -2+3+2Z=0,

      что дает Z=-\frac{1}{2}.

      Проверяя третье уравнение,

      4-3-1=0,

      который работает. Это дает нам собственный вектор

      \left(1,-\frac{3}{2},-\frac{1}{2}\right).

      Для удобства мы можем увеличить масштаб в 2 раза, чтобы получить

      \влево(2,-3,-1\вправо).

      Собственный вектор, соответствующий собственному значению -5

      В случае \lambda =-5 имеем

      \left(\begin{массив}{ccc}3&-4&2\\-2&6&2\\4&2&10\end{массив}\right)\left(\begin{массив}{c}X\\Y\\Z\ конец {массив}\справа)=\влево(\начало{массив}{с}0\\0\\0\конец{массив}\справа).

      Установка X=1 дает, как и наши первые два уравнения,

      3-4Y+2Z=0,

      -2+6Y+2Z=0.

      Вычитание первого из второго:

      -5+10Y=0,

      и, таким образом, Y=\frac{1}{2}.

      Подставляя обратно во второе уравнение,

      -2+3+2Z=0,

      что дает Z=-\frac{1}{2}.

      Проверяя третье уравнение,

      4+1-5=0,

      который работает. Это дает нам собственный вектор

      \left(1,-\frac{1}{2},\frac{1}{2}\right).

      Еще раз, мы можем масштабироваться в 2 раза, чтобы получить

      \влево(2,-1,1\вправо).

      Собственный вектор, соответствующий собственному значению 6

      В случае \lambda =6 имеем

      \left(\begin{array}{ccc}-8&-4&2\\-2&-5&2\\4&2&-1\end{массив}\right)\left(\begin{array}{c}X\\ Y\\Z\end{массив}\right)=\left(\begin{array}{c}0\\0\\0\end{массив}\right).

      Установка X=1 дает, как и наши первые два уравнения,

      -8-4Y+2Z=0,

      -2-5Y+2Z=0.

      Вычитание первого из второго:

      6-Y=0,

      и, таким образом, Y=6.

      Подставляя обратно во второе уравнение,

      -2-30+2Z=0,

      давая Z=16.

      Проверяя третье уравнение,

      4+12-16=0,

      который работает. Это дает нам собственный вектор

      \влево(1,6,16\вправо).

      Общие Соображения

      В общем, собственные значения реальной матрицы 3 на 3 могут быть

      (i) три различных действительных числа, как здесь;

      (ii) три действительных числа с повторениями;

      (iii) одно действительное число и два сопряженных недействительных числа.

      Геометрическая интерпретация преобразования зависит от того, что из вышеперечисленного верно: первое будет включать растяжение в трех направлениях собственного вектора, третье — вращение и растяжение вдоль своей оси, а второе — обычно один из нескольких типов трехмерного изображения. сдвиг. Однако мы оставляем эти детали на данный момент.

      Еще одним важным применением собственных значений и собственных векторов является диагонализация, и именно к ней мы сейчас и обратимся.

      Создано с помощью пакета ExportAsWebPage в Wolfram Mathematica 7.0

      Определители матриц 2 на 2 и 3 на 3

      Привет. Этот пост посвящен теме линейной алгебры вычисления определителей матриц 2 на 2 и 3 на 3. В этом посте будут рассмотрены формулы, а не метод кофакторов. Метод кофакторов при вычислении определителей будет в другом посте.

      Введение

       

      Определитель матрицы — это число. Сам по себе он не имеет большого применения, но влияет на множество результатов в математической области линейной алгебры.

      Обозначение определителя матрицы A часто обозначается как \(\text{det}(A)\). Иногда вместо квадратных скобок, подобных этим [ ], у вас будут вертикальные черточки (как абсолютные значения), обозначающие определитель матрицы. Вот пример определителя 3 на 3:

       

      \[\displaystyle \text{A} = \begin{vmatrix} a & d & g \\ b & e & h\\ c & f & i \\ \end{vmatrix} \]

       

      Вычисление определителей 2 на 2 и 3 на 3

       

      Существует альтернативный и более простой способ вычисления определителей для матриц 2 на 2 и 3 на 3. У нас есть формулы и средства памяти для вычисления таких определителей. Они не работают для матриц более высокой размерности, таких как 4 на 4 или матрица 1000 на 1000.

       

      Два на два Определяющий случай

       

      Предположим, что квадратная матрица A 2 на 2 имеет вид:

       

      \[\displaystyle \text{A} = \begin{bmatrix} а & b \\ c & d \\ \end{bmatrix} \]

       

      Определитель A равен \(\text{det}(A) = ad — bc\).

      Учитывая квадратную матрицу A 2 на 2 формы, как указано выше, и определитель отличен от нуля, тогда обратная матрица A:

        9{-1} = \dfrac{1}{\text{det}(A)}\begin{bmatrix} d & -b \\ -c & a \\ \end{bmatrix} \, = \, \dfrac{ 1}{(ad — bc)}\begin{bmatrix} d & -b \\ -c & a \\ \end{bmatrix} \]

       

      Записи \(a\) и \(d\) переключаются, а записи \(b\) и \(c\) меняют знаки. Кроме того, вы видите, почему определитель должен быть ненулевым. Математическая полиция будет преследовать вас, если вы разделите на ноль!

       

      Три на три Определяющий случай

       

      Формула для вычисления определителя матрицы 3 на 3 сложнее, чем для случая 2 на 2, но она не слишком сложна, как только вы ее поймете.

      Предположим, что матрица A имеет вид:

      \[\displaystyle \text{A} = \begin{bmatrix} a & b & c \\ d & e & f\\ e & f & g \\ \ end{bmatrix} \]

      Чтобы вычислить определитель 3 на 3, мы добавим еще один столбец a,b,c и еще один столбец d,e,f справа от A. Рисунок ниже поможет проиллюстрировать это.

       

      «Трудная» часть

       

      Объяснять словами несколько долго. Если вы хотите увидеть (и краткое изложение) приведенных ниже шагов, обратитесь к изображению ниже.

       

      Источник изображения

       

      Источник ссылки на изображение: http://thejuniverse.org/PUBLIC/LinearAlgebra/LOLA/detDef/special.html

       

      Начиная с верхнего левого \(a\) элемента, мы проводим нисходящую правую диагональную линию к получите один из шести терминов, который равен \(aei\). Затем от записи \(b\) в строке 1 столбца 2 проведите еще одну нисходящую правую диагональную линию, чтобы получить терм \(bfg\). Продолжайте от \(c\) в строке 1, столбце 3 и до того же, чтобы получить \(cdh\).

      (Напомним, что строка 1 — это верхняя строка матрицы, а столбец 1 — крайний левый вертикальный столбец.)

      Пока у нас есть \(aei + bfg + cdh\).

      Чтобы получить последние три члена, мы проделываем аналогичную процедуру, но начинаем сверху справа и идем вниз влево.

      X 2 2y 2 0: Mathway | Популярные задачи

      36Risolvere per ?cos(x)=1/27Risolvere per xsin(x)=-1/28Преобразовать из градусов в радианы2259Risolvere per ?cos(x)=( квадратный корень из 2)/210Risolvere per xcos(x)=( квадратный корень из 3)/211Risolvere per xsin(x)=( квадратный корень из 3)/212Графикg(x)=3/4* корень пятой степени из x13Найти центр и радиусx^2+y^2=914Преобразовать из градусов в радианы120 град. 2+n-72)=1/(n+9)

      Mathway | Популярные задачи

      1Найти точное значениеsin(30)
      2Найти точное значениеsin(45)
      3Найти точное значениеsin(30 град. )
      4Найти точное значениеsin(60 град. )
      5Найти точное значениеtan(30 град. )
      6Найти точное значениеarcsin(-1)
      7Найти точное значениеsin(pi/6)
      8Найти точное значениеcos(pi/4)
      9Найти точное значениеsin(45 град. )
      10Найти точное значениеsin(pi/3)
      11Найти точное значениеarctan(-1)
      12Найти точное значениеcos(45 град. )
      13Найти точное значениеcos(30 град. )
      14Найти точное значениеtan(60)
      15Найти точное значениеcsc(45 град. )
      16Найти точное значениеtan(60 град. )
      17Найти точное значениеsec(30 град. )
      18Найти точное значениеcos(60 град. )
      19Найти точное значениеcos(150)
      20Найти точное значениеsin(60)
      21Найти точное значениеcos(pi/2)
      22Найти точное значениеtan(45 град. )
      23Найти точное значениеarctan(- квадратный корень из 3)
      24Найти точное значениеcsc(60 град. )
      25Найти точное значениеsec(45 град. )
      26Найти точное значениеcsc(30 град. )
      27Найти точное значениеsin(0)
      28Найти точное значениеsin(120)
      29Найти точное значениеcos(90)
      30Преобразовать из радианов в градусыpi/3
      31Найти точное значениеtan(30)
      32Преобразовать из градусов в радианы45
      33Найти точное значениеcos(45)
      34Упроститьsin(theta)^2+cos(theta)^2
      35Преобразовать из радианов в градусыpi/6
      36Найти точное значениеcot(30 град. )
      37Найти точное значениеarccos(-1)
      38Найти точное значениеarctan(0)
      39Найти точное значениеcot(60 град. )
      40Преобразовать из градусов в радианы30
      41Преобразовать из радианов в градусы(2pi)/3
      42Найти точное значениеsin((5pi)/3)
      43Найти точное значениеsin((3pi)/4)
      44Найти точное значениеtan(pi/2)
      45Найти точное значениеsin(300)
      46Найти точное значениеcos(30)
      47Найти точное значениеcos(60)
      48Найти точное значениеcos(0)
      49Найти точное значениеcos(135)
      50Найти точное значениеcos((5pi)/3)
      51Найти точное значениеcos(210)
      52Найти точное значениеsec(60 град. )
      53Найти точное значениеsin(300 град. )
      54Преобразовать из градусов в радианы135
      55Преобразовать из градусов в радианы150
      56Преобразовать из радианов в градусы(5pi)/6
      57Преобразовать из радианов в градусы(5pi)/3
      58Преобразовать из градусов в радианы89 град.
      59Преобразовать из градусов в радианы60
      60Найти точное значениеsin(135 град. )
      61Найти точное значениеsin(150)
      62Найти точное значениеsin(240 град. )
      63Найти точное значениеcot(45 град. )
      64Преобразовать из радианов в градусы(5pi)/4
      65Найти точное значениеsin(225)
      66Найти точное значениеsin(240)
      67Найти точное значениеcos(150 град. )
      68Найти точное значениеtan(45)
      69Вычислитьsin(30 град. )
      70Найти точное значениеsec(0)
      71Найти точное значениеcos((5pi)/6)
      72Найти точное значениеcsc(30)
      73Найти точное значениеarcsin(( квадратный корень из 2)/2)
      74Найти точное значениеtan((5pi)/3)
      75Найти точное значениеtan(0)
      76Вычислитьsin(60 град. )
      77Найти точное значениеarctan(-( квадратный корень из 3)/3)
      78Преобразовать из радианов в градусы(3pi)/4
      79Найти точное значениеsin((7pi)/4)
      80Найти точное значениеarcsin(-1/2)
      81Найти точное значениеsin((4pi)/3)
      82Найти точное значениеcsc(45)
      83Упроститьarctan( квадратный корень из 3)
      84Найти точное значениеsin(135)
      85Найти точное значениеsin(105)
      86Найти точное значениеsin(150 град. )
      87Найти точное значениеsin((2pi)/3)
      88Найти точное значениеtan((2pi)/3)
      89Преобразовать из радианов в градусыpi/4
      90Найти точное значениеsin(pi/2)
      91Найти точное значениеsec(45)
      92Найти точное значениеcos((5pi)/4)
      93Найти точное значениеcos((7pi)/6)
      94Найти точное значениеarcsin(0)
      95Найти точное значениеsin(120 град. )
      96Найти точное значениеtan((7pi)/6)
      97Найти точное значениеcos(270)
      98Найти точное значениеsin((7pi)/6)
      99Найти точное значениеarcsin(-( квадратный корень из 2)/2)
      100Преобразовать из градусов в радианы88 град.
      3-8 9 Оценить квадратный корень из 12 10 Оценить квадратный корень из 20 11 Оценить квадратный корень из 50 94 18 Оценить квадратный корень из 45 19 Оценить квадратный корень из 32 20 Оценить квадратный корень из 18 92-2y-2=0 Tiger Algebra Solver

      Пошаговое решение :

      Шаг 1 :

      Уравнение в конце шага 1 :
       (2  2  y  2  - 2г) - 2 = 0
       3. 1 2  =   2 • (2г  2  - г - 1)  

      Попытка факторинга путем разделения среднего члена

       3.2     Факторинг  2y 2 - y - 1

      Первый член равен 2y 2   его коэффициент равен 2 .
      Средний член равен  -y , его коэффициент равен -1 .
      Последний член, "константа", равен  -1 

      Шаг-1: умножьте коэффициент первого члена на константу   2 • -1 = -2 равен коэффициенту среднего члена, который равен   -1 .

            -2    +    1    =    -1    Вот и все


      Шаг 3. Перепишите полином, разделяющий средний член, используя два множителя, полученные на шаге 2. выше,  -2  и  1 
                                  2 года 2 - 2y + 1y - 1

      Шаг 4. Сложите первые 2 слагаемых, выделив одинаковые множители :
                           2y • (y-1)
                    Сложите последние 2 слагаемых, выделив общие множители :
                           1 • (y- 1)
      Шаг 5 : Сложите четыре члена шага 4 :
                          (2y+1)  •  (y-1)
                    Какая нужна факторизация

      Уравнение в конце шага 3  : 90 911
       2 • (у - 1 ) • (2у + 1) = 0
       

      Шаг 4 :

      Теория – корни произведения:

       4. 1    Произведение нескольких членов равно нулю.

       Если произведение двух или более слагаемых равно нулю, то хотя бы одно из слагаемых должно быть равно нулю.

       Теперь мы решим каждое слагаемое = 0 отдельно 

       Другими словами, мы собираемся решить столько уравнений, сколько членов в произведении 

       Любое решение term = 0 также решает произведение = 0.

      Уравнения, которые никогда не бывают истинными :

       4.2      Решите:    2   = 0

      Это уравнение не имеет решения.
      A ненулевая константа никогда не равна нулю.

      Решение уравнения с одной переменной :

       4.3      Решение  :    y-1 = 0 

       Добавьте  1 к обеим частям уравнения :  91  из обеих частей уравнения : 
                            2y = -1
      Разделите обе части уравнения на 2:
      y = -1/2 = -0,500

      Дополнение: прямое решение квадратного уравнения

       прямое решение 2y  2  -y-1 = 0 

      Ранее мы факторизовали этот многочлен, разделив средний член. давайте теперь решим уравнение, заполнив квадрат и используя квадратную формулу

      Парабола, нахождение вершины:

       5.1      Найдите вершину t = 2y 2 -y-1

      Параболы имеют самую высокую или самую низкую точку, называемую вершиной. Наша парабола раскрывается и, соответственно, имеет низшую точку (абсолютный минимум). Мы знаем это еще до того, как начертили «t», потому что коэффициент первого члена, 2 , положителен (больше нуля).

       Каждая парабола имеет вертикальную линию симметрии, проходящую через ее вершину. Из-за этой симметрии линия симметрии, например, будет проходить через середину двух точек пересечения x (корней или решений) параболы. То есть, если парабола действительно имеет два действительных решения.

       Параболы могут моделировать многие реальные жизненные ситуации, такие как высота над землей объекта, брошенного вверх через некоторый период времени. Вершина параболы может предоставить нам такую ​​информацию, как максимальная высота, на которую может подняться объект, брошенный вверх. По этой причине мы хотим иметь возможность найти координаты вершины.

       Для любой параболы, Ay 2 +By+C, y -координата вершины задается как -B/(2A) . В нашем случае координата y равна  0,2500  

       Подставив в формулу параболы 0,2500 вместо y, мы можем вычислить t-координату:
       t = 2,0 * 0,25 * 0,25 - 1,0 * 0,25 - 1,0
      или t = -1,125

      909 10 Парабола, графическая вершина и X-перехваты:

      Корневой график для:  t = 2y 2 -y-1
      Ось симметрии (штриховая)  {y}={ 0,25} 
      Вершина в  {y,t} = {0,25,-1,12} 
       y -Перехваты (корни ) :
      Корень 1 при {y,t} = {-0,50, 0,00} 
      Корень 2 при {y,t} = {1,00, 0,00} 

      Решите квадратное уравнение, заполнив квадрат

       5.2     Решение   2y 2 -y-1 = 0 путем заполнения квадрата .

       Поделите обе части уравнения на  2  , чтобы получить 1 в качестве коэффициента при первом члене:
         y 2 -(1/2)y-(1/2) = 0

      Добавьте  1/2  к обеим частям уравнения:
         y 2 -(1/2)y = 1/2

      Теперь немного хитрости: возьмем коэффициент y, который равен 1/2, разделим на два, получим 1/4, и, наконец, возвести его в квадрат, что дает 1/16 

      Добавить 1/16 к обеим частям уравнения:
        В правой части мы имеем :
         1/2  +  1/16   Общий знаменатель двух дробей равен 16   Сложение (8/16)+(1/16) дает 9/16 
       get :
         y 2 -(1/2)y+(1/16) = 9/16

      Добавление  1/16  завершило левую часть в полный квадрат:
         y 2 -(1/2 )y+(1/16)  =
         (y-(1/4)) • (y-(1/4))  =
        (y-(1/4)) 2
      Вещи, равные одному и тому же вещи также равны друг другу. С
         y 2 -(1/2)y+(1/16) = 9/16 и
         y 2 -(1/2)y+(1/16) = (y-(1/4)) 2
      , тогда, согласно закону транзитивности,
         (y-(1/4)) 2 = 9/16

      Мы будем называть это уравнение уравнением #5.2.1  

      Принцип квадратного корня гласит, что когда две вещи равны, их квадратные корни равны.

      Обратите внимание, что квадратный корень из
         (y-(1/4)) 2   равен
         (y-(1/4)) 2/2  =
        (y-(1/4)) 1  =
         y-(1/4)

      Теперь, применяя принцип квадратного корня к уравнению #5.2.1  получаем:
         y-(1/4) = √ 9/16

      Добавьте 1/4  к обеим частям, чтобы получить:
         y = 1/4 + √ 9/16

      Поскольку квадратный корень имеет два значения, одно положительное, а другое отрицательное 1/4 - √ 9/16

      Обратите внимание, что √ 9/16 можно записать как
        √ 9 / √ 16   что равно 3/4

      Решение квадратного уравнения с помощью квадратной формулы

       5. 3     Решение    2y 2 -y-1 = 0 с помощью квадратной формулы .

       Согласно квадратичной формуле,  y  , решение для Ay 2 +By+C = 0  , где A, B  и C – числа, часто называемые коэффициентами, определяется следующим образом:
                                           
                  - B  ±  √ B 2 -4AC
        y =   ————————
                           2A

        В нашем случае  A   =     2
                            B   =    -1
                             C   =   -1

      Соответственно ,  B 2   -  4AC   =
                           1 - (-8) =
                           9

      Применение формулы квадрата :

      90 924                1 ± √ 9
         y  =    ————
                         4

      Можно ли упростить √ 9 ?

      Да! Первичная факторизация 9   это
         3•3 
      Чтобы иметь возможность удалить что-то из-под корня, должно быть 2 экземпляра этого числа (потому что мы берем квадрат, т.е. второй корень).

      14 00 это сколько: Как по английски будет 7.00 7.10 7.15 7.20 7.30 13.15 13.20 14.00 16.00 16.45 17.15…

      Разница во времени между Новосибирском и Москвой

      Мировое время Конвертер времени Разница во времени между Новосибирском и Москвой

      24 timezonestz

      e.g. Москва, Новосибирск, Екатеринбург

      • X
      • Точное время

        Точное время в мире

        ГородаСтраныВремя по GMT Время UTC

        Сравнить время

      • Часовые пояса

        Часовые пояса мира

        UTCGMT

        Карта часовых поясов

      • Конвертер времени

        Узнать разницу во времениПопулярные сравнения

        Москва и КазахстанCET и МоскваМосква и ВладивостокPST и Киев

      • Карты

        Карта часовых поясов

        Часовые пояса мираАвстралияКанадаЕвропаСША

        Карта мира

      • Виджеты часов для сайта

        Цифровые часыАналоговые часыТекстовые часы

      • Анонс мероприятия
      • Свяжитесь с нами
      • ru

        en es pt fr ru nl da

      Время в Москве, Россия на 4:00 часа позади от времени в Новосибирске, Россия

      AM/PM

      Новосибирск, Россия

      —:—

       

      =

      Москва, Россия

      —:—

       

      Лучшее время для звонка из Новосибирска в Москву

      При планировании звонка между Новосибирском и Москвой необходимо учитывать, что города находятся в разных часовых поясах. Время в Новосибирске на 4 часа опережает время в Москве.

      Если вы находитесь в Новосибирске, то для проведения конференц-связи или встречи наиболее удобное время для всех сторон будет между 13:00 и 18:00. В Москве будет обычное рабочее время — между 09:00 и 14:00.

      Если вы хотите позвонить в Москву и вам подходит любое время, вы можете выбрать время между 11:00 и 03:00. Это будет между 07:00 и 23:00 по времени Москвы.

      Разница во времени Новосибирск Москва

      Новосибирск

      12:00 am 00:00

      01:00 am 01:00

      02:00 am 02:00

      03:00 am 03:00

      04:00 am 04:00

      05:00 am 05:00

      06:00 am 06:00

      07:00 am 07:00

      08:00 am 08:00

      09:00 am 09:00

      10:00 am 10:00

      11:00 am 11:00

      12:00 pm 12:00

      01:00 pm 13:00

      02:00 pm 14:00

      03:00 pm 15:00

      04:00 pm 16:00

      05:00 pm 17:00

      06:00 pm 18:00

      07:00 pm 19:00

      08:00 pm 20:00

      09:00 pm 21:00

      10:00 pm 22:00

      11:00 pm 23:00

      Москва MSK

      08:00 pm 20:00

      09:00 pm 21:00

      10:00 pm 22:00

      11:00 pm 23:00

      12:00 am 00:00

      01:00 am 01:00

      02:00 am 02:00

      03:00 am 03:00

      04:00 am 04:00

      05:00 am 05:00

      06:00 am 06:00

      07:00 am 07:00

      08:00 am 08:00

      09:00 am 09:00

      10:00 am 10:00

      11:00 am 11:00

      12:00 pm 12:00

      01:00 pm 13:00

      02:00 pm 14:00

      03:00 pm 15:00

      04:00 pm 16:00

      05:00 pm 17:00

      06:00 pm 18:00

      07:00 pm 19:00

      Анонсируйте событие для разных часовых поясов

       

      Copyright © 2005 — 2023 24TimeZones. com. Все права защищены. What time is it right now? 🙂

      Посмотрели на часы, а там ровно 14:00? Это неспроста! Суеверия на каждый час дня | Lifestyle

      Если вы посмотрели на часы, а там 12:00, то знайте, что это все не просто так. Каждый час дня или ночи имеет свое значение. Если вы хотите узнать эти суеверия, внимательно прочитайте следующие объяснения. Они помогут вам в дальнешей жизни.

      Что означает, когда часы показывают 00:00?

      Если посреди ночи вы проснулись, как Золушка на балу, это значит, что у вашей любовной истории будет счастливый конец, как в сказках. Так что если вы посмотрите на часы в 00:00, вы и ваш избранник будете жить долго и счастливо!

      Значение часа 01:00

      Вы часто несобраны в последнее время? Это значит, что вы влюблены и всегда думаете только о нем. И если вам случится смотреть на часы в 1:00 на этот раз, то означает, что ваша любовь очень сильна

      Какой секрет скрывается в 02:00?

      Если вам нравится конкретный человек и вы не знаете, как заставить его обратить на себя внимание, вам следует проявить немного терпения. Вскоре, похоже, вы получите неожиданное приглашение в город. Это также подтверждается тем фактом, что вы увидели на часах 14:00.

      Что нужно знать о 03:00?

      Когда вы смотрите на часы, а там 03:00, то знайте, что ваш партнер искренне любит вас и старается каждый день показывать вам, насколько сильны его чувства.

      Значение 04:00

      Если вы вовлечены в новые отношения, будьте готовы! Вскоре все станет серьезно.

      Что 05:00 означает?

      Если вы смотрите на часы и там 5 утра, то это означает, что ваш любимый скучает по вам. Так что если он далеко от вас, позвоните ему. Он наверняка будет рад услышать ваш голос.

      Каково значение 06:00?

      Если вы посмотрели на часы, а на них 6:00, то оставьте печаль в стороне, потому что вскоре вы получите должное внимание от кого-то нового знакомого.

      Тайны часа 7:00

      Вы рано проснулись и посмотрели на часы в 7:00? Вы узнаете совершенно неожиданные вещи о человеке, которого считаете неприятным. Похоже, у вас есть тайный поклонник!

      Значение часа 08:00

      Если вы посмотрели на часы в 8:00, это значит, что сегодня вы сможете выполнить все, что хотели. Удача на вашей стороне в любовной сфере. Поэтому наслаждайтесь и организуйте ужин при свечах или романтическую прогулку. Вы приятно удивите своего партнера!

      Что значит то, что вы посмотрите на часы в 9:00?

      Ваш любимый человек сообщит хорошие новости сегодня.

      Каково значение 10:00?

      Если вы посмотрели на часы в 10:00, вполне возможно, что сегодня у вас будет особый момент, который вы получите подарок. Скорее всего, вам сделают предложение.

      Значение 11:00

      Ваша любовь сильна и преодолеет любые препятствия. Наслаждайтесь моментами, проведенными вместе с любимым человеком.

      Секреты 12:00

      Время, когда солнце находится прямо над вашей головой, подтверждает, что с какими бы проблемами вы ни столкнулись до сих пор, они скоро будут решены.

      Что означает 13:00?

      Ваши отношения на правильном пути, но все равно нужно немного их пересмотреть чтобы не нарушить ваше и партнера спокойствие.

      Что означает 14:00?

      Ваш любовник — это не только человек, на которого вы можете положиться в любое время, но и ваш счастливый талисман.

      Таинственный час — 15:00

      Вы часто смотрите на часы в 15:00? Это означает, что есть человек, который был очарован вашим таинственным взглядом.

      Каково значение 16:00?

      Когда часы показывают вам 16:00, это означает, что у вас есть тайный поклонник, который стесняется показывать вам, что он чувствует. Попробуйте найти его, и вы не пожалеете об этом!

      Что означает 17:00?

      Вы получите приглашение на важное событие. Если вы одиноки, возможно, вы встретите своего партнера.

      Скрытые значения 18:00

      Если вы только в начале отношений, то партнер готов узнать вас лучше и лучше. Постарайтесь показать своему ему, насколько вы очаровательны.

      Значение 19:00

      Сосредоточьтесь на том, что важно для вас, и вы добьетесь успеха во всем, что вы предлагаете.

      Что говорит 20:00?

      Если в вашем кругу есть друг, который в последнее время странно себя ведет, он может испытывать к вам сильные чувства.

      Какие тайны скрывает 21:00?

      Даже если вы еще не встретили свою великую любовь, не нужно грустить. Имейте терпение и надежду, что любовь скоро войдет в вашу жизнь.

      Каково значение 22:00?

      Если вы смотрите на часы в 10 вечера, это означает, что вы ревнивый человек. Ревность — это негативное чувство. Постарайся сдержать ее, если хотите быть спокойным.

      Значение 23:00

      23:00 означает, что чувства вашего партнера усиливаются. Покажите ему, что он не ошибся, когда выбрал вас!

      Раскрыть значение фиксированного времени может быть интересно. Нет необходимости следить за часами, судьба позаботится о том, чтобы вы посмотрели на часы в нужное время и нашли сообщение, которое предназначено для вас.

      Нашли нарушение? Пожаловаться на содержание

      Convert $ 1400 US Dollars

      . = 697,97 доллары США 9000 4000 4000 4000 4000 4000 4000 4000 4000 4000 4000 4000 4000 4000 4000 4000 4000 4000. = 45,3 доллара США 9000 1400 14004 USD USD. 0005 9000 9000 14004 14004.0005 9.0004 1400 USD до HNL 9.000300 Иранский риал11111111111111111111111111111111111111111111111111111111111111111111111111111111ЖВАНСИВНЫЙ. KES до USD 9000 14004 14004 US.0005 9000 9000 40004.9000.9000.9000.9000.9000.9000. 9000. 9000.9000.9000.9000.9000.9000.9000.9000.9000.9000.9000. 9000. 9000. 9000. 9000. 9000. 9000. 9000. 9000. 9000.9000. 9000. 9000. 9000. 9000. 9000. 9000. 9000. 9000. 9000. 9000. 9000. 9000. 9000. 9000. 1400 KPW в KPW 9003 9000 9000 9 000 9.0008 70013777777400 3 9005
      1400 USD to AED = 5141. 78 UAE Dirham 1400 AED to USD = 381.19 US Dollars
      1400 USD to AFN = 122640.6 Afghani 1400 AFN до = 15,98 доллара США
      1400 USD для всех = 150149,9 LEK 1400 Все до = 13,05 доллара США
      1400 USD до AMD 9D 9000
      1400 USD.0005 = 540318.44 Dram 1400 AMD to USD = 3.63 US Dollars
      1400 USD to ANG = 2510.83 Guilder 1400 ANG to USD = 780.62 US Dollars
      1400 USD до AOA = 710500 Kwanza 1400 AOA до = 2,76 доллара США
      1400 USD TO ARS = 281080.05 Argentine Peso 1400 до USD 900.0005 = 6.97 US Dollars
      1400 USD to AUD = 2126.69 AU dollar 1400 AUD to USD = 921. 62 US Dollars
      1400 USD to AWG = 2521.75 Aruban Florin 1400 AWG до USD = 777,24 доллара США
      1400 USD до AZN = 2380 Azeri Manat 1400 AZN до USD = 823.53 US
      14004 14004 9000 4004.0005 = 2573.06 Convertible Mark 1400 BAM to USD = 761.74 US Dollars
      1400 USD to BBD = 2800 Barbados Dollar 1400 BBD to USD = 700 US Dollars
      1400 долл. США до BDT = 146860,88 TAKA 1400 BDT до = 13,35 доллара США
      1400 USD до BGN = 2567.27 Laverian 9000 14.0005 = 763.46 US Dollars
      1400 USD to BHD = 527.95 Bahraini Dinar 1400 BHD to USD = 3712.46 US Dollars
      1400 USD to BIF = 2896563. 25 Burundi Franc = 1400 бермудских долларов0003 1400 USD to BND = 1887.5 Brunei Dollar 1400 BND to USD = 1038.41 US Dollars
      1400 USD to BOB = 9626.27 Boliviano 1400 BOB to USD = 203.61 US Доллары
      1400 USD до BRL = 7304.08 Бразилия Реал 1400 BRL до = 268,34 доллара США
      1400 USD до BSD = 14003
      1400 USD.0005 1400 BSD до = 1400 долларов США
      1400 USD до BTC = 0,07 Bitcoin 1400 BTC до = 28860159.75.75) US0013 9000.9003 9000 9000 9000 9000 9000 9000 9000 9000 9000 9000 9000 9000 9000 9000 9000 9000 9000 9000 9000 9000 9000 9000 9000 9000 9000 9000 9000 9000 9000 9000 9000 9000 9000 9000 9000 9000 9000 9000 9000 9000 9000 9000 9000 9000 9000 9000 9000 9000 9000 9000 9000 9000 9000 9000 9000 9000 9000 9000 9000 9000 9000 = 28860159. 75. 114258.38 Bhutanese Ngultrum 1400 BTN to USD = 17.15 US Dollars
      1400 USD to BWP = 18526.34 Botswana Pula 1400 BWP to USD = 105.8 US Dollars
      1400 USD до BYN = 3516,48 от рубля 1400 до = 557,38 доллара США
      1400 USD до CAD = 1940,33 CAN Dollar 1400 CAD до = 1010,14 доллара США
      1400 USD TO CDF = 295
      1400 USD.04725.41 Congo Franc 1400 CDF to USD = 0.67 US Dollars
      1400 USD to CHF = 1288.08 Swiss Franc 1400 CHF to USD = 1521.64 US Dollars
      1400 USD to CLF = 40,42 UF Chile 1400 CLF до = 48486,53 доллара США
      1400 USD до CLP = 1115408 Chilean Peso 14004 CLP TO US,8. 0005 = 1.76 US Dollars
      1400 USD to CNY = 9668.68 Yuan 1400 CNY to USD = 202.72 US Dollars
      1400 USD to COP = 6611962.63 Colombian Peso 1400 COP до = 0,3 доллара США
      1400 USD до CRC = 763708,72 COLUN 1400 CRC до = 2,57 доллара США
      1400 USD до Cuct
      1400 USD.0005 = 1400 Convertible Peso 1400 CUC to USD = 1400 US Dollars
      1400 USD to CUP = 36050 Cuban Peso 1400 CUP to USD = 54.37 US Dollars
      1400 долл. США до CVE = 145058,49 CAPE VERDEAN ESCUDO 1400 CVE до = 13,51 доллара США
      1400 USD до CZK = 31092.88 Czech Koruna = 31092. 88.0004 1400 CZK to USD = 63.04 US Dollars
      1400 USD to DJF = 248057.19 Djibouti Franc 1400 DJF to USD = 7.9 US Dollars
      1400 USD to DKK = 9793.14 Датский Кроне 1400 DKK до = 200,14 доллара США
      1400 USD до DOP = 76859.91 Dominican Peso 1400 DOP TO USD = 25.5. Долл 1400 Дол.0005
      1400 USD до DZD = 1.2 Алжирский динар 1400 DZD до = 10,25 доллара США
      1400 USD до EGP
      1400 USD до ERN = 21000 NAKFA 1400 ERN до = 93,33 доллары США
      1400 USD до = 749943.64 Birr 1400 ETB to USD = 26. 15 US Dollars
      1400 USD to EUR = 1314.99 Euro 1400 EUR to USD = 1490.51 US Dollars
      1400 USD to FJD = 3131,38 Фиджи доллар 1400 FJD до = 625,92 доллара США
      1400 USD до FKP = 1163.37 Falkland Pound 1400 FKP до USD 9000 9000 9000 9000 9000 9000 9000 9000 9000 9000 9000 9000 9000 9000 9000 9000 9000 9000 9000 9000 9000 9000 9000 9000 9000 9000 9000 9000 = 1684.76 US Dollars
      1400 USD to GBP = 1163.37 Pound Sterling 1400 GBP to USD = 1684.76 US Dollars
      1400 USD to GEL = 3612 Lari 1400 GEL до = 542,64 доллара США
      1400 USD до GGP = 1163,37 Guernsey Pound 1400 GGP до = 1684,76 US Dollars
      = 17205.47 Cedi 1400 GHS to USD = 113.92 US Dollars
      1400 USD to GIP = 1163.37 Gibraltar Pound 1400 GIP to USD = 1684.76 US Dollars
      1400 USD до GMD = 85750 DALASI 1400 GMD до = 22,86 доллара США
      1400 USD до GNF = 11994255.73 FRANC = 0.16 US Dollars
      1400 USD to GTQ = 10880.05 Quetzal 1400 GTQ to USD = 180.15 US Dollars
      1400 USD to GYD = 293958.54 Guyana Dollar 1400 GYD до = 6,67 доллара США
      1400 USD до HKD = 10989,79 HK Dollar 1400 HKD до = 178,35 US Dollars
      = 34357 Lempira 1400 HNL до = 57,05 доллара США
      1400 USD до HRK = 9915. 11 KUNA15 9000 4.7004..9004..9004 14.7004...9004 14.7004...9004 14.700 4.7004.15.9004 9000 4.7004.
      1400 USD до HTG = 213798,81 Gourdes 1400 HTG до = 9,17 доллара США
      1400 USD до HUF = 504997 для 9000 9004 9000 4004 9000 4008 9000 4004 9000 4004 9000 4004 9000 4004 9000 4004 9000 4004 9000 4004 9000 4004 9000 4004.0005 = 3,89 доллара США
      1400 USD до IDR = 21703570 Индонезия Рупия 1400 IDR до = 0,09 доллара
      14005 до рент. 1400 ILS до = 390,43 доллара США
      1400 USD до IMP = 1163,37 MANX FOUNT 1400 IMP до = 1684. 76 US Dollars
      1400 USD до INR = 114751,7 Индийская рупия 1400 INR до = 17,08 доллара США
      1400 USD до IQD = 2033261,43 9000 9000 4000 4000 4000 4000 4000 4000 4000444844448494448444844844844844844849448494484448448448448448448944444448. Доллары США
      1400 USD до IRR = 5 1400 IRR — = 0,03 доллары США
      1400 USD до ISK = 1977999999
      1400 USD до ISK = 197799999992 Icelandic Krona 1400 ISK to USD = 9.91 US Dollars
      1400 USD to JEP = 1163.37 Jersey Pound 1400 JEP to USD = 1684.76 US Dollars
      1400 USD to JMD = 213114.67 Jam Dollar 1400 JMD до = 9,2 доллара США
      1400 USD до JOD = 993. 16 Jordanian Dinar 1400 до USD 9000 9000 9000 9.0005 = 1973,5 доллара США
      1400 USD до JPY = 188992,92 ИЕН 1400 JPY до = 10,37 доллара США
      14004 = 10,91 доллар США
      1400 USD до KGS = 122388 Kyrgyz SOM 1400 KGS = 16,01 US Dollars
      = 5637345.47 Камбоджиан RIEL 1400 кГР до = 0,35 доллара США
      1400 USD до KMF = 646589.58 FRAND = 1260000 Северокорейских вон0005 1400 KRW to USD = 1. 06 US Dollars
      1400 USD to KWD = 429.85 Kuwaiti Dinar 1400 KWD to USD = 4559.76 US Dollars
      1400 USD to KYD = 1161,02 Cayman Dollar 1400 KYD до = 1688,17 доллара США
      1400 USD до KZT = 6298455.39 Tenge 1400 KZT до USD = 3,114 = 3,114 = 3,114 = 3,114 = 3,114.0005
      1400 USD to LAK = 23506111.55 Lao Kip 1400 LAK to USD = 0.08 US Dollars
      1400 USD to LBP = 202.47 Lebanon Pound 1400 LBP to USD = 0,09 доллары США
      1400 USD до LKR = 452804.31 Sri Lankan Rupee 1400 LKR до = 4,33 доллара США
      1400 USD до LRD
      1400 USD. 0005 = 224700.05 Liberian Dollar 1400 LRD to USD = 8.72 US Dollars
      1400 USD to LSL = 25544.82 Lesotho Loti 1400 LSL to USD = 76.73 US Dollars
      1400 долл. США до LYD = 6758,26 Libyan Dinar 1400 LYD до = 290,02 доллара США
      1400 USD.0008 1400 MAD to USD = 135.4 US Dollars
      1400 USD to MDL = 26224.72 Moldovan Leu 1400 MDL to USD = 74.74 US Dollars
      1400 USD to MGA = 5993881.21 Malagasy Ariary 1400 MGA to USD = 0.33 US Dollars
      1400 USD to MKD = 81070.34 Macedonian Denar 1400 MKD to USD = 24.18 US Dollars
      1400 USD to MMK = 28.95 Kyat 1400 MMK to USD = 0. 67 US Dollars
      1400 USD to MNT = 4769751.37 Tugrik 1400 MNT to USD = 0.41 Доллары США
      1400 USD TO MOP = 11264.07 Macau Pataca 1400 MOP до = 174 доллара США
      1400 USD TO MRO = 0 MARITIAN 14004 1400 USD TO MRO = 0 MARITIAN 14004.0005 1400 MRO to USD = INF US Dollars
      1400 USD to MUR = 65877.72 Mauritian Rupee 1400 MUR to USD = 29.75 US Dollars
      1400 USD to MVR = 21504 Rufiyaa 1400 MVR to USD = 91.15 US Dollars
      1400 USD to MWK = 1416907.19 Malawi Kwacha 1400 MWK to USD = 1.38 US Dollars
      1400 USD to MXN = 25890.76 Mexican Peso 1400 MXN to USD = 75. 7 US Dollars
      1400 USD to MYR = 6327.3 Ringgit 1400 MYR to USD = 309,77 доллары США
      1400 USD до MZN = 89459,99 Метальный 1400 MZN до = 21,91 доллары США
      1400 USD до = 256999913
      1400 USD.7 Namibian доллар 1400 NAD до = 76,27 доллара США
      1400 USD до NGN = 641619.91 NIGERIAN NIARA 1400 Нг. Nio = 50953,34 Nicaraguan Cordoba 1400 Nio до = 38,47 доллара США
      1400 USD = 14935.38 KRONE 14004 = 14935.38.0005 = 131.23 US Dollars
      1400 USD to NPR = 182813.71 Nepali Rupee 1400 NPR to USD = 10.72 US Dollars
      1400 USD to NZD = 2283.85 NZ Dollar 1400 NZD до = 858,2 доллара США
      1400 USD до OMR = 539. 13 Омандиаал 1400 OMR до = 3635.46 США
      = 3635.46 США
      = 3635.46 США
      0004 1400 USD до PAB = 1400 BALBOA 1400 PAB до = 1400 долларов США
      1400 USD до ручки = 5272.81 Preuvian Nuevo Sol 14004. Доллары
      1400 USD до PGK = 4909,83 KINA 1400 PGK до = 399,2 доллара США
      1400 USD до PHP = 77251.9999 1400 USD до PHP = 77251.9999 1400 4000.0005 1400 PHP to USD = 25.37 US Dollars
      1400 USD to PKR = 3.64 Pakistani Rupee 1400 PKR to USD = 5.01 US Dollars
      1400 USD to PLN = 6156.01 Zloty 1400 PLN до = 318,39 доллары США
      1400 USD до PYG = 10028771,58 Гуарани 1400 PYG до USD 9000. 9000.9000.0005
      1400 USD to QAR = 5097.4 Qatar Riyal 1400 QAR to USD = 384.51 US Dollars
      1400 USD to RON = 6466.32 Romanian Leu 1400 RON to USD = 303,11 доллара США
      1400 USD до RSD = 153972 Сербский Динар 1400 RSD до = 12,73 доллара США
      1400 USD.0004 = 106680.01 Russian Ruble 1400 RUB to USD = 18.37 US Dollars
      1400 USD to RWF = 1521761.28 Rwandan Franc 1400 RWF to USD = 1.29 US Dollars
      1400 USD до SAR = 5256,9 Saudi Riyal 1400 SAR до = 372,84 доллара США
      1400 USD до SBD = 11478,5 Соломон Доллар 9.0004 1400 SBD to USD = 170. 75 US Dollars
      1400 USD to SCR = 18249.34 Seychelles Rupee 1400 SCR to USD = 107.4 US Dollars
      1400 USD to SDG = 830900 Sudan Pound 1400 SDG до = 2,36 доллара США
      1400 USD до SEK = 14996,38 Swedish Krona 1400 SEK TO = 130,7 US Dollars 1400 SEK TO = 130,7 US Dollars 9000 1400 SEK TO = 130,7 US Dollars 14004.0005
      1400 USD to SGD = 1888.46 Singapore Dollar 1400 SGD to USD = 1037.88 US Dollars
      1400 USD to SHP = 1163.37 Saint Helena Pound 1400 SHP to USD = 1684,76 доллары США
      1400 USD до SLL = 24731000 LEONE 1400 SLL до = 0,08 доллара США
      1400 USD = 0,08 доллара США
      1400 USD. 0005 = 7.77 Somali Shilling 1400 SOS to USD = 2.47 US Dollars
      1400 USD to SRD = 47795.3 Suriname Dollar 1400 SRD to USD = 41.01 US Dollars
      1400 долл. США до SSP = 182364 Южный Суданский фунт 1400 SSP до = 10,75 доллара США
      1400 USD до STD = 31953586,71 DOBRA 1400 STD to USD = 0.06 US Dollars
      1400 USD to SVC = 12190.39 Salvador Colon 1400 SVC to USD = 160.78 US Dollars
      1400 USD to SYP = 3517542 Сирийский фунт 1400 SYP до = 0,56 доллара США
      1400 USD до SZL = 25534.56 Swazi Lilangeni 1400400 SZL TO USD 9000 46.76.76.76.76.76.76.76.76.76.76.76.76.76.76.76.76.76.76. 76.76.76.76.76.76.76.76.76.76.76.76.76.76.76.76.76.76.76.76.76.76.76.76.76.76.76.76.76.76.76.76.76.76.76.76.76.76.76.76.76.76.0005
      1400 USD до THB = 48735.23 Thai Baht 1400 THB до = 40,22 доллара США
      1400 9000 9000. 9000.9000. 9000.9000. 9000.9000. 9000.9000. 9000.9000 9000. 9000.9000. 9000.9000. 9000.9000. 9000.9000 9000. = 139,81 доллар США
      1400 USD до TMT = 4914 Turkmen Manat 1400 TMT до = 398,86 доллара США
      1400 USD TO TND = 437.770.8 = 4370.8 = 4377.770.8 = 43777.870.8 = 437.870.8 = 437.89.8.0005 1400 TND to USD = 448.43 US Dollars
      1400 USD to TOP = 3317.4 Tonga Paanga 1400 TOP to USD = 590.82 US Dollars
      1400 USD to TRY = 26540,5 Турецкая лира 1400 Попробуйте = 73,85 доллара США
      1400 USD до TTD = 9468,4 Trinidad Dollar 1400 TTD. 0005
      1400 USD до TWD = 43181.18 Новый Тайваньский доллар 1400 TWD до = 45,39 доллары США
      1400 USD до TZS. = 0,6 доллара США
      1400 USD до UAH = 51419.73 HRYVNIA 1400 UAH до = 38,12 доллара США
      14005 USD TO TO UGX
      1400 USD TO TO UGX 9005 9000 = 5178204.19 Ugandan Shilling 1400 UGX to USD = 0.38 US Dollars
      1400 USD to UYU = 54878.59 Uruguayan Peso 1400 UYU to USD = 35.72 US Dollars
      1400 = 0.12 Долларов США0008 1400 VND to USD = 0.06 US Dollars
      1400 USD to VUV = 165261.6 Vanuatu Vatu 1400 VUV to USD = 11.86 US Dollars
      1400 USD to WST = 3776. 57 Samoa Tala 1400 WST до = 518,99 доллара США
      1400 USD до XAF = 862574.38 CAR FRANC 14004 XAF до U.SD = 277 USS = 2.27 US.277 US.277 US.277 US. 277 US = 2.27 US =.0005
      1400 USD to XCD = 3783.57 East Caribbean Dolla 1400 XCD to USD = 518.03 US Dollars
      1400 USD to XDR = 1047.48 Special Drawing Righ 1400 XDR to USD = 1871,15 доллара США
      1400 USD до XOF = 862574.38 CFA FRANC 1400 XOF до = 2,27 доллары США
      14004 Дол.0005 = 156919.59 CFP Franc 1400 XPF to USD = 12.49 US Dollars
      1400 USD to YER = 350489.95 Yemeni Rial 1400 YER to USD = 5.59 US Dollars
      1400 долл. США до ZAR = 25650.24 Южноафриканский Рэнд 1400 ZAR до = 76,41 доллара США
      1400 USD до ZMW = 28142.08 KWACHA = 28142.08 Kwacha0004 1400 ZMW to USD = 69.65 US Dollars
      1400 USD to ZWL = 450800 Zimbabwe Dollar 1400 ZWL to USD = 4.35 US Dollars

      Percentage Calculator. Что такое 1400 процентов?

      1400% от 1 = 14,00 1400% от 131 = 1834,00 1400% от 261 = 3654,00 1400% от 391 = 5474,00
      1400% от 2 = 28,00 1400% от 132 = 1848,00 1400% от 262 = 3668,00 1400% от 392 = 5488,00
      1400% от 3 = 42,00 1400% от 133 = 1862,00 1400% от 263 = 3682,00 1400% от 393 = 5502,00
      1400% от 4 = 56,00 1400% от 134 = 1876,00 1400% от 264 = 3696,00 1400% от 394 = 5516,00
      1400% от 5 = 70,00 1400% от 135 = 1890,00 1400% от 265 = 3710,00 1400% от 395 = 5530,00
      1400% от 6 = 84,00 1400% от 136 = 1904,00 1400% от 266 = 3724,00 1400% от 396 = 5544,00
      1400% от 7 = 98,00 1400% от 137 = 1918,00 1400% от 267 = 3738,00 1400% от 397 = 5558,00
      1400% от 8 = 112,00 1400% от 138 = 1932,00 1400% от 268 = 3752,00 1400% от 398 = 5572,00
      1400% от 9 = 126,00 1400% от 139 = 1946,00 1400% от 269 = 3766,00 1400% от 399 = 5586,00
      1400% от 10 = 140,00 1400% от 140 = 1960,00 1400% от 270 = 3780,00 1400% от 400 = 5600,00
      1400% от 11 = 154,00 1400% от 141 = 1974,00 1400% от 271 = 3794,00 1400% от 401 = 5614,00
      1400% от 12 = 168,00 1400% от 142 = 1988,00 1400% от 272 = 3808,00 1400% от 402 = 5628,00
      1400% от 13 = 182,00 1400% от 143 = 2002,00 1400% от 273 = 3822,00 1400% от 403 = 5642,00
      1400% от 14 = 196,00 1400% от 144 = 2016,00 1400% от 274 = 3836,00 1400% от 404 = 5656,00
      1400% от 15 = 210,00 1400% от 145 = 2030,00 1400% от 275 = 3850,00 1400% от 405 = 5670,00
      1400% от 16 = 224,00 1400% от 146 = 2044,00 1400% от 276 = 3864,00 1400% от 406 = 5684,00
      1400% от 17 = 238,00 1400% от 147 = 2058,00 1400% от 277 = 3878,00 1400% от 407 = 5698,00
      1400% от 18 = 252,00 1400% от 148 = 2072,00 1400% от 278 = 3892,00 1400% от 408 = 5712,00
      1400% от 19 = 266,00 1400% от 149 = 2086,00 1400% от 279 = 3906,00 1400% от 409 = 5726,00
      1400% от 20 = 280,00 1400% от 150 = 2100,00 1400% от 280 = 3920,00 1400% от 410 = 5740,00
      1400% от 21 = 294,00 1400% от 151 = 2114,00 1400% от 281 = 3934,00 1400% от 411 = 5754,00
      1400% от 22 = 308,00 1400% от 152 = 2128,00 1400% от 282 = 3948,00 1400% от 412 = 5768,00
      1400% от 23 = 322,00 1400% от 153 = 2142,00 1400% от 283 = 3962,00 1400% от 413 = 5782,00
      1400% от 24 = 336,00 1400% от 154 = 2156,00 1400% от 284 = 3976,00 1400% от 414 = 5796,00
      1400% от 25 = 350,00 1400% от 155 = 2170,00 1400% от 285 = 3990,00 1400% от 415 = 5810,00
      1400% от 26 = 364,00 1400% от 156 = 2184,00 1400% от 286 = 4004,00 1400% от 416 = 5824,00
      1400% от 27 = 378,00 1400% от 157 = 2198,00 1400% от 287 = 4018,00 1400% от 417 = 5838,00
      1400% от 28 = 392,00 1400% от 158 = 2212,00 1400% от 288 = 4032,00 1400% от 418 = 5852,00
      1400% от 29 = 406,00 1400% от 159 = 2226,00 1400% от 289 = 4046,00 1400% от 419 = 5866,00
      1400% от 30 = 420,00 1400% от 160 = 2240,00 1400% от 290 = 4060,00 1400% от 420 = 5880,00
      1400% от 31 = 434,00 1400% от 161 = 2254,00 1400% от 291 = 4074,00 1400% от 421 = 5894,00
      1400% от 32 = 448,00 1400% от 162 = 2268,00 1400% от 292 = 4088,00 1400% от 422 = 5908,00
      1400% от 33 = 462,00 1400% от 163 = 2282,00 1400% от 293 = 4102,00 1400% от 423 = 5922,00
      1400% от 34 = 476,00 1400% от 164 = 2296,00 1400% от 294 = 4116,00 1400% от 424 = 5936,00
      1400% от 35 = 490,00 1400% от 165 = 2310,00 1400% от 295 = 4130,00 1400% от 425 = 5950,00
      1400% от 36 = 504,00 1400% от 166 = 2324,00 1400% от 296 = 4144,00 1400% от 426 = 5964,00
      1400% от 37 = 518,00 1400% от 167 = 2338,00 1400% от 297 = 4158,00 1400% от 427 = 5978,00
      1400% от 38 = 532,00 1400% от 168 = 2352,00 1400% от 298 = 4172,00 1400% от 428 = 5992,00
      1400% от 39 = 546,00 1400% от 169 = 2366,00 1400% от 299 = 4186,00 1400% от 429 = 6006,00
      1400% от 40 = 560,00 1400% от 170 = 2380,00 1400% от 300 = 4200,00 1400% от 430 = 6020,00
      1400% от 41 = 574,00 1400% от 171 = 2394,00 1400% от 301 = 4214,00 1400% от 431 = 6034,00
      1400% от 42 = 588,00 1400% от 172 = 2408,00 1400% от 302 = 4228,00 1400% от 432 = 6048,00
      1400% от 43 = 602,00 1400% от 173 = 2422,00 1400% от 303 = 4242,00 1400% от 433 = 6062,00
      1400% от 44 = 616,00 1400% от 174 = 2436,00 1400% от 304 = 4256,00 1400% от 434 = 6076,00
      1400% от 45 = 630,00 1400% от 175 = 2450,00 1400% от 305 = 4270,00 1400% от 435 = 6090,00
      1400% от 46 = 644,00 1400% от 176 = 2464,00 1400% от 306 = 4284,00 1400% от 436 = 6104,00
      1400% от 47 = 658,00 1400% от 177 = 2478,00 1400% от 307 = 4298,00 1400% от 437 = 6118,00
      1400% от 48 = 672,00 1400% от 178 = 2492,00 1400% от 308 = 4312,00 1400% от 438 = 6132,00
      1400% от 49 = 686,00 1400% от 179 = 2506,00 1400% от 309 = 4326,00 1400% от 439 = 6146,00
      1400% от 50 = 700,00 1400% от 180 = 2520,00 1400% от 310 = 4340,00 1400% от 440 = 6160,00
      1400% от 51 = 714,00 1400% от 181 = 2534,00 1400% от 311 = 4354,00 1400% от 441 = 6174,00
      1400% от 52 = 728,00 1400% от 182 = 2548,00 1400% от 312 = 4368,00 1400% от 442 = 6188,00
      1400% от 53 = 742,00 1400% от 183 = 2562,00 1400% от 313 = 4382,00 1400% от 443 = 6202,00
      1400% от 54 = 756,00 1400% от 184 = 2576,00 1400% от 314 = 4396,00 1400% от 444 = 6216,00
      1400% от 55 = 770,00 1400% от 185 = 2590,00 1400% от 315 = 4410,00 1400% от 445 = 6230,00
      1400% от 56 = 784,00 1400% от 186 = 2604,00 1400% от 316 = 4424,00 1400% от 446 = 6244,00
      1400% от 57 = 798,00 1400% от 187 = 2618,00 1400% от 317 = 4438,00 1400% от 447 = 6258,00
      1400% от 58 = 812,00 1400% от 188 = 2632,00 1400% от 318 = 4452,00 1400% от 448 = 6272,00
      1400% от 59 = 826,00 1400% от 189 = 2646,00 1400% от 319 = 4466,00 1400% от 449 = 6286,00
      1400% от 60 = 840,00 1400% от 190 = 2660,00 1400% от 320 = 4480,00 1400% от 450 = 6300,00
      1400% от 61 = 854,00 1400% от 191 = 2674,00 1400% от 321 = 4494,00 1400% от 451 = 6314,00
      1400% от 62 = 868,00 1400% от 192 = 2688,00 1400% от 322 = 4508,00 1400% от 452 = 6328,00
      1400% от 63 = 882,00 1400% от 193 = 2702,00 1400% от 323 = 4522,00 1400% от 453 = 6342,00
      1400% от 64 = 896,00 1400% от 194 = 2716,00 1400% от 324 = 4536,00 1400% от 454 = 6356,00
      1400% от 65 = 910,00 1400% от 195 = 2730,00 1400% от 325 = 4550,00 1400% от 455 = 6370,00
      1400% от 66 = 924,00 1400% от 196 = 2744,00 1400% от 326 = 4564,00 1400% от 456 = 6384,00
      1400% от 67 = 938,00 1400% от 197 = 2758,00 1400% от 327 = 4578,00 1400% от 457 = 6398,00
      1400% от 68 = 952,00 1400% от 198 = 2772,00 1400% от 328 = 4592,00 1400% от 458 = 6412,00
      1400% от 69 = 966,00 1400% от 199 = 2786,00 1400% от 329 = 4606,00 1400% от 459 = 6426,00
      1400% от 70 = 980,00 1400% от 200 = 2800,00 1400% от 330 = 4620,00 1400% от 460 = 6440,00
      1400% от 71 = 994,00 1400% от 201 = 2814,00 1400% от 331 = 4634,00 1400% от 461 = 6454,00
      1400% от 72 = 1008,00 1400% от 202 = 2828,00 1400% от 332 = 4648,00 1400% от 462 = 6468,00
      1400% от 73 = 1022,00 1400% от 203 = 2842,00 1400% от 333 = 4662,00 1400% от 463 = 6482,00
      1400% от 74 = 1036,00 1400% от 204 = 2856,00 1400% от 334 = 4676,00 1400% от 464 = 6496,00
      1400% от 75 = 1050,00 1400% от 205 = 2870,00 1400% от 335 = 4690,00 1400% от 465 = 6510,00
      1400% от 76 = 1064,00 1400% от 206 = 2884,00 1400% от 336 = 4704,00 1400% от 466 = 6524,00
      1400% от 77 = 1078,00 1400% от 207 = 2898,00 1400% от 337 = 4718,00 1400% от 467 = 6538,00
      1400% от 78 = 1092,00 1400% от 208 = 2912,00 1400% от 338 = 4732,00 1400% от 468 = 6552,00
      1400% от 79 = 1106,00 1400% от 209 = 2926,00 1400% от 339 = 4746,00 1400% от 469 = 6566,00
      1400% от 80 = 1120,00 1400% от 210 = 2940,00 1400% от 340 = 4760,00 1400% от 470 = 6580,00
      1400% от 81 = 1134,00 1400% от 211 = 2954,00 1400% от 341 = 4774,00 1400% от 471 = 6594,00
      1400% от 82 = 1148,00 1400% от 212 = 2968,00 1400% от 342 = 4788,00 1400% от 472 = 6608,00
      1400% от 83 = 1162,00 1400% от 213 = 2982,00 1400% от 343 = 4802,00 1400% от 473 = 6622,00
      1400% от 84 = 1176,00 1400% от 214 = 2996,00 1400% от 344 = 4816,00 1400% от 474 = 6636,00
      1400% от 85 = 1190,00 1400% от 215 = 3010,00 1400% от 345 = 4830,00 1400% от 475 = 6650,00
      1400% от 86 = 1204,00 1400% от 216 = 3024,00 1400% от 346 = 4844,00 1400% от 476 = 6664,00
      1400% от 87 = 1218,00 1400% от 217 = 3038,00 1400% от 347 = 4858,00 1400% от 477 = 6678,00
      1400% от 88 = 1232,00 1400% от 218 = 3052,00 1400% от 348 = 4872,00 1400% от 478 = 6692,00
      1400% от 89 = 1246,00 1400% от 219 = 3066,00 1400% от 349 = 4886,00 1400% от 479 = 6706,00
      1400% от 90 = 1260,00 1400% от 220 = 3080,00 1400% от 350 = 4900,00 1400% от 480 = 6720,00
      1400% от 91 = 1274,00 1400% от 221 = 3094,00 1400% от 351 = 4914,00 1400% от 481 = 6734,00
      1400% от 92 = 1288,00 1400% от 222 = 3108,00 1400% от 352 = 4928,00 1400% от 482 = 6748,00
      1400% от 93 = 1302,00 1400% от 223 = 3122,00 1400% от 353 = 4942,00 1400% от 483 = 6762,00
      1400% от 94 = 1316,00 1400% от 224 = 3136,00 1400% от 354 = 4956,00 1400% от 484 = 6776,00
      1400% от 95 = 1330,00 1400% от 225 = 3150,00 1400% от 355 = 4970,00 1400% от 485 = 6790,00
      1400% от 96 = 1344,00 1400% от 226 = 3164,00 1400% от 356 = 4984,00 1400% от 486 = 6804,00
      1400% от 97 = 1358,00 1400% от 227 = 3178,00 1400% от 357 = 4998,00 1400% от 487 = 6818,00
      1400% от 98 = 1372,00 1400% от 228 = 3192,00 1400% от 358 = 5012,00 1400% от 488 = 6832,00
      1400% от 99 = 1386,00 1400% от 229 = 3206,00 1400% от 359 = 5026,00 1400% от 489 = 6846,00
      1400% от 100 = 1400,00 1400% от 230 = 3220,00 1400% от 360 = 5040,00 1400% от 490 = 6860,00
      1400% от 101 = 1414,00 1400% от 231 = 3234,00 1400% от 361 = 5054,00 1400% от 491 = 6874,00
      1400% от 102 = 1428,00 1400% от 232 = 3248,00 1400% от 362 = 5068,00 1400% от 492 = 6888,00
      1400% от 103 = 1442,00 1400% от 233 = 3262,00 1400% от 363 = 5082,00 1400% от 493 = 6902,00
      1400% от 104 = 1456,00 1400% от 234 = 3276,00 1,400% от 364 = 5,096.

      Arcsin x arcsin y: Элементарная математика

      Mathway | Популярные задачи

      1Найти точное значениеsin(30)
      2Найти точное значениеsin(45)
      3Найти точное значениеsin(30 град. )
      4Найти точное значениеsin(60 град. )
      5Найти точное значениеtan(30 град. )
      6Найти точное значениеarcsin(-1)
      7Найти точное значениеsin(pi/6)
      8Найти точное значениеcos(pi/4)
      9Найти точное значениеsin(45 град. )
      10Найти точное значениеsin(pi/3)
      11Найти точное значениеarctan(-1)
      12Найти точное значениеcos(45 град. )
      13Найти точное значениеcos(30 град. )
      14Найти точное значениеtan(60)
      15Найти точное значениеcsc(45 град. )
      16Найти точное значениеtan(60 град. )
      17Найти точное значениеsec(30 град. )
      18Найти точное значениеcos(60 град. )
      19Найти точное значениеcos(150)
      20Найти точное значениеsin(60)
      21Найти точное значениеcos(pi/2)
      22Найти точное значениеtan(45 град. )
      23Найти точное значениеarctan(- квадратный корень из 3)
      24Найти точное значениеcsc(60 град. )
      25Найти точное значениеsec(45 град. )
      26Найти точное значениеcsc(30 град. )
      27Найти точное значениеsin(0)
      28Найти точное значениеsin(120)
      29Найти точное значениеcos(90)
      30Преобразовать из радианов в градусыpi/3
      31Найти точное значениеtan(30)
      32Преобразовать из градусов в радианы45
      33Найти точное значениеcos(45)
      34Упроститьsin(theta)^2+cos(theta)^2
      35Преобразовать из радианов в градусыpi/6
      36Найти точное значениеcot(30 град. )
      37Найти точное значениеarccos(-1)
      38Найти точное значениеarctan(0)
      39Найти точное значениеcot(60 град. )
      40Преобразовать из градусов в радианы30
      41Преобразовать из радианов в градусы(2pi)/3
      42Найти точное значениеsin((5pi)/3)
      43Найти точное значениеsin((3pi)/4)
      44Найти точное значениеtan(pi/2)
      45Найти точное значениеsin(300)
      46Найти точное значениеcos(30)
      47Найти точное значениеcos(60)
      48Найти точное значениеcos(0)
      49Найти точное значениеcos(135)
      50Найти точное значениеcos((5pi)/3)
      51Найти точное значениеcos(210)
      52Найти точное значениеsec(60 град. )
      53Найти точное значениеsin(300 град. )
      54Преобразовать из градусов в радианы135
      55Преобразовать из градусов в радианы150
      56Преобразовать из радианов в градусы(5pi)/6
      57Преобразовать из радианов в градусы(5pi)/3
      58Преобразовать из градусов в радианы89 град.
      59Преобразовать из градусов в радианы60
      60Найти точное значениеsin(135 град. )
      61Найти точное значениеsin(150)
      62Найти точное значениеsin(240 град. )
      63Найти точное значениеcot(45 град. )
      64Преобразовать из радианов в градусы(5pi)/4
      65Найти точное значениеsin(225)
      66Найти точное значениеsin(240)
      67Найти точное значениеcos(150 град. )
      68Найти точное значениеtan(45)
      69Вычислитьsin(30 град. )
      70Найти точное значениеsec(0)
      71Найти точное значениеcos((5pi)/6)
      72Найти точное значениеcsc(30)
      73Найти точное значениеarcsin(( квадратный корень из 2)/2)
      74Найти точное значениеtan((5pi)/3)
      75Найти точное значениеtan(0)
      76Вычислитьsin(60 град. )
      77Найти точное значениеarctan(-( квадратный корень из 3)/3)
      78Преобразовать из радианов в градусы(3pi)/4
      79Найти точное значениеsin((7pi)/4)
      80Найти точное значениеarcsin(-1/2)
      81Найти точное значениеsin((4pi)/3)
      82Найти точное значениеcsc(45)
      83Упроститьarctan( квадратный корень из 3)
      84Найти точное значениеsin(135)
      85Найти точное значениеsin(105)
      86Найти точное значениеsin(150 град. )
      87Найти точное значениеsin((2pi)/3)
      88Найти точное значениеtan((2pi)/3)
      89Преобразовать из радианов в градусыpi/4
      90Найти точное значениеsin(pi/2)
      91Найти точное значениеsec(45)
      92Найти точное значениеcos((5pi)/4)
      93Найти точное значениеcos((7pi)/6)
      94Найти точное значениеarcsin(0)
      95Найти точное значениеsin(120 град. )
      96Найти точное значениеtan((7pi)/6)
      97Найти точное значениеcos(270)
      98Найти точное значениеsin((7pi)/6)
      99Найти точное значениеarcsin(-( квадратный корень из 2)/2)
      100Преобразовать из градусов в радианы88 град.

      Элементарная математика

        

      Сканави М.И. Элементарная математика. 2-е изд., перераб. и доп., М.: 1974г. — 592с.

      Книга представляет собой повторительный курс элементарной математики и рассчитана на тех, кто хочет пополнить, укрепить и систематизировать свои знания. Как и в первом издании, содержание ориентировано на программы вступительных экзаменов в технические вузы и, в особенности, на программы подготовительных отделений при высших учебных заведениях, для учащихся которых, как мы надеемся, книга окажется полезной.

      (Книга включает в себя Ч1 — Арифметика, алгебра и элементарные функции и Ч2 — Геометрия. Каждый раздел включает в себя теоретическую часть и большое количество задач с решениями.)



      Оглавление

      ВВЕДЕНИЕ
      Часть первая. АРИФМЕТИКА, АЛГЕБРА И ЭЛЕМЕНТАРНЫЕ ФУНКЦИИ
      Глава I. ДЕЙСТВИТЕЛЬНЫЕ И КОМПЛЕКСНЫЕ ЧИСЛА
      2. Простые и составные числа. Признаки делимости.
      3. Наибольший общий делитель и наименьшее общее кратное.
      4. Целые числа. Рациональные числа.
      5. Десятичные дроби. Представление рациональных чисел десятичными дробями.
      6. Иррациональные числа. Действительные числа.
      7. Действия с приближенными числами.
      8. Числовая ось. Координаты точки на плоскости.
      § 2. Степени и корни
      9. Степени с натуральными показателями.
      10. Степени с целыми показателями.
      11. Корни.
      12. Степени с рациональными показателями. Степени с действительными показателями.
      13. Алгоритм извлечения квадратного корня.
      § 3. Комплексные числа
      14. Основные понятия и определения.
      15. Рациональные действия с комплексными числами.
      16. Геометрическое изображение комплексных чисел. Тригонометрическая форма комплексного числа.
      17. Действия с комплексными числами, заданными в тригонометрической форме. Формула Муавра.
      18. Извлечение корня из комплексного числа.
      Глава II. ТОЖДЕСТВЕННЫЕ ПРЕОБРАЗОВАНИЯ
      19. Алгебраические выражения. Одночлены и многочлены.
      20. Формулы сокращенного умножения.
      21. Бином Ньютона. n.
      41. Обратная пропорциональная зависимость. Степенная функция с рациональным показателем степени.
      42. Показательная функция.
      43. Логарифмическая функция.
      § 3. Преобразование графиков
      44. Параллельный сдвиг графика.
      45. График квадратного трех члена.
      46. График дробно-линейной функции.
      47. Преобразование симметрии. Сжатие и растяжение графика.
      48. Построение графиков функций.
      49. Сложение графиков.
      § 4. Некоторые сведения о рациональных функциях
      50. Целые и дробные рациональные функции. Деление многочленов.
      51. Схема Горнера. Теорема Безу.
      52. Нули многочлена. Разложение многочлена на множители.
      Глава V. УРАВНЕНИЯ
      53. Уравнение. Корни уравнения.
      54. Равносильные уравнения.
      55. Системы уравнений.
      56. Графическое решение уравнений.
      §. 2. Алгебраические уравнения с одной неизвестной
      57. Число и кратность корней.
      58. Уравнения первой степени (линейные уравнения).
      59. Уравнения второй степени (квадратные уравнения).
      60. Формулы Виета. Разложение квадратного трехчлена на множители.
      61. Исследование квадратного уравнения.
      62. Уравнения высших степеней. Целые корни.
      63. Двучленные уравнения.
      64. Уравнения, сводящиеся к квадратным.
      65. Возвратные уравнения.
      § 3. Системы алгебраических уравнений
      66. Линейные системы.
      67. Определители второго порядка. Исследование линейных систем двух уравнений с двумя неизвестными.
      68. Системы, состоящие из уравнения второй степени и линейного уравнения.
      69. Примеры систем двух уравнений второй степени. Системы уравнений высших степеней.
      § 4. Иррациональные, показательные и логарифмические уравнения
      70. Иррациональные уравнения.
      71. Показательные уравнения.
      72. Логарифмические уравнения.
      73. Разные уравнения. Системы уравнений.
      Глава VI. НЕРАВЕНСТВА
      74. Свойства неравенств. Действия над неравенствами.
      75. Алгебраические неравенства.
      § 2. Решение неравенств
      76. Множество решений неравенства. Равносильные неравенства.
      77. Графическое решение неравенств.
      79. Квадратные неравенства.
      80. Неравенства высших степеней. Неравенства, содержащие дробные рациональные функции от х.
      81. Иррациональные, показательные и логарифмические неравенства.
      82. Неравенства с двумя неизвестными.
      Глава VII. ПОСЛЕДОВАТЕЛЬНОСТИ
      83. Числовая последовательность.
      84. Предел числовой последовательности.
      85. Бесконечно малые. Правила предельного перехода.
      § 2. Арифметическая прогрессия
      86. Арифметическая прогрессия. Формула общего члена.
      87. Свойства арифметической прогрессии.
      88. Формула для суммы n членов арифметической прогрессии.
      § 3. Геометрическая прогрессия
      89. Геометрическая прогрессия. Формула общего члена.
      90. Свойства геометрической прогрессии.
      91. Формулы для суммы n членов геометрической прогрессии.
      92. Бесконечно убывающая геометрическая прогрессия.
      Глава VIII. ТРИГОНОМЕТРИЧЕСКИЕ ФУНКЦИИ УГЛА (ДУГИ)
      93. Вектор, проекция вектора.
      94. Положительные углы и дуги, меньшие 360°.
      95. Углы и дуги, большие 360°.
      96. Отрицательные углы. Сложение и вычитание углов.
      § 2. Тригонометрические функции произвольного угла
      97. Определение основных тригонометрических функций.
      98. Изменение основных тригонометрических функций при изменении угла от 0 до 2pi.
      § 3. Соотношения между тригонометрическими функциями одного и того же угла
      99. Основные тригонометрические тождества.
      100. Вычисление значений тригонометрических функций по значению одной из них.
      101. Значения тригонометрических функций некоторых углов.
      § 4. Четность, нечетность и периодичность тригонометрических функций
      102. Четность и нечетность.
      103. Понятие периодической функции.
      104. Периодичность тригонометрических функций.
      § 5. Формулы приведения
      105. Зависимость между тригонометрическими функциями дополнительных углов.
      106. Формулы приведения.
      Глава IX. ТРИГОНОМЕТРИЧЕСКИЕ ФУНКЦИИ ЧИСЛОВОГО АРГУМЕНТА И ИХ ГРАФИКИ
      § 1. Тригонометрические функции числового аргумента
      108. Области определения и области изменения значений тригонометрических функций.
      109. Некоторые неравенства и их следствия.
      § 2. Графики тригонометрических функций
      110. Первоначальные сведения о таблицах тригонометрических функций.
      111. Основные графики.
      112. Примеры построения графиков некоторых других тригонометрических функций.
      113. Дальнейшие примеры построения графиков функций.
      Глава X. ПРЕОБРАЗОВАНИЕ ТРИГОНОМЕТРИЧЕСКИХ ВЫРАЖЕНИЙ
      114. Расстояние между двумя точками на плоскости.
      115. Косинус суммы и разности двух аргументов.
      116. Синус суммы и разности двух аргументов.
      117. Тангенс суммы и разности двух аргументов.
      118. О формулах сложения для нескольких аргументов.
      § 2. Формулы для двойного и половинного аргумента. Выражение sin na и cos na через степени sin a и cos a
      119. Тригонометрические функции двойного аргумента.
      120. Выражение sin na и cos na через степени sin a и cos a при натуральном числе n.
      121. Тригонометрические функции половинного аргумента.
      122. Выражение основных тригонометрических функций аргумента а через tg(a/2).
      § 3. Преобразование в сумму выражений вида sina•cosb, cosa•cosb и sinа•sinb
      § 4. Преобразование в произведение сумм вида
      § 5. Преобразование некоторых выражений в произведения с помощью введения вспомогательного аргумента
      127. Преобразование в произведение выражения a•sina + b•cosa.
      128. Преобразование в произведение выражений a•sina+b и a•cosa+b
      129. Преобразование в произведение выражения a•tga+b.
      Глава XI. ОБРАТНЫЕ ТРИГОНОМЕТРИЧЕСКИЕ ФУНКЦИИ И ИХ ГРАФИКИ
      130. Функция у = arcsin x (арксинус).
      131. Функция y = arccos x (арккосинус).
      132. Функция y = arctg x (арктангенс).
      133. Функция y = arcctg x (арккотангенс).
      134. Пример.
      § 2. Операции над обратными тригонометрическими функциями
      135. Тригонометрические операции.
      136. Операции сложения (вычитания).
      § 3. Обратные тригонометрические операции над тригонометрическими функциями
      137. Функция у = arcsin (sin x).
      138. Функция y = arctg (tg x).
      Глава XII. ТРИГОНОМЕТРИЧЕСКИЕ УРАВНЕНИЯ И НЕРАВЕНСТВА
      139. Уравнение sin х = а.
      140. Уравнение cos х = a.
      141. Уравнение tg x = a.
      142. Уравнение ctg x = a.
      143. Некоторые дополнения.
      § 2. Способ приведения к одной функции одного и того же аргумента
      145. Некоторые типы уравнений, приводящихся к уравнениям относительно функции одного аргумента.
      146. Способ разложения на множители.
      147. Решение рациональных тригонометрических уравнений с помощью универсальной тригонометрической подстановки tg(x/2) = t.
      § 3. Некоторые частные приемы решения тригонометрических уравнений и систем
      148. Введение вспомогательного аргумента.
      149. Преобразование произведения в сумму или разность.
      150. Переход к функциям удвоенного аргумента.
      151. Решение уравнения типа…
      152. Применение подстановок sinx ± соsx = y.
      § 4. Решение тригонометрических неравенств
      154. Простейшие тригонометрические неравенства.
      155. Примеры тригонометрических неравенств, сводящихся к простейшим.
      Часть вторая. ГЕОМЕТРИЯ
      156. Точка. Прямая. Луч. Отрезок.
      157. Плоскость. Фигуры и тела.
      160. Равенство фигур. Движение.
      161. Равенство тел.
      § 2. Измерение геометрических величин
      162. Сложение отрезков. Длина отрезка.
      163. Общая мера двух отрезков.
      164. Сравнительная длина отрезков и ломаных.
      165. Измерение углов.
      166. Радианная мера угла.
      167. Измерение площадей.
      168. Площадь прямоугольника. Объем прямоугольного параллелепипеда.
      Глава XIV. ПЕРПЕНДИКУЛЯРНЫЕ И ПАРАЛЛЕЛЬНЫЕ ПРЯМЫЕ. ЗАДАЧИ НА ПОСТРОЕНИЕ
      169. Перпендикуляр и наклонные.
      170. Свойство перпендикуляра, проведенного к отрезку в его середине.
      171. Параллельные прямые.
      172. Углы, образованные двумя параллельными прямыми и секущей.
      173. Углы с параллельными или перпендикулярными сторонами.
      § 2. Геометрические места точек. Окружность
      174. Геометрическое место точек.
      175. Свойство биссектрисы угла.
      176. Окружность.
      177. Взаимное расположение прямой и окружности. Касательная и секущая.
      178. Хорда и диаметр. Сектор и сегмент.
      179. Взаимное расположение двух окружностей.
      § 3. Основные задачи на построение
      181. Деление отрезка пополам. Построение перпендикуляров.
      182. Построение углов.
      183. Другие задачи на построение.
      Глава XV. ТРЕУГОЛЬНИКИ, ЧЕТЫРЕХУГОЛЬНИКИ
      184. Стороны и углы треугольника.
      185. Биссектрисы треугольника. Вписанная окружность.
      186. Оси симметрии сторон треугольника. Описанная окружность.
      187. Медианы и выcоты треугольника.
      188. Равенство треугольников.
      189. Построение треугольников.
      190. Равнобедренные треугольники.
      191. Прямоугольные треугольники.
      § 2. Параллелограммы
      192. Четырехугольники.
      193. Параллелограмм и его свойства.
      194. Прямоугольник.
      § 3. Трапеция
      196. Трапеция.
      197. Средняя линия треугольника.
      198. Средняя линия трапеции.
      199. Деление отрезка на равные части.
      § 4. Площади треугольников и четырехугольников
      200. Площадь параллелограмма.
      201. Площадь треугольника.
      202. Площадь трапеции.
      Глава XVI. ПОДОБИЕ ГЕОМЕТРИЧЕСКИХ ФИГУР
      203. Пропорциональные отрезки.
      204. Свойства биссектрис внутреннего и внешнего углов треугольника.
      § 2. Подобное преобразование фигур (гомотетия)
      205. Определение гомотетичных фигур.
      206. Свойства преобразования подобия.
      § 3. Общее подобное соответствие фигур
      207. Подобные фигуры.
      208. Периметры и площади подобных треугольников.
      209. Применение подобия к решению задач на построение.
      Глава XVII. МЕТРИЧЕСКИЕ СООТНОШЕНИЯ В ТРЕУГОЛЬНИКЕ И КРУГЕ
      210. Углы с вершиной на окружности.
      211. Углы с вершиной внутри и вне круга.
      212. Угол, под которым виден данный отрезок.
      213. Четырехугольники, вписанные в окружность.
      214. Пропорциональные отрезки в круге.
      215. Задачи на построение.
      § 2. Метрические соотношения в треугольнике
      216. Пропорциональные отрезки в прямоугольном треугольнике. Теорема Пифагора.
      218. Теорема синусов. Формула Герона.
      217. Квадрат стороны, лежащей против острого или тупого утла и треугольнике. Теорема косинусов.
      218. Теорема синусов. Формула Герона.
      219. Радиусы вписанной и описанной окружностей.
      § 3. Решение треугольников
      220. Таблицы функций.
      221. Решение треугольников. Сводка основных формул.
      222. Решение прямоугольных треугольников.
      223. Решение косоугольных треугольников.
      Глава XVIII. ПРАВИЛЬНЫЕ МНОГОУГОЛЬНИКИ. ДЛИНА окружности И ПЛОЩАДЬ КРУГА
      224. Выпуклые многоугольники.
      225. Правильные многоугольники.
      226. Соотношения между стороной, радиусом и апофемой.
      227. Периметр и площадь правильного n-угольника.
      228. Удвоение числа сторон правильного многоугольника.
      § 2. Длина окружности. Площадь круга и его частей
      229. Длина окружности.
      230. Площадь круга и его частей.
      Глава XIX. ПРЯМЫЕ И ПЛОСКОСТИ В ПРОСТРАНСТВЕ
      231. Взаимное расположение двух прямых в пространстве.
      232. Взаимное расположение прямой линии и плоскости.
      233. Взаимное расположение двух плоскостей.
      234. Свойства параллельных прямых и плоскостей.
      235. Построения в стереометрии.
      § 2. Перпендикулярность прямых и плоскостей
      236. Перпендикуляр к плоскости.
      237. Перпендикуляр и наклонные.
      238. Угол между прямой и плоскостью.
      239. Связь между перпендикулярностью и параллельностью прямых и плоскостей.
      240. Общий перпендикуляр двух скрещивающихся прямых.
      § 3. Двугранные и многогранные углы
      241. Двугранный угол.
      242. Взаимно перпендикулярные плоскости.
      243. Трехгранные углы.
      244. Многогранные углы.
      § 4. Многогранники
      245. Многогранники.
      246. Правильные многогранники.
      Глава XX. МНОГОГРАННИКИ И КРУГЛЫЕ ТЕЛА
      247. Цилиндры и призмы.
      248. Параллелепипеды.
      249. Объемы призм и цилиндров.
      250. Площадь боковой поверхности призмы.
      251. Площадь поверхности цилиндра.
      § 2. Пирамида. Конус
      252. Свойства пирамиды и конуса.
      253. Объем пирамиды и конуса.
      254. Площадь боковой поверхности правильной пирамиды и конуса.
      255. Усеченный конус и усеченная пирамида.
      § 3. Шаровая поверхность. Шар
      256. Шар и шаровая поверхность.
      257. Объем шара и его частей.
      258. Площадь поверхности шара и ее частей.
      259. Понятие телесного угла.
      Ответы к упражнениям
      Приложения

      тригонометрия — показывает, что $\arcsin x + \arccos y = \frac{\pi}{2}$ тогда и только тогда, когда $x = y$

      Задавать вопрос

      спросил

      Изменено 3 года, 11 месяцев назад

      Просмотрено 1к раз

      $\begingroup$

      Мне просто интересно узнать об этой личности:

      $$\arcsin x + \arccos x = \frac{\pi}{2} . $$

      Что мне пришла в голову мысль, что вообще $$\arcsin x + \arccos y = \frac{\pi}{2} \qquad \textrm{если и только если} \qquad x = y .$$ У меня есть предчувствие, что это правда, и я сделал самоудовлетворительное, но незаконное доказательство, используя пробную версию, а позже я также попытался использовать графики, но я все еще застрял.

      • тригонометрия
      • занимательная математика
      • обратная функция

      $\endgroup$

      2

      $\begingroup$

      $$arcsin (x)+arccos(y)=\frac{\pi}{2}$$ $$\Longleftrightarrow arccos(y)=\frac{\pi}{2}-arcsin (x)$$ $$\Longleftrightarrow arccos(y)=arccos(x)$$ $$\Longleftrightarrow y=x$$ Последний шаг следует из того, что $arccos(x)$ обратим. 9{-1}{y} = \pi/2$. Это стандартный результат. См., например. здесь для доказательства.

      $\endgroup$

      $\begingroup$

      Подсказка Для оставшегося направления используйте тождество $$\arcsin x + \arccos x = \frac{\pi}{2}$$ вместе с тем фактом, что $x \mapsto \arcsin x$ строго возрастает.

      $\endgroup$

      $\begingroup$

      Если $x=y$, то рассмотрим прямоугольный треугольник, длина одной стороны которого составляет $1$, а длина другой стороны — $x$ единиц. Тогда $\arcsin(x)+\arccos(x)$ просто складывает два непрямых угла, так что сумма будет $\frac{\pi}{2}$. А так как мы предположили здесь $x=y$, то получаем $\arcsin(x)+\arccos(y)=\frac{\pi}{2}$. 92)}=1.$$

      • тригонометрия

      $\endgroup$

      $\begingroup$

      $\arcsin x=\dfrac\pi2-\arcsin y=\arccos y$

      $\arcsin x=\arccos y=A$ (скажем)

      $\имплициты x=\sin A,y=\ cos A$

      Надеюсь, вы сможете взять его отсюда!

      $\endgroup$

      2

      $\begingroup$

      Возьмите синус обеих сторон для прямой демонстрации.

      © 2015 - 2019 Муниципальное казённое общеобразовательное учреждение «Таловская средняя школа»

      Карта сайта